You are on page 1of 180

Current Affairs- Subject wise compilation of 60 Day Plan- 2016

Q.1) Consider the following statements with respect to Asiatic Lion


1. The Gir Forest, in Gujarat, is the last natural range wild Asiatic lions.
2. It is critically endangered as per IUCN red list
Select the correct answer
a) 1 only
b) 2 only
c) Both
d) None
Q.1) Solution (a)
It is endangered as per IUCN red list
http://www.asiaticlion.org/asiatic-lion-history.htm
Q.2) India successfully test-fires advanced interceptor missile from Abdul kalam Island.
Select the correct option with respect to Abdul kalam Island.
a) It is the new name of Sri Hari Kota located in Andhra Pradesh.
b) It is located in French Guinea
c) It is an island located off Odisha coast.
d) It is located in Andaman and Nicobar islands.
Q.2) Solution (c)
Abdul Kalam Island, formerly known as Wheeler Island, is an island off the coast of Odisha,
India, approximately 150 kilometres (93 mi) from the state capital Bhubaneswar. The
Integrated Test Range maintaining a missile testing facility is located in the island.
http://www.thehindu.com/news/national/india-successfully-testfires-advanced-interceptormissile/article8604747.ece?homepage=true
Q.3) Consider the following statements with respect to Porpoises
1. Porpoises are group of Marine amphibians
2. Vaquita marina is the worlds smallest porpoise
3. They are critically endangered
4. They are endemic to Gulf of California and Mexico
Select the correct option
a) 1, 2 and 3 only
b) 2, 3 and 4 only
c) 1, 3 and 4 only
d) 1,2,3 and 4 only
Q.3) Solution (b)

www.iasbaba.com

Current Affairs- Subject wise compilation of 60 Day Plan- 2016


Porpoises are a group of fully aquatic marine mammals.
https://en.wikipedia.org/wiki/Porpoise
Q.4) Chinnar Wildlife Sanctuary accounted for a large number of fatalities over the past six
months. Chinnar Wild life sanctuary is located in which of the following state/UT of India
a) Karnataka
b) Tamil Nadu
c) Puducherry
d) Kerala
Q.4) Solution (d)
http://www.thehindu.com/sci-tech/energy-and-environment/conservation-suffers-asroadkills-in-chinnar-sanctuary-shoot-up/article8604749.ece
Q.5) Consider the following statements about the Fiscal Policy.
1. Fiscal policy is the use of government spending and taxation to influence the economy
2. The most immediate effect of fiscal policy is to change the aggregate demand for goods
and services.
3. Fiscal policy is said to be loose or expansionary when revenue is higher than spending.
Which of the above statements are correct?
a) 1 and 2 only
b) 2 and 3 only
c) 1 and 3 only
d) All
Q.5) Solution (a)
Fiscal policy is said to be tight or contractionary when revenue is higher than spending (i.e.,
the government budget is in surplus).
http://www.investopedia.com/articles/04/051904.asp
Q.6) The High Court of Rajasthan has declared the practice of Santhara as illegal. Which
of the statements correctly explains Santhara?
a) It is a ritual of sacrificing Bulls in front of Goddess Kali in local annual festival.
b) It is a Jain custom of voluntary death by performing fast until death.
c) It is the name given to the performance of Child marriage in Rajasthan.
d) It is a practice of honour killing of a couple if a girl and a boy of same village gets married
to each other.
Q.6) Solution (b)

www.iasbaba.com

Current Affairs- Subject wise compilation of 60 Day Plan- 2016


Santhara is a Jain custom of embracing voluntary death it involves practitioners taking an
oath to stop eating until they die of starvation. According to Jains, this is a way to purge
oneself of bad karma and attain moksha.
Every year a question comes from indigenous ancient religions. Since Santhara was in news
it becomes important. A related question on Jainism can be asked like- which famous
mauryan ruler is believed to have performed Santhara. (Chandragupta Maurya)
Q.7) Consider the following statements regarding the scheme Sakhi:
1. The scheme is meant to help women suffering from sexual, mental or emotional
harassments.
2. It is an initiative of Union Government under the Nirbhaya Fund for women safety.
3. Women will be provided vocational training to make them feel confident and earn
livelihood as a part of Skill India initiative.
Which of the above statements are correct?
a) 1 only
b) 1 and 2
c) 3 only
d) All of the above
Q.7) Solution (b)
A scheme called Sakhi, was inaugurated to help the women, who are suffering from sexual,
mental and emotional harassments (Including eve-teasing, domestic violence and rape). It is
an initiative of Union Government under the Nirbhaya Fund for women safety pertaining
to strategic areas of prevention, protection and rehabilitation. The Objective of this scheme
is to provide necessary help to women victims and others, who are suffering from any kind
of harassment.
Q.8) The National Aeronautics and Space Administration (NASA) and the US Agency for
International Development (USAID) have launched a new joint project named SERVIRMekong.
Which of the following statements regarding this project are correct?
1. The SERVIR program helps governments and development stakeholders in incorporating
Earth observations and geospatial technologies into natural disaster response to
improve food security, safeguard human health and manage water and natural
resources.
2. It is recently launched for five South Asian countries namely Burma, Cambodia, Laos,
Thailand and Vietnam.
Which of the above statements are correct?
a) 1 only

www.iasbaba.com

Current Affairs- Subject wise compilation of 60 Day Plan- 2016


b) 2 only
c) Both 1 and 2
d) Neither 1 nor 2
Q.8) Solution (a)
The National Aeronautics and Space Administration (NASA) and the US Agency for
International Development (USAID) have launched a new joint project named SERVIRMekong to strengthen regional environmental monitoring in five countries of Southeast
Asia namely Burma, Cambodia, Laos, Thailand and Vietnam.
It was launched in the Asian Disaster Preparedness Center in Bangkok, Thailand. Under
this project, researchers will draw from a continuous stream of space-based climate,
weather and other Earth observation data from Nasa and its partners. The main focus of
this project is to host vital issues in the Mekong river basin region in those five countries
which is also called as Rice Bowl of Asia.
Q.9) Consider the following statements:
1. Bank of Baroda has introduced Maharaja Sayyaji Rao Gaekwad award.
2. The award will be conferred to such personality every year which has made significant
contribution in creating awareness and spreading Marathi language amongst people.
Which of the above statements are incorrect?
a) 1 only
b) 2 only
c) Both 1 and 2
d) Neither 1 nor 2
Q.9) Solution (b)
Bank of Baroda has introduced Maharaja Sayajirao Bhasha Samman an award in the
memory of its founder Maharaja Sir Sayajirao Gaekwad III. Sayajirao Gaekwad III the
erstwhile ruler of Baroda state had played a vital role in economic growth of the country
and propagating the use of Hindi.
"The award will be conferred upon such a personality every year who has made significant
contribution for creating awareness and spreading Hindi among public at large,"
Q.10) Consider the following statements regarding Sugamya Bharat Abhiyan:
1. The campaign has been launched by Ministry of Social Justice and Empowerment.
2. The campaign focuses on making public buildings accessible to Persons with Disabilities
(PWDs).
3. The campaign also focuses on enhancing the accessibility and usability of Public
Transport, and information & communication.

www.iasbaba.com

Current Affairs- Subject wise compilation of 60 Day Plan- 2016

Which of the above statements are correct?


a) 1 and 2
b) 2 and 3
c) 1 and 3
d) All of the above
Q.10) Solution (d)
The Ministry of Social Justice and Empowerment launched the Accessible India Campaign
(Sugamya Bharat Abhiyan) that focuses on building accessible government buildings for
persons with disabilities (PwDs).
The campaign was initiated by the government to increase the accessibility of the physical
environment in the country. In addition, the campaign focuses on enhancing the
accessibility and usability of Public Transport, and information & communication.
Q.11) Consider the following statements and identify the personality?
1. 2015 marks 108th Birth Anniversary of this historical personality
2. He founded India nationalist youth organization, Naujawan Bharat Sabha
3. He was inspired by Anarchism and Marxism
Select the correct code
a) Udham Singh
b) Lala Lajpat Rai
c) Bhagat Singh
d) Chandra Shekhar Azad
Q.11) Solution (c)
Recently controversies like
1. Bipan Chandra book mentioning Bhagat Singh as Revolutionary Terrorist
2. Shashi Tharoor relating Kanhayiya and Rohit Vemula on the path of Bhagat Singh were in
news.
Prepare Bhagat Singh and his contributions in Freedom Struggle. Books written by him,
organizations, he was part of or established. His role models and ideological influences etc.
Bhagat Singh was born on 27 September 1907 at Banga in Lyallpur district (now Pakistan) to
Kishan Singh and Vidya Vati.
Q.12) ISRO has successfully launched ASTROSAT. Consider the following statements w.r.t
it
1. It is a kind of telescope

www.iasbaba.com

Current Affairs- Subject wise compilation of 60 Day Plan- 2016


2. It is Indias first Space Observatory
3. It is the first of its kind in the world
Select the correct code
a) 1 and 2
b) Only 2
c) 2 and 3
d) 1, 2 and 3
Q.12) Solution (a)
http://www.thehindu.com/sci-tech/astrosat-indias-unique-spaceobservatory/article7692305.ece
http://www.thehindu.com/sci-tech/science/isro-launches-astrosat-first-spaceobservatory/article7697707.ece
Q.13) Consider the following about Samvatsari
1. It is known as 'Introspection, is a process during which Jain repent for their sins
2. It is also known as International Forgiveness Day and is the holiest day of Jain Calendar
Chose the incorrect statement/s
a) Only 1
b) Only 2
c) Both
d) None
Q.13) Solution (a)
http://pib.nic.in/newsite/PrintRelease.aspx
Q.14) Consider the following
1. Nuakhai
2. Gudhi Parwa
3. Chavang Kut
4. Gawai Dayak
Which of the above is/are Agricultural/Harvest festivals of India?
a) Only 2
b) 1, 2 and 4
c) 1, 2 and 3
d) 2, 3 and 4
Q.14) Solution (c)

www.iasbaba.com

Current Affairs- Subject wise compilation of 60 Day Plan- 2016


Nuakhai- Oddisa
Chavang Kut- celebrated by the Kuki-chin group in North-east India on 1 November
Gudhi Parwa- elebrated by the Marathi people in Maharashtra, Karnataka
Gawai Dayak- Sarawak, Malaysia and West Kalimantan, Indonesia
Q.15) Bhama Kalapam, Parvathi Parinayam and Bhakta Prahlada are related to which
of the dance forms
a) Kathakali
b) Kuchipudi
c) Bharatnatyam
d) Odissi
Q.15) Solution (b)
Q.16) Consider the following
1. Atropa Belladona
2. Datura
3. Morphine
4. Opioids
Which of the following does not belong to the category of Drugs?
a) 1 and 3 only
b) 1 only
c) 2 and 3 only
d) None of these
Q.16) Solution (d)
http://www.ncert.nic.in/ncerts/l/lebo108.pdf
Q.17) Rio Olympics is round the corner. Consider the following statements
1. The official mascot of Rio 2016 Olympics is Vinicius
2. The Olympics will be held in Capital city of Brazil known as Rio De Generio
Select the correct answer
a) 1 only
b) 2 only
c) Both
d) None
Q.17) Solution (a)
The capital city of Brazil is known as Brasilia

www.iasbaba.com

Current Affairs- Subject wise compilation of 60 Day Plan- 2016

Q.18) Consider the following


1. Ringworms
2. Ascaris
3. Filariasis
Which of the following is caused by Fungi?
a) 1 only
b) 2 only
c) 1 and 3 only
d) 2 and 3 only
Q.18) Solution (a)
1. Ringworms are caused by Fungi.
2. Ascaris and Filariasis is caused by parasites like round worms and tape worms etc
Q.19) Consider the following statements with respect to Rehabilitation of Bonded labour
scheme 2016
1. The quantum of financial assistance under this scheme is increased from Rs 20 thousand
to one lakh rupees.
2. The scheme aims to address new forms of bondage such as organised begging rings,
forced prostitution and child labour for which females are mercilessly used by the
powerful elements.
3. The scheme is implemented by Ministry of women and child development.
Select the correct option
a) 1 only
b) 2 and 3 only
c) 1 and 2 only
d) All the above
Q.19) Solution (c)
The scheme is implemented by Ministry of Labour and Employment.
Q.20) Consider the following
1. AmritMahal
2. Khillari
3. Sahiwal
4. Hallikar
5. Gir

www.iasbaba.com

Current Affairs- Subject wise compilation of 60 Day Plan- 2016


Which of the following are Indigenous drought breed cattles?
a) 1, 2, and 3 only
b) 3 and 5 only
c) 1, 2 and 4 only
d) None of the above
Q.20) Solution (c)
AmritMahal, Khillari, Hallikar, Kangayam,Bargur,Umblacherry, Pullikulam, are some of
the indigenous drought breeds.
Gir, Sahiwal, Red sindhi, Deoni are some of the indigenous dairy breeds.
Tharparkar,Hariana, Kankrej,Ongole, Krishna valley are some of the dual purpose breeds.
Jersey, Holstein Friesian,Brown swiss, Red dane, Ayrshire, Guernsey.
Murrah, Surthi, jaffarbandi,Bhadawari,Nili ravi, Mehsana, Nagpuri,Toda.
Q.21) Consider the following
1. Udaan is a Special Industry initiative that aims to provide an exposure to the
unemployed graduates to the best of Corporate India.
2. It is implemented by Ministry of skill development and Entrepreneurship.
Select the correct answer with respect to UDAAN scheme
a) 1 only
b) 2 only
c) Both
d) None
Q.21) Solution (c)
It is advised to the candidates that there is a similar scheme UDAAN by MHRD, UDAAN by
MHRD is an initiative of the Central Board of Secondary Education (CBSE) to enable
disadvantaged girl students and other students from SC/ST & minorities to transit from
school to post-school professional education specially in Science and Math.
http://www.skilldevelopment.gov.in/udaan.html
https://www.icsi.edu/docs/webmodules/Highlights_MHRD.pdf
Q.22) Consider the following
1. Lebanon
2. Croatia
3. Cyprus
Which of the following surrounds Mediterranean Sea?
a) 1 only
b) 1, 2 and 3 only

www.iasbaba.com

Current Affairs- Subject wise compilation of 60 Day Plan- 2016


c) 2 and 3 only
d) 2 only
Q.22) Solution (b)
All countries surround Mediterranean Sea
Q.23) Consider the following statements with respect to Green bonds
1. Green bond is a debt instrument issued by an entity for raising funds from investors
2. The funds generated are used to fund both green and non-green projects
3. In India IDBI,EXIM, Yes bank have started issuing green bonds
Select the correct answer using the code given below
a) 1 and 3 only
b) 2 and 3 only
c) All the above
d) None of the above
Q.23) Solution (a)
Fund generated from bonds are used only to fund green projects
http://www.thehindu.com/business/what-are-green-bonds/article7070840.ece.
Q.24) Consider the following statements with respect to Ethanol blending.
1. Ethanol is derived only from Sugarcane.
2. Recently Central Government has scaled up blending targets from 5% to 10% to
promote blending of ethanol with petrol and its use as alternative fuel
3. It reduces the carbon monoxide emissions by up to 30 per cent.
4. Normal cars can be used up 10% ethanol without any up gradation
Select the incorrect statement/s from the above
a) 1 only
b) 1 and 2 only
c) 3 and 4 only
d) 2 ,3 and 4 only
Q.24) Solution (a)
Ethanol is a renewable, domestically produced alcohol fuel made from plant material,
such as corn, sugar cane, or grasses. Using ethanol can reduce oil dependence and
greenhouse gas emissions
Motor vehicles can use ethanol up to 10 % without any up gradation. It also reduces the
carbon monoxide emissions by up to 30 percent
http://pib.nic.in/newsite/PrintRelease.aspx?relid=133773

www.iasbaba.com

10

Current Affairs- Subject wise compilation of 60 Day Plan- 2016

Doubts: @iasbaba please check Q21 again..


Q21 option 1 is correct according to wiki
Clarifications: There is no error/mistake in the question (Option 1 is wrong because it says
only sugarcane) Explanations above explains the same.
Q.25) An issue of updating the National Citizen Register (NCR) in Assam was in news.
Consider the following statements w.r.t NCR
1. The provisions governing NCR updation in Assam are Citizenship Act, 1951 and The
Citizenship (Registration of Citizens and Issue of National Identity cards) Rules, 2003
2. Policy decisions, guidelines, funds for NRC updation and implementation are provided by
and are done through the State Government machinery.
3. Persons whose names appear in NCR, 1955 are eligible for inclusion in NCR
Select the correct code
a) 1 and 2
b) 2 and 3
c) 1, 2 and 3
d) None
Q.25) Solution (d)
The provisions governing NCR updation in Assam are Citizenship Act, 1955 and The
Citizenship (Registration of Citizens and Issue of National Identity cards) Rules, 2003
Policy decisions, guidelines and funds for NRC updation are provided by the Central
Government of India, but its implementation is done through the State Government
machinery under the Registrar General of India
Eligibility criterion for inclusion in NCR
Persons whose names appear in NRC, 1951
Persons whose names appear in any of the Electoral Rolls up to 24 March (midnight),
1971
Descendants of the above-mentioned persons
Persons who came to Assam on or after 1 January 1966 but before 25 March 1971 and
registered themselves in accordance to the rules made by the Central Government with
the Foreigners Registration Regional Officer (FRRO) and those who have not been
declared as illegal migrants or foreigners by the authority
Persons who are original inhabitants of Assam and their children & descendants, who
are citizens of India (provided the citizenship of such persons is ascertained)

www.iasbaba.com

11

Current Affairs- Subject wise compilation of 60 Day Plan- 2016

D voters can apply for inclusion of their names in the updated NRC. However, their
names will. Be finally included only when the appropriate Foreigner Tribunal declares
them as non-foreigners.
All Indian Citizens including their children and descendants who have moved to Assam
post 24 March 1971 would be eligible for inclusion in the updated NRC on adducing
satisfactory proof of residence in any part of the country (outside Assam) as on 24
March 1971.
Persons who can provide any one of the documents issued up to midnight of 24 March
1971 as mentioned in the list of documents admissible for citizenship.

Read these articles


http://indianexpress.com/article/india/india-others/national-register-of-citizens-67-lakhforms-filed-assam-gears-up-to-verify-citizenship-claims/
http://www.thehindu.com/opinion/lead/the-starkness-of-beingnowhere/article7633845.ece
Q.26) The Union Cabinet chaired by the Prime Minister has given its approval to the
National Offshore Wind Energy Policy (OWEP). Consider the following w.r.t Indias Wind
Energy Potential and policies
1. Indias onshore wind energy potential is more than offshore
2. National Institute of Wind Energy (NIWE) has been authorized as the Nodal Agency for
development of offshore wind energy in the country
3. Offshore Wind Energy Policy allows wind energy development, research and
development activities upto seaward distance of territorial waters i.e 12 nautical miles
Select the correct statement/s
a) Only 1
b) 1 and 2
c) 1, 2 and 3
d) 2 and 3
Q.26) Solution (b)
The Union cabinet approved the National Offshore Wind Energy Policy, aiming to harness
wind power along Indias 7,600 km coastline. With this approval, the Ministry of New &
Renewable Energy (MNRE) has been authorized as the Nodal Ministry for use of offshore
areas within the Exclusive Economic Zone (EEZ) of the country and the National Institute of
Wind Energy (NIWE) has been authorized as the Nodal Agency for development of offshore
wind energy in the country and to carry out allocation of offshore wind energy blocks,
coordination and allied functions with related ministries and agencies.

www.iasbaba.com

12

Current Affairs- Subject wise compilation of 60 Day Plan- 2016


The decision paves the way for offshore wind energy development, including projects and
research and development activities up to a seaward distance of 200 nautical miles, which
constitutes the exclusive economic zone (EEZ) of the country.
Of Indias 36,642 MW of installed renewable power capacity, around 23,864 MW is onshore
wind energy alone.
http://www.livemint.com/Politics/hjYjnG8v4SWLIPKr5yo2FJ/Cabinet-approves-nationalwind-energy-policy.html
Q.27) Consider the following statements
1. It is also referred as dancing deer
2. Its habitat is located along the largest fresh water lake in eastern India.
3. The dancing deer is found in its natural habitat over the floating biomass locally called
"phumdi"
Identify the species based on given statements
a) Reindeer
b) Goral
c) Himalayan Tahr
d) Sangai
Q.27) Solution (d)
http://timesofindia.indiatimes.com/home/environment/flora-fauna/Wildlife-Institute-ofIndia-to-relocate-endangered-dancing-deer-of-Manipur/articleshow/52282276.cms
Q.28) Consider the following w.r.t Inactivated Polio Vaccine (IPV)
1. The inactivated polio vaccine produces antigens in the blood to all three types of
poliovirus.
2. IPV is not a 'live' vaccine
3. It is also called the Salk vaccine
Select the incorrect statement/s
a) 2 and 3
b) Only 1
c) Only 3
d) None
Q.29) Solution (b)
Inactivated polio vaccine (IPV) was developed in 1955 by Dr Jonas Salk. Also called the Salk
vaccine, IPV consists of inactivated (killed) poliovirus strains of all three poliovirus types.
IPV is given by intramuscular injection and needs to be administered by a trained health
worker

www.iasbaba.com

13

Current Affairs- Subject wise compilation of 60 Day Plan- 2016


The inactivated polio vaccine produces antibodies in the blood to all three types of
poliovirus. In the event of infection, these antibodies prevent the spread of the virus to the
central nervous system and protect against paralysis.
Advantages
As IPV is not a 'live' vaccine, it carries no risk of vaccine-associated polio paralysis.
IPV triggers an excellent protective immune response in most people.
Disadvantages
IPV induces very low levels of immunity in the intestine. As a result, when a person
immunized with IPV is infected with wild poliovirus, the virus can still multiply inside the
intestines and be shed in the faeces, risking continued circulation.
IPV is over five times more expensive than oral polio vaccine.
Administering the vaccine requires trained health workers and sterile injection equipment
and procedures.
Q.30) Atlas mountain is present in which of the following African countries
a) Algeria and Morocco
b) Mali and Libya
c) Tunisia and Sudan
d) Algeria and Egypt
Q.30) Solution (a)
The Atlas Mountains extend some 2,500km across northwestern Africa, spanning Morocco,
Algeria and Tunisia, separating the Atlantic and Mediterranean coastline from the Sahara
Desert.
Q.31) Consider the following
1. The scheme aims to provide financial assistance to patients of BPL families.
2. Rashtriya Aarogya Nidhi is an initiative of Ministry of health and family welfare.
Select the correct answer with respect to Rashtriya Aarogya Nidhi?
a) 1 only
b) 2 only
c) Both
d) None
Q.31) Solution (c)
Self explanatory
Q.32) Consider the following

www.iasbaba.com

14

Current Affairs- Subject wise compilation of 60 Day Plan- 2016


1. Indus
2. Ravi
3. Jhelum
4. Chenab
5. Sutlej
6. Beas
Arrange the following rivers tributaries of Indus from north to south
a) 123456
b) 124356
c) 214365
d) 134265
Q.32) Solution (d)
Refer Atlas
Q.33) Consider the following
1. Wild life sanctuaries
2. National parks
3. Bio sphere reserves
Arrange the above in decreasing order of their number
a) 231
b) 213
c) 321
d) 123
Q.33) Solution (d)
Wild life sanctuaries - 551
National parks - 96
Bio sphere reserves 14
Q.34) Which of the following statement/s is/are true with respect to CURIE
1. It is known as Consolidation of University Research for Innovation and Excellence
2. It is launched by Dept of S & T
3. The programme is launched for Women only Universities
Select the incorrect statement with respect to CURIE
a) 1 only
b) 2 and 3 only
c) 3 only

www.iasbaba.com

15

Current Affairs- Subject wise compilation of 60 Day Plan- 2016


d) None of the above
Q.34) Solution (d)
CURIE (Consolidation of University Research for Innovation and Excellence) is a unique
programme of Department of Science and Technology for strengthening S&T infrastructure
and also to enhance research facilities in women only universities.
Q.35) Consider the following statements with respect to Indian Ocean Naval Symposium
(IONS)
1. IONS is a voluntary and co-operative initiative between 30 countries of the Indian Ocean
Region for maritime security and cooperation.
2. The first inaugural IONS Seminar was held by the Indian Navy in 2008
3. Pakistan, France reunion, Mauritius are some of the members of the IONS
Select the correct option
a) 1, 2 and 3
b) 2 and 3 only
c) 1 and 3 only
d) 1 and 2 only
Q.35) Solution (a)
All the statements are correct
Clarification: Our answer is wrong. First statement is incorrect. Total members are 35 not 30
so answer should be (b)
http://ions.gov.in/about_ions
Q.36) Consider the following
1. Chanderi silk Madhya Pradesh
2. Dorukha shawls Tamil Nadu
3. Illkal saree Karnataka
Which of the following sarees/shawl is wrongly matched?
a) 1 and 2 only
b) 2 only
c) 1 only
d) 2 and 3 only
Q.36) Solution (b)
Dorukha shawl is from Kashmir

www.iasbaba.com

16

Current Affairs- Subject wise compilation of 60 Day Plan- 2016


Q.37) Consider the following
1. Nepal
2. Bhutan
3. Maldives
4. Malaysia
5. Bangladesh
Which of the above are BIMSTEC (Bay of Bengal initiative for multi sectoral technical and
economic cooperation) countries?
a) 3, 4 and 5 only
b) 1 and 2 only
c) 1, 2 and 5 only
d) 2, 3, 4 and 5 only
Q.37) Solution (c)
Self explanatory
Q.38) Consider the following statement with respect to TAPI pipeline
1. The TAPI pipeline transport Crude oil
2. It is funded by ADB (Asian development bank)
3. The TAPI pipeline terminates at Indian town of Ludhiana
Select the correct statement
a) 1 and 2 only
b) 2 and 3 only
c) 2 only
d) 1, 2 and 3
Q.38) Solution (c)
It transport natural gas
TAPI pipeline ends in Fazilka Punjab
Q.39) Which of the following statements with respect to initiative of AMRIT is correct?
1. It is headed by Ministry of Health and family welfare.
2. AMRIT aims to provide drugs for cardiovascular and cancer diseases at highly affordable
prices.
Select the correct option from the codes given below:
a) 1 only
b) 2 only
c) Both

www.iasbaba.com

17

Current Affairs- Subject wise compilation of 60 Day Plan- 2016


d) None
Q.39) Solution (c)
Union Ministry of Health & Family Welfare has opened countrys first Affordable
Medicines and Reliable Implants for Treatment (AMRIT) outlet at AIIMS in New Delhi to
sell drugs at highly discounted rates.
Key facts
The pilot AMRIT outlet will provide drugs for cancer and cardiovascular diseases at
highly discounted rates. At this outlet the prices of 202 cancer and 186 cardio-vascular

The pilot AMRIT outlet will be managed by the government-owned HLL Life care Ltd

Provide drugs at affordable prices only on the prescription of a doctor for AIIMS patients
and also other hospitals

Q.40) Consider the following statements with respect to Swarna Bharat Yojana
1. The scheme is aimed to decrease gold imports by mobilising gold in the domestic market
2. Gold monetisation, Indian gold coins, Gold bond scheme are some of the ancillary
schemes related to Swarna Bharat Yojana
Select the correct answer using the codes given below
a) 1 only
b) 2 only
c) Both
d) None
Q.40) Solution (c)
Both the statements are true
http://timesofindia.indiatimes.com/india/Now-Swarna-Bharat-campaign-by-centralgovernment/articleshow/49660918.cms
Q.41) Swasth Dharaa Khet Haraa is a slogan of which of the following scheme by
Government of India
a) Pradhan Mantri Krishi sinchana Yojana
b) Pradhan Mantri Fasal Bima Yojana
c) Rashtriya Krishi Vikas Yojana
d) Soil health card scheme
Q.41) Solution (d)
Soil health card scheme has the slogan of Swasth Dharaa Khet Haraa

www.iasbaba.com

18

Current Affairs- Subject wise compilation of 60 Day Plan- 2016


Q.42) Consider the following statements with respect to World Wild Life Fund
1. The fund is created under the aegis of United Nations environment programme
2. Panda is the official logo of world wildlife fund
Select the correct option
a) 1 only
b) 2 only
c) Both
d) None
Q.42) Solution (b)
WWF is not created under UNEP,
World Wildlife Fund (WWF) and its Aims
When Sir Julian Huxley, the renowned British biologist, went to Africa for a research trip.
What he did sea there, shook him completely. He was dismayed by the unclenched
destruction of nature and the hunting of endangered species.
When Sir Huxley returned in 1960, he shared his concern, that many species would become
extinct it the condition persists in Africa, with fellow scientist and other prominent
personalities. Finally they agreed to begin an international organization with the scientific,
technical and financial resources to conduct and fund conservation efforts across the globe.
In 1961, World Wildlife Fund (WWF) was created. Sir Huxley and Sir Peter Scott, the famous
ornithologist who created the Panda logo, were among the founder members of World
Wildlife Fund.
The aims of the World Wildlife Fund are to halt the destruction of the earths natural
environment and to prepare a future in which all human beings can live in harmony with
nature. World Wildlife Fund does this by protecting all forms of life on earth, and making
sure that the earths natural resources like water and forests are not consumed faster than
they can be renewed. World Wildlife Fund encourages the people to curtail the amount of
waste they normally produce, and to cut down on pollution.
http://www.thehindu.com/sci-tech/energy-and-environment/global-tiger-population-upby-22-per-cent/article8461131.ece
Q.43) Shyam Benegal Committee is related to which of the following subjects
a) Tribal alienation
b) Public distribution system
c) Film certification
d) Regulation of MSME
Q.43) Solution (c)

www.iasbaba.com

19

Current Affairs- Subject wise compilation of 60 Day Plan- 2016


Shyam Benegal committee on film certification
Shyam Benegal committee was set up by the Union Government in January 2016 to lay
down norms for film certification to give give sufficient and adequate space for artistic and
creative expression. The committee was asked to take note of best practices in various parts
of the world.
The committee was also asked to lay down procedures and guidelines for the benefit of the
Central Board of Film Certification (CBFC) to follow and examine staffing patterns with a
view to recommending a framework that would provide efficient and transparent user
friendly services.
Recommendations
1. CBFC as a Certification Body
The CBFC should only be a film certification body. Its scope should be restricted to
categorizing the suitability of the film to audience groups on the basis of age and maturity.
2. Refusing Certification
The CBFC should refuse certification only when a film contains anything that contravenes
the provisions of Section 5B (1) of the Cinematograph Act, 1952 or when its content crosses
the ceiling laid down in the highest category of certification.
3. Categorisation of Films
The committee recommended that the categorisation of films should be more specific. The
UA Category can be broken up into further sub-categories UA12+ and UA15+.
The A category should also be sub-divided into A and AC (Adult with Caution) categories.
http://www.thehindu.com/news/national/shyam-benegal-to-head-panel-to-revampcensor-board/article8054774.ece
Q.44) Chaitra Vaishaka months in traditional Indian calendar corresponds to which of the
following months in English calendar?
a) March-April
b) May-June
c) Jan-Feb
d) April-May
Q.44) Solution (a)
It refers to March and April
Additional Value Addition
There are 12 months in Hindu lunar Calendar:
Chaitra (30 / 31* Days) Begins March 22 / 21*
Vaisakha (31 Days) Begins April 21

www.iasbaba.com

20

Current Affairs- Subject wise compilation of 60 Day Plan- 2016

Jyaistha (31 Days) Begins May 22


Asadha (31 Days) Begins June 22
Shravana (31 Days) Begins July 23
Bhadra (31 Days) Begins August 23
Asvina (30 Days) Begins September 23
Kartika (30 Days) Begins October 23
Agrahayana (30 Days) Begins November 22
Pausa (30 Days) Begins December 22
Magha (30 Days) Begins January 21
Phalguna (30 Days) Begins February 20 Leap years

FESTIVALS/RITUALS
1. Chaitra
The month of Chaitra is also associated with the coming of Spring, since Holi, the spring
festival of colour, is celebrated on the eve of Chaitra (namely, the last day of Phalgun
month). Exactly 6 days after which the festival of Chaiti Chhath is observed.
In lunar religious calendars, Chaitra begins with the new moon in March/April and is the first
month of the year. The first of Chaitra is celebrated as New Year's Day, known as Gudi
Padwa in Maharashtra, Chaitrai Vishu and Ugadi in Karnataka and Andhra Pradesh. Other
important festivals in the month are; Ram Navami, the birth anniversary of Lord Ram
celebrated on the 9th day of Chaitra, and Hanuman Jayanti that falls on the last day
(purnima) of Chaitra.
2. Vaikha The harvest festival of (Baisakhi) is celebrated in this month. Vaisakha Purnima
is celebrated as Buddha Purnima or the birthday of Gautama Buddha amongst southern
Buddhists or the Theravada school. Purnima refers to the Full Moon. Known in Sinhalese as
Vesak, it is observed in the full moon of May
3. Jyaiha
Vat Pournima is a celebration observed in Maharashtra and Karnataka, India. It is celebrated
on the full moon day (the 15th) of the month of Jyeshtha on the Hindu Calendar, which falls
in June on the Gregorian Calendar. Women pray for their husbands by tying threads around
a banyan tree on this day. It honors Savitri, the legendary wife of Satyavan who escaped
death for her husband's life.
Snana Yatra is a bathing festival celebrated on the Purnima the Hindu month of Jyeshtha. It
is an important festival of the Jagannath Cult. The deities Jagannath, Balabhadra, Subhadra,
Sudarshan, and Madanmohan are brought out from the Jagannath Temple (Puri) and taken
in a procession to the Snana Bedi. They are ceremonially bathed and decorated for a public
audience. Sitalsasthi Carnival is being conducted in this month on the day of Jyeshtha
Shuddha Shashthi in Odisha for many centuries

www.iasbaba.com

21

Current Affairs- Subject wise compilation of 60 Day Plan- 2016


4. Asadha
Guru Purnima, a festival dedicated to the Guru, is celebrated on the Purnima (Full Moon)
day of the month. Prior to it Shayani Ekadashi, is observed on the eleventh lunar day
(Ekadashi) of the bright fortnight.
5. Sravana
Shravana(jupaka) is considered to be a holy month in the Hindu calendar due to the many
festivals that are celebrated during this time. Krishna Janmashtami, marking the birth of
Krishna, falls on the 8th day after the full moon. Raksha Bandhan, the festival of brothers
and sisters, is celebrated on Shraavana Poornima (Full Moon). This day in Maharashtra is
also celebrated as Narali Poornima (Naral in Marathi language means coconut). In the
coastal regions of Maharashtra i.e. Konkan, a coconut is offered to the sea for calming it
down after the monsoon season. Fishermen now start fishing in the sea after this ceremony.
Nag Panchami is also celebrated in many parts of India on the fifth day after Amavasya of
Shraavana month. The snake god Nga is worshiped. The last day of the Shraavana is
celebrated as Pola, where the bull is worshiped by farmers from Maharashtra.
In TamilNadu (& also in Kerala) Aadi Amavasaya is celebrated with great importance in all
temples. It is an equivalent to Mahalaya Amavasaya of north India.In KarnatakaBasava
Panchami is celebrated on 5th day after amavasya.
Shravani Mela is a major festival time at Deoghar in Jharkhand with thousands of saffronclad pilgrims bringing holy water around 100 km on foot from the Ganges at
Sultanganj.Shravan is also the time of the annual Kanwar Yatra, the annual pilgrimage of
devotees of Shiva, known as Kanwaria make to Hindu pilgrimage places of Haridwar,
Gaumukh and Gangotri in Uttarakhand to fetch holy waters of Ganges River
6. Bhdrapada or Bhdra also Prohapada
Anant Chaturdashi is a Jain religious observance is performed on the fourteenth day
(Chaturdashi) of the bright fortnight (Shukla paksha) of Bhadrapad month.
Madhu Purnima (Bengali for 'honey full-moon') is a Buddhist festival celebrated in India and
Bangladesh, especially in the region of Chittagong. It occurs on the day of the full moon in
the month of Bhadro (August/September).
7. Asvina
Several major religious holidays take place in Ashvin, including Durga Puja (6-10 Ashvin),
Dasehra (10 Ashvin) and Divali (29 Ashvin), Kojagiri festivals and Kali Puja (new moon of
Ashvin),
8. Kartika
The festival of Kartik Poornima (15th day Full Moon) falls in this month, celebrated asDev
Deepavali in Varanasi. This coincides with the nirvana of the Jain Tirthankara - Mahavira and

www.iasbaba.com

22

Current Affairs- Subject wise compilation of 60 Day Plan- 2016


the birth of the Sikh Guru Nanak Guru Nanak Jayanti. And also, the well known festival, for
the god of Sabarimalai, Ayyappan's garland festival.
9. Agrahyaa
Vaikuha Ekda, the Ekda (i.e. 11th lunar day) of this Mrgaa month, is celebrated
also as Mokad Ekda. The 10th Canto, 22nd Chapter of Bhgavata Pura, mentions
young marriageable daughters (gopis) of the cowherd men of Gokula, worshiping Goddess
Ktyyan and taking a vrata or vow, during the entire month of Mrgaa, the first month
of the winter season (iira), to get r Ka as their husband.
Klabhairava Aam (or Klabhairava Jayanti) falls on Ka Paka Aam of this month of
Mrgaa. On this day it is said that Lord iva appeared on earth in the fierce manifestation
(avatra) as r Klabhairava. This day is commemorated with special prayers and rituals.
10. Pausa
The harvest festival of Pongal/Makar Sankranti is celebrated on this month.
11. Magha
Vasant Panchami, sometimes referred to as Saraswati Puja, Shree Panchami, or the Festival
of Kites is a Sikh and Hindu festival held on the fifth day of Magha (in early February)
marking the start of spring and the Holi season. On this day Hindus worship Saraswati, the
goddess of knowledge, music, art and culture.
Ratha Saptami or Rathasapthami is a Hindu festival that falls on the seventh day (Saptami) in
the bright half (Shukla Paksha) of the Hindu month Maagha. It marks the seventh day
following the Suns northerly movement (Uttarayana) of vernal equinox starting from
Capricorn (Makara).
12. Phalguna
Most parts of North India see early celebration of the famous Hindu festival Holi in this
month. Holi is celebrated at the end of the winter season on the last full moon day of the
lunar month Phalguna (Phalguna Purnima), which usually falls in the later part of February
or March. The Hindu festival of Shigmo is also celebrated in Goa and Konkan in the month of
Phalguna. Celebrations can stretch over a month.
Source: Hindu calendar
Q.45) Consider the following regarding The Convention on the Rights of the Child
1. It is a legally binding treaty under the ambit of United Nations.
2. According to the convention a child is any person who has not reached the age of
sixteen unless a different age of maturity is specified in any country's law.
3. It talks about civil, political, economic, social, health and cultural rights of children.
Select the correct code

www.iasbaba.com

23

Current Affairs- Subject wise compilation of 60 Day Plan- 2016


a) 1 and 2
b) 2 and 3
c) 1 and 3
d) 1, 2 and 3
Q.45) Solution (c)
http://www.thehindu.com/news/national/world-day-against-child-labour-has-therebeen-a-change/article8720756.ece
http://www.childlineindia.org.in/united-nations-convention-on-the-rights-of-the-child.htm
https://en.wikipedia.org/wiki/Convention_on_the_Rights_of_the_Child
World Day Against Child Labour: 12 June
Q.46) Consider the following countries
1. Bulgaria
2. Albania
3. Romania
Which of the above listed country is not bordering Black Sea?
a) 1 and 2 only
b) 2 only
c) 1 and 3 only
d) 2 and 3 only
Q.46) Solution (b)
Albania is not bordering Black Sea
Q.47) Consider the following statements with respect to ParamParagt Krishi Vikas Yojana
1. The scheme follows cluster based approach of 50 farmers to promote organic farming
2. Each farmer will be provided 20000 rupees per acre in three years from seed to
harvesting crops and to transport them to market
Select the correct answer using the codes given below
a) 1 only
b) 2 only
c) Both
d) None
Q.47) Solution (c)
http://pib.nic.in/newsite/PrintRelease.aspx?relid=118622

www.iasbaba.com

24

Current Affairs- Subject wise compilation of 60 Day Plan- 2016


Q.48) Consider the following statements with respect to Rashtriya Gokul Mission
1. The scheme focuses on conservation and development of indigenous and exotic breeds.
2. Gokul Grams will be established to develop indigenous breeds under Rashtriya Gokul
mission
Select the correct answer using the codes given below
a) 1 only
b) 2 only
c) Both
d) None
Q.48) Solution (b)
The scheme focuses on indigenous breeds only
Q.49) Consider the following statements with respect to Jan Aaushadi scheme
1. The schemes aims to make generic medicines available to BPL and APL families only
2. The scheme is spear headed by Ministry of Health and Family Welfare
Select the correct answer
a) 1 only
b) 2 only
c) Both
d) None
Q.49) Solution (d)
1. The schemes aims to make generic medicines available to ALL citizens of India
irrespective of here economic status
2. The scheme is sphere headed by ministry of chemicals and fertilisers
Q.50) Consider the following statements with respect to recently announced Price
Stabilization Fund
1. It functions under Consumer Affairs Ministry
2. The Price Stabilization Fund aims to stabilize the prices of tea, coffee, rubber and tobacco
to contain the distress of the farmers
Select the correct answer
a) 1 only
b) 2 only
c) Both
d) None

www.iasbaba.com

25

Current Affairs- Subject wise compilation of 60 Day Plan- 2016


Q.50) Solution (a)
Initially the fund was created under ministry of agriculture but recently the government will
shift the Rs 500-crore Price stabilization Fund (PSF) to consumer affairs ministry from
agriculture ministry for effective control of price rise in essential commodities and provide
relief to the consumers.
Last year, the PSF was created with a corpus of Rs 500 crore under the agriculture ministry.
The fund was to be used to support market interventions for managing prices of perishable
agri-horticultural commodities by procuring directly from farmers and later supplying at
reasonable rates to consumers.
Q.51) Consider the following
1. Muga
2. Eri
3. Mulberry
4. Tasar
Arrange the above varieties of silk in increasing order of their production
a) 3241
b) 3412
c) 2413
d) 2143
*Note: Answer and explanation is corrected, please kindly note and update the same.
Q.51) Solution (a)

India is the second largest producer of silk in the world. Among the four varieties of silk
produced in the country, Mulberry accounts for 74 per cent (19476 MT), Tasar 10 per
cent (2619 MT), Eri 16.0 per cent (4237 MT) and Muga 0.5 per cent (148 MT) of the total
raw silk production of 26,480 MT in 2013-14.
Sericulture provides gainful occupation to around 63 Lakh persons in rural and
semiurban areas in India.
Muga is unique in having a golden sheen and is a prized possession of India. Muga is
largely restricted to Assam and other north-eastern states and recently spread to West
Bengal.
About 80% of the silk produced in the country is of mulberry silk, majority of which is
produced in the three southern States of Karnataka, Andhra Pradesh and Tamil Nadu
followed by West Bengal and Jammu & Kashmir. Mulberry sericulture is practiced since
time immemorial in the North Eastern Region (NER).
The tropical Tasar silk is produced largely in the central India covering the tribal areas of
Jharkhand , Chhattisgarh, Bihar, Madhya Pradesh, Maharashtra, Uttar Pradesh, Andhra
Pradesh, Orissa, etc. Oak Tasar is produced in Manipur, Mizoram, Nagaland,
Uttarakhand, Jammu & Kashmir, etc.
Eri is grown in Assam and the adjacent north-eastern states, Bihar, West Bengal and
Odisha.

www.iasbaba.com

26

Current Affairs- Subject wise compilation of 60 Day Plan- 2016


Q.52) Shalimer Garden was created by which of the following Mughal ruler
a) Akbar
b) Humayun
c) Shah Jahan
d) Jehangir
Q.52) Solution (d)
It is here that Emperor Jahangir built his celebrated Shalimar Bagh, his dream project to
please his queen. He enlarged the ancient garden in 1619 into a royal garden and called it
'Farah Baksh' ('the delightful'). He built it for his wife Nur Jahan ('light of the world').
http://www.thehindu.com/features/metroplus/brickbybrick/article8718597.ece
Q.53) The article which deals with abolition and creation of legislative council in states is
a) Article 171
b) Article 169
c) Article 175
d) Article 182
Q.53) Solution (b)
Article 169 deals with abolition and creation of legislative council in states.
http://www.thehindu.com/news/national/other-states/up-legislative-council-pollstoday/article871170
Q.54) Recently government of India announced setting up of Asias first rice technology
park.
The proposed park is based on
a) Amaravati Andhra Pradesh
b) Gangavathi Karnataka
c) Nellore Andhra Pradesh
d) Ludhiana Punjab
Q.54) Solution (b)
It is in Gangavathi Karnataka
http://www.thehindu.com/news/national/karnataka/gangavati-to-get-asias-first-ricetechnology-park/article85637
Q.55) The restoration of Noyyal River was in news recently. Noyyal River is found in which
of the following state
a) Kerala
b) Assam
c) West Bengal

www.iasbaba.com

27

Current Affairs- Subject wise compilation of 60 Day Plan- 2016


d) Tamil Nadu
Q.55) Solution (d)
http://www.thehindu.com/news/national/tamil-nadu/dyers-look-for-rs-200croregrant/article8750235.ec.
Q.56) Consider the following statements and select the incorrect statement/s from the
codes given below:
1) Foreign-origin companies can give donations to political parties
2) Donations made by such [foreign shareholding] companies to entities including parties
will not attract provisions of the Foreign Contribution Regulation Act (FCRA)
3) The Representation of the People Act bars parties from receiving foreign funds
Choose the appropriate code:
a) 1 only
b) 1 and 2 only
c) 3 only
d) None
Q.56) Solution (d)
Explanation:
Government recently brought amendment to Foreign Contribution Regulation Act
(FCRA), 2010
The amendment not only help foreign-origin companies fund NGOs but has also cleared
the way for them to give donations to political parties
The amendment would ensure that donations made by such [foreign shareholding]
companies to entities including parties will not attract provisions of the Act.
The statement assumes significance as such funding from foreign donors will bypass
government scrutiny. The Representation of the People Act bars parties from receiving
foreign funds.
Q.57) Choose the incorrect pair from below:
Missile
Associated Country
a) Dong-Feng 26 (DF 26)
North Korea
b) Guam killer
China
c) Poseidon
US
d) Babur
Pakistan
Q.58) Solution (a)
Explanation:
Dong-Feng 26 (DF 26) is actual name of Guam killer missile

www.iasbaba.com

28

Current Affairs- Subject wise compilation of 60 Day Plan- 2016

This missile belongs to China, which has a range of 5,500km


The DF-26s range enables it to reach Guam and threaten American military installations
on the island, particularly Anderson Air Force Base, earning it the nickname of Guam
Killer

Q.59) Consider the following statements related to Indian Rhinos Vision 2020:
1) The goal of IRV 2020 is to increase the total rhino population in Assam from 2,000 to
3,000 by the year 2020
2) Pabitora National Park has the largest number of one horned rhinos in the world
3) Kaziranga National Park has the highest density of rhinos in the world.
4) Translocations of rhinos from source populations in Kaziranga and Pabitora to target
areas such as Manas.
Which of the statements given above are correct?
a) 1 and 2
b) 1 and 4
c) 1, 2 and 3
d) 2, 3 and 4
Q.59) Solution (b)
Explanation:
Kaziranga National Park has the largest number of one horned rhinos in the world
whereas Pabitora National Park has the highest density of rhinos in the world.
Q.60) Consider the following with regard to Intellectual Property Rights:
1) Every year World Intellectual Property Day (WIPD) is being observed on 26 April
2) It highlights the importance of intellectual property rights (patents, trademarks,
industrial designs, copyright) in encouraging innovation and creativity.
3) 2016 Theme: Digital Creativity: Culture Reimagined.
Choose the correct code
a) 1 only
b) 1 and 3 only
c) 1 and 2 only
d) All of the above
Q.60) Solution (d)
Explanation:
Every year World Intellectual Property Day (WIPD) is being observed on 26 April to
highlight importance of intellectual property rights (patents, trademarks, industrial
designs, copyright) in encouraging innovation and creativity.

www.iasbaba.com

29

Current Affairs- Subject wise compilation of 60 Day Plan- 2016

2016 Theme: Digital Creativity: Culture Reimagined. The theme seeks to explore the
future of culture in the digital age in order to answer questions how we create it, how
we access it, how we finance it related to it.
The idea of declaring a particular day as a World Intellectual Property Day was mooted
in October 1999 at the General Assembly of the WIPO

Q.61) Consider the following with regard to Trade Facilitation Agreement:


1) India yet to formally ratify the WTOs (World Trade Organisation) Trade Facilitation
Agreement, which aims at easing customs procedures to boost commerce.
2) The TFA is the WTOs first-ever multilateral accord that aims to simplify customs
regulations for the cross-border movement of goods.
3) It was outcome of WTOs 9th Bali (Indonesia) ministerial package of 2013.
Choose the correct code
a) 1 only
b) 1 and 3 only
c) 2 and 3 only
d) All of the above
Q.61) Solution (c)
Explanation:
India has formally ratified the WTOs (World Trade Organisation) Trade Facilitation
Agreement, which aims at easing customs procedures to boost commerce.
The TFA is the WTOs first-ever multilateral accord that aims to simplify customs
regulations for the cross-border movement of goods. It was outcome of WTOs 9th Bali
(Indonesia) ministerial package of 2013. The agreement includes provisions for
1) Lowering import tariffs and agricultural subsidies: It will make it easier for developing
countries to trade with the developed world in global markets.
2) Abolish hard import quotas: Developed countries would abolish hard import quotas on
agricultural products from the developing world and instead would only be allowed to
charge tariffs on amount of agricultural imports exceeding specific limits.
3) Reduction in red tape at international borders: It aims to reduce red-tapism to facilitate
trade by reforming customs bureaucracies and formalities.
Q.62) Consider the following statements:
1. After Ice Age, Mars pole has become warmer than lower latitudes.
2. After Ice Age, the polar region and high latitudes had become cooler than average on
Earth
Select the incorrect option/s from the following:
a) Only 1

www.iasbaba.com

30

Current Affairs- Subject wise compilation of 60 Day Plan- 2016


b) Only 2
c) Both 1 and 2
d) None of the above
Q.62) Solution (d)
https://www.newscientist.com/article/dn18949-the-history-of-ice-on-earth/
http://www.thehindu.com/sci-tech/science/mars-emerging-from-ice-age-saysstudy/article8656584.ece
Q.63) Identify the committee set up to simplify tax administration?
a) Urjit Patel Committee
b) R.V.Easwar Committee
c) Parthasarthi Shome Committee
d) Mohanty Committee
Q.63) Solution (c)
Parthasarthi Shome Committee
Income Tax Return forms should also include wealth tax details
Retrospective amendments to tax laws should be avoided as a principle
The post of Revenue Secretary should be abolished
CBDT and the CBEC should be merged
Use of Permanent Account Number (PAN) should be widened
Ashok Mehta Committee: Committee on Panchayati Raj institutions
R.V.Easwar Committee: To simplify the provisions of the Income Tax Act, 1961
Deepak Mohanty Committee: On financial inclusion
Q.64) Which port became the first in the country to implement logistics data tagging of
containers?
a) Jawaharlal Nehru Port
b) Enayam Port
c) Port of Kolkata
d) Haldia Port
Q.64) Solution (a)
Jawaharlal Nehru Port, Mumbai Will help importers/exporters track their goods in transit through logistics data bank
service.
An RFID (Radio Frequency Identification Tag) tag would be attached to each container
which would be tracked through RIFD readers installed at different locations.

www.iasbaba.com

31

Current Affairs- Subject wise compilation of 60 Day Plan- 2016

It would provide the Visibility and Transparency of the EXIM Container Movement by
covering the entire movement through rail or road till the ICDs (Inland Container Depot)
and CFSs (Container Freight Station).
Service will integrate the information available with various agencies across the supply
chain to provide detailed real time information within a single window
Help in reducing the overall lead time of the container movement across the western
corridor and lower the transaction costs incurred by shippers and consignees

Enayam Port- To be developed near Colachel in Tamil Nadu


Q.65) What do you mean by Fly-by-night Money?
a) Investment in technology
b) Investment in stock markets in order to make capital gains
c) Investment in technology and the production process
d) Investment in provision of services
Q.65) Solution (b)
Investment in stock markets in order to make capital gains
Also known as Hot Money has the tendency to move from one country to other for
quick profit leading to a sudden volatility in the market post leaving.
Q.66) The World Investment Report, 2016 has been released by the
a) World Bank
b) International Monetary Fund
c) AIIB
d) UNCTAD
Q.66) Solution (d)
India
10th highest recipient of FDI in 2015
Attractiveness as a business destination: 6th
Q.67) Interpol has issued a Red Corner Notice (RCN) against Shahid Latif, Pakistan-based
handler of Jaish-e-Mohammed terrorists who had attacked Pathankot Air Force base in
January. Consider the following statements with respect to INTERPOL
1. It is the second largest political organization after the United Nations in terms of
international representation.
2. INTERPOL is not an organ of UN
3. INTERPOL headquarters is located in Lyon, France
Select the correct option
www.iasbaba.com

32

Current Affairs- Subject wise compilation of 60 Day Plan- 2016


a) 1 and 2 only
b) 2 and 3 only
c) 3 only
d) 1, 2 and 3
Q.67) Solution (d)
http://www.interpol.int/en
Q.68) The cyclonic storm ROANU has affected which of the following three states of India
a) Odisha, Andhra Pradesh, West Bengal
b) Odisha, Kerala, Tamil Nadu
c) West Bengal, Tamil Nadu, Karnataka
d) Andhra Pradesh, Tamil Nadu, Kerala
Q.68) Solution (a)
http://www.newsonair.com/news.asp?cat=National&id=NN19610&bigger=bigger
Q.69) Consider the following
1. Ammonia
2. Insecticides
3. Sulphur dioxide
Which of the following does not belong to the category of inorganic compound which
cause water pollution?
a) 1 and 3 only
b) 2 only
c) 2 and 3 only
d) None of the above
Q.69) Solution (b)
http://www.lenntech.com/water-pollutants-faq.htm
http://www.thehindu.com/todays-paper/water-in-520-villages-in-ramanagaram-districtfound-unfit-to-drink/article8622936.ece
Q.70) Consider the following
1. Fibrinogen
2. Globulins
3. Albumins
Which of the above are the components of Plasma?
a) 1 only

www.iasbaba.com

33

Current Affairs- Subject wise compilation of 60 Day Plan- 2016


b) 2 and 3 only
c) 3 only
d) All
Q.70) Solution (d)
http://www.ncert.nic.in/ncerts/l/kebo118.pdf
Q.71) The indigenous people Pygmies are found in which of the following deserts
a) Kalahari Desert
b) Arabian Desert
c) Gobi Desert
d) Atacama Desert
Q.71) Solution (a)
Q.72) Consider the following countries
1. Congo
2. Gabon
3. Mali
4. Eritrea
Which of the African countries are located above equator?
a) 1 and 2 only
b) 2 and 3 only
c) 3 and 4 only
d) 1, 2 and 3 only
Q.72) Solution (c)
Refer Atlas
Q.73) Mahila E Haat is an online platform for women for
a) Showcasing products made/manufactured/sold by women entrepreneurs.
b) Disburse loans for women belonging to BPL families
c) Provide helping hand in the form of financial assistance during Pregnancy period
d) E learning portal for rural women under Bharat Nirman programme
Q.73) Solution (a)
http://mahilaehaat-rmk.gov.in/en/innerpage/about-e-haat.php
Q.74) Consider the following statements with respect to MUDRA bank
1. It is a subsidiary of SIDBI

www.iasbaba.com

34

Current Affairs- Subject wise compilation of 60 Day Plan- 2016


2. It is a Non banking financial company
3. Loans under MUDRA are issued in three categories - Shishu, Kishore and Tarun
4. Only PSB(public sector banks) are entitled to disburse MUDRA loans
Select the correct option
a) 1 and 2 only
b) 2 and 3 only
c) 1, 2 and 3 only
d) 2, 3 and 4 only
Q.74) Solution (c)
http://www.mudra.org.in/FAQ
Q.75) Consider the following statements with respect to Food Safety & Standards
Authority of India (FSSAI)
1. It is a non statutory body
2. It works under ministry of food processing
Select the correct option
a) 1 only
b) 2 only
c) Both
d) None
Q.75) Solution (d)
It is statutory body and works under ministry of health and family welfare
Q.76) Consider the following statements with respect to National Air Quality Index
1. It is an initiative under Swacch Bharat
2. The proposed air quality index includes 8 pollutants
3. CO2, CO, NH3 are some of the air pollutants included in the list
Select the correct option
a) 1 and 3 only
b) 1 and 2 only
c) 2 and 3 only
d) All
Q.76) Solution (b)
Carbon dioxide is not included in the list of pollutants.
http://pib.nic.in/newsite/PrintRelease.aspx?relid=110654

www.iasbaba.com

35

Current Affairs- Subject wise compilation of 60 Day Plan- 2016

Q.77) Sahayog Kaijin is a joint coast guard exercise between India and
a) Oman
b) Japan
c) Bangladesh
d) Sri Lanka
Q.77) Solution (b)
http://www.thehindu.com/news/national/india-japan-conduct-joint-exercise-sahyogkaijinoff-chennai-coast/article8
Q.78) Consider the following
1. Mitra shakti India and Sri Lanka
2. Indradhanush India and UK
3. Varuna India and Russia
Which of the following military exercises are wrongly matched?
a) 1 and 2 only
b) 2 and 3 only
c) 3 only
d) 1 and 3 only
Q.78) Solution (c)
Varuna is a naval exercise between India and France
Q.79) Consider the following
1. Tsomoriri
2. Deepor Beel
3. Koleru Lake
Which of the above given lakes/Sanctuary is/are a designated Ramsar sites?
a) 1 and 2 only
b) 2 and 3 only
c) 1 and 3 only
d) All
Q.79) Solution (d)
https://en.wikipedia.org/wiki/List_of_Ramsar_sites_in_India
Q.80) Razmnama is a Persian translation of which of the following Indian works
a) Ramayana

www.iasbaba.com

36

Current Affairs- Subject wise compilation of 60 Day Plan- 2016


b) Mahabharata
c) Bhagavathgeetha
d) Atarvanaveda
Q.80) solution (b)
PM modi mentioned this in a speech at a cultural exchange function organised during his
visit to Iran
Q.81) Leelavathi of Bhaskara is an ancient treatise related to which of the following
subject
a) Mathematics
b) Medicine
c) Surgery
d) Poetry
Q.81) solution (a)
PM modi mentioned this in a speech at a cultural exchange function organised during his
visit to Iran
Q.82) Consider the following
1. Decrease in temperature.
2. Exposure to low tides
3. Run off and pollution
Which of the above is/are major cause of coral bleaching?
a) 1 and 2 only
b) 2 and 3 only
c) 1 and 3 only
d) All
Q.82) solution (d)
Q.83) Consider the following
1. Olive Ridley turtles
2. Hawks bill turtles
3. Leather back turtle
Which of the above group of turtles come for mass nesting to India?
a) 1 and 2 only
b) 2 and 3 only
c) 1 and 3 only

www.iasbaba.com

37

Current Affairs- Subject wise compilation of 60 Day Plan- 2016


d) All
Q.83) solution (d)
Hawksbill turtle come for mass nesting in Lakshadweep and Andaman Islands, leather back
turtles in Andaman and Olive Ridley turtles in Orissa
Q.84) Pradhan Mantri Fasal bhima yojana aims to unburden the farmers who are exposed
to the vulnerabilities of natural calamities. Consider the following statements
1. The scheme aims to provide 2% annual insurance premium for Rabi crops
2. For horticulture crops the rate of premium is 10 %
Select the correct answer
a) 1 only
b) 2 only
c) Both
d) None
Q.84) solution (d)
For Rabi crop 1.5 percent
For horticulture crop 5 percent
Q.85) Consider the following rivers
1. Mandovi
2. Zuari
3. Barak
Which of the above river does not flow in the state of Goa?
a) 2 and 3 only
b) 1 and 3 only
c) 3 only
d) 1 and 2 only
Q.85) solution (c)
Mandovi and zuari flow in the state of Goa
Q.86) Operation Red lotus is a group of Maratha Mughal alliance led by which of the
following freedom fighter
a) Balaji baji rao
b) Shah Alam
c) Tantya tope
d) Balaji Vishwanath

www.iasbaba.com

38

Current Affairs- Subject wise compilation of 60 Day Plan- 2016

Q.86) solution (c)


Operation Red Lotus, 1857 A Maratha-Mughal alliance against the British led by Tatya
Tope
Q.87) Consider the following
1. World bank New York
2. WTO Washington
3. UNESCO Paris
Which of the above international organisation and there headquarters is wrongly matched?
a) 1 and 3 only
b) 1 and 2 only
c) 2 and 3 only
d) None of the above
Q.87) solution (b)
World bank headquarters is in Washington and WTO headquarters is in Geneva Switzerland
Q.88) Amir Khusro, a scholar poet and musicologist of rare talents was present in the
court of which medieval Indian ruler
a) Allauddin khilji
b) Akbar
c) Iltimush
d) Balban
Q.88) solution (a)
Amir Khusro, a scholar poet and musicologist of rare talent in the court of Allauddin Khilji
(13th Century, A.D.) is credited with the introduction of entirely new forms and styles in
Hindustani music which are still in practice today.
Q.89) Consider the following statement with respect to CERN
1. CERN is the worlds largest body of experts in nuclear and particle physics, where
scientists and engineers are probing the fundamental structure of universe by using the
most sophisticated scientific instruments and advanced systems.
2. CERN is the birthplace of www (World Wide Web) which is regularly used to browse
the internet.
Select the correct option with respect to CERN
a) 1 only
b) 2 only

www.iasbaba.com

39

Current Affairs- Subject wise compilation of 60 Day Plan- 2016


c) Both
d) None
Q.89) solution (c)
CERN is the worlds largest body of experts in nuclear and particle physics, where
scientists and engineers are probing the fundamental structure of universe by using the
most sophisticated scientific instruments and advanced systems. CERN was established
in 1954 and is based in Geneva.
Significantly, CERN is the birthplace of www (World Wide Web) which is regularly used
to browse the internet. Similarly, the touch screen widely used in smart phones was also
first developed by CERN.
Q.90) Consider the following w.r.t Asian and African Elephants?
1. Both the males and females have tusks in Asian and African elephants
2. African elephants have keener sense of smell as compared to Asian elephants
Select the correct code
a) Only 1
b) Only 2
c) Both
d) None
Q.90) Solution (d)
Why is there an increase in tuskless males among Asian elephants? Why do only males have
tusks in Asia, while in Africa, both the sexes have tusks? Why does the elephant in the subcontinent have a keener sense of smell sharper than that of a dog than its African
counterparts?
The answers to these questions are there in the genome of the Asian elephant, which has
been sequenced perhaps for the first time in India
Read more- http://www.thehindu.com/news/national/karnataka/genome-of-the-asianelephant-sequenced-for-the-first-time-in-india/article8006026.ece
Q.91) Consider the following statements
1. It is not a member of United Nations but a member of IMF and World Bank
2. It declared its independence from Serbia
3. It is a landlocked country
4. It is one of the Muslim majority territories on European mainland
Identify the country?
a) Albania

www.iasbaba.com

40

Current Affairs- Subject wise compilation of 60 Day Plan- 2016


b) Kosovo
c) Bosnia
d) Slovenia
Q.91) Solution (b)
Statement 1 As of 23 June 2015, 108 UN member countries recognise the Republic of
Kosovo, while 85 member countries do not recognize them.] It is not itself a member of the
UN, however it is a member of the International Monetary Fund (IMF), World Bank.
Statement 2 It declared its independence from Serbia in 2008
Statement 3 Kosovo is landlocked in the central Balkan Peninsula. It is bordered by the
Republic of Macedonia and Albania to the south, Montenegro to the west, and the
uncontested territory of Serbia to the north and east
Statement 4 https://en.wikipedia.org/wiki/Islam_in_Europe
Refer this
http://www.thehindu.com/todays-paper/tp-international/unesco-rejects-kosovos-bid-formembership/article7863025.ece
Religion in Kosovo Religion
Islam
Christians
Catholic
Orthodox
Other
None
Not stated

Population
1,663,202
64,275 38,438 25,837

%
95.60
3.69 2.20 1.48

1,188
1,842
10,023

0.06
0.10
0.55%

Q.92) Consider the following statements about Jal-Tarang:


1. It is an Indian percussion instrument.
2. It consists of ceramic or metal bowls tuned by putting different amount of water in them.
3. It is played by hitting the bowls with sticks.
Which of the above statements are correct?
a) 2 and 3
b) All of the above
c) 1 and 3
d) None of the above
Q.92) Solution (b)
Jal Tarang is an Indian melodic percussion instrument. It consists of a set of ceramic or metal
bowls tuned with water. The bowls are played by striking the edge with beaters, one in each
hand.

www.iasbaba.com

41

Current Affairs- Subject wise compilation of 60 Day Plan- 2016


https://www.youtube.com/watch?v=BhNcjLy5CeA
Q.93) The 2014 Nobel Prize for physics has been awarded to a trio of scientists in Japan
and the US for the invention of blue light emitting diodes (LEDs). Why the invention of
blue LED was so special?
1. It was the first LED to be invented.
2. Together with Green and red LED, white light could be produced.
3. Blue LED is the most difficult to make as it uses, Gallium nitride and not Gallium
Phosphide.
Select the correct code from the following:
a) 1 and 2
b) 2 and 3
c) 1 and 3
d) All of the above
Q.93) Solution (b)
LEDs are basically semiconductors that have been built so they emit light when they're
activated. Different chemicals give different LEDs their colors. Engineers made the first LEDs
in the 1950s and 60s. Early iterations included laser-emitting devices that worked only when
bathed in liquid nitrogen. At the time, scientists developed LEDs that emitted everything
from infrared light to green light but they couldn't quite get to blue. That required
chemicals, including carefully-created crystals that they weren't yet able to make in the lab.
A white LED light is easy to make from a blue one. Engineers use a blue LED to excite some
kind of fluorescent chemical in the bulb. That converts the blue light to white light.
Two of the prize winners, Isamu Akasaki and Hiroshi Amano, worked together on producing
high-quality gallium nitride, a chemical that appears in many of the layers in a blue LED. The
previous red and green LEDs used gallium phosphide, which was easier to produce. Akasaki
and Amano discovered how to add chemicals to gallium nitride semiconductors in such a
way that they would emit light efficiently. The pair built structures with layers of gallium
nitride alloys.
The third prize-winner, Shuji Nakamura, also worked on making high-quality gallium nitride.
He figured out why gallium nitride semiconductors treated with certain chemicals glow. He
built his own gallium nitride alloy-based structures.
Q.94) Consider the following with respect to Nilgai
1. Nilgai is endemic to Indian subcontinent
2. It is the largest antelope in Asia
3. It is vulnerable as per IUCN red list

www.iasbaba.com

42

Current Affairs- Subject wise compilation of 60 Day Plan- 2016


Select the correct answer using the codes given below
a) 1 and 3 only
b) 1 and 2 only
c) 2 and 3 only
d) All
Q.94) Solution (b)
It is least concerned
http://www.thehindu.com/news/national/other-states/revered-nilgai-turns-farmersenemy/article8711052.ece
http://www.thehindu.com/news/cities/Delhi/nilgai-blues-hit-capitalsheart/article8652515.ece
Q.95) Consider the following countries
1. Honduras
2. Belize
3. El Salvador
Select the countries not bordering Mexico
a) 1 and 3 only
b) 1 and 2 only
c) 2 and 3 only
d) All border Mexico
Q.95) Solution (a)
Guatemala and Belize are the two countries in the south bordering Mexico
Mexico is in news for backing India for NSG membership. Also Mexican President took a
drive with our PM for dinner
Learn briefly about Mexico in particular. You never know UPSC :P
Q.96) Consider the following statements with respect to SURYAMITRA Mobile App
1. It is developed by National Institute of Solar Energy (NISE)
2. SURYAMITRA is a skill development program designed to create skilled manpower in
installation, commissioning, and Operation & Maintenance of solar power plants and
equipment.
Select the correct option
a) 1 only
b) 2 only
c) Both
d) None

www.iasbaba.com

43

Current Affairs- Subject wise compilation of 60 Day Plan- 2016

Q.96) Solution (c)


http://pib.nic.in/newsite/erelease.aspx?relid=0
Q.97) Hand in Hand is a joint military exercise conducted between which of the
following two countries
a) India and Sri Lanka
b) Sri Lanka and China
c) China and India
d) China and Bangladesh
Q.97) Solution (c)
http://pib.nic.in/newsite/PrintRelease.aspx?relid=128642
Q.98) Consider the following statements regarding Gold monetisation Scheme
1. Its objective is to mobilize unutilized gold from individuals, households and institutions
and make them available to gold-base industries
2. The tenure of gold deposits is likely to be for a minimum of two years.
3. The minimum quantity of deposits is pegged at 10 gram to encourage even small deposits
Choose the incorrect answer using the code given below
a) 1 and 3 only
b) 2 and 3 only
c) 1 and 2 only
d) All the above
Q.98) Solution (b)
Gold Monetisation Scheme, which would replace both the present Gold Deposit and
Gold Metal Loan Schemes. The new scheme would allow the depositors of gold to earn
interest in their metal accounts and the jewellers to obtain loans in their metal account.
Banks/ other dealers would also be able to monetise this gold.
The tenure of gold deposits is likely to be for a minimum of one year. The minimum
quantity of deposits is pegged at 30 gram to encourage even small deposits. The gold
can be in any form, bullion or jewellery.
http://www.thehindu.com/business/Industry/government-announces-third-round-of-goldbond-scheme/article8314695.ece
Q.99) Match List I with List II and select the correct answer using the code given below the
Lists:
LIST I
LIST II

www.iasbaba.com

44

Current Affairs- Subject wise compilation of 60 Day Plan- 2016


A. Tuber
B. Bulb
C. Rhizome
D. Corm

1.Saffron
2.Potato
3.Garlic
4.Turmeric

A-B-C-D
a) 3-4-1-2
b) 2-3-4-1
c) 2-4-3-1
d) 2-3-1-4
Q.99) Solution is (b)
Explanation:
Underground stem
1. Tuber like potato
2. Corm like Colocasia, Saffron
3. Bulb like Onion, Garlic etc.
4. Rhizome like Turmeric, Ginger etc.
Q.100) Match the following
1. Operation Cactus Srilanka
2. Operation Pawan Maldives
3. Operation Green Hunt China
Select the correct match of Indian Military Operations with other countries
a) 1 and 2
b) Only 2
c) 2 and 3
d) None
Q.100) Solution (d)
Operation Cactus was in news recently (It was with Maldives)
http://www.thehindu.com/specials/indiamaldives-relations-at-a-glance/article8460835.ece
Operation Green Hunt- Anti-Naxalite Operation of GoI
Operation Pawan- Srilanka
Q.101) Consider the following
1. Evaporation of gas
2. Compression of the gas
3. Solubility of Gas

www.iasbaba.com

45

Current Affairs- Subject wise compilation of 60 Day Plan- 2016


4. Expansion of the gas
5. Diffusion of gas
Which of the above phenomenon (in liquid medium) will increase with the increase in
Temperature?
a) 1, 2, 3, 4 and 5
b) 1, 4 and 5
c) 1, 3, 4 and 5
d) 2, 4 and 5
Q.101) Solution (b)
NCERT- Class 9th, Chapter 1
In understanding the effects of temperature on the solubility of gases, it is first important to
remember that temperature is a measure of the average kinetic energy. As temperature
increases, kinetic energy increases. The greater kinetic energy results in greater molecular
motion of the gas particles. As a result, the gas particles dissolved in the liquid are more
likely to escape to the gas phase and the existing gas particles are less likely to be dissolved.
The converse is true as well. The trend is thus as follows: increased temperatures mean
lesser solubility and decreased temperatures mean higher solubility.
Le Chatelier's principle allows better conceptualization of these trends. First, note that the
process of dissolving gas in liquid is usually exothermic. As such, increasing temperatures
result in stress on the product side (because heat is on the product side). In turn, Le
Chatelier's principle predicts that the system shifts towards the reactant side in order to
alleviate this new stress. Consequently, the equilibrium concentration of the gas particles in
gaseous phase increases, resulting in lowered solubility.
Conversely, decreasing temperatures result in stress on the reactant side (because heat is
on the product side). In turn, Le Chtelier's principle predicts that the system shifts toward
the product side in order to compensate for this new stress. Consequently, the equilibrium
concentration of the gas particles in gaseous phase would decrease, resulting in greater
solubility.
Q.102) Which of the following schemes will promote Urban Infrastructure?
1. Swatch Bharat Abhiyan
2. Smart City Scheme
3. Bharat Nirman
4. Indira Awas Yojana
5. HRIDAY
Select the correct code

www.iasbaba.com

46

Current Affairs- Subject wise compilation of 60 Day Plan- 2016


a) 1, 3 and 4
b) 2, 4 and 5
c) 1, 2 and 5
d) All
Q.102) Solution (c)
Bharat Nirman and IAY- Rural infrastructure schemes
Geoengineers have long explored the possibility of sealing CO2 gas in voids underground,
such as in abandoned oil and gas reservoirs, but these are susceptible to leakage. So
attention has now turned to the mineralisation of carbon to permanently dispose of CO2.
Q.103) Touch screens now have become an important component of many electronic
devices like mobiles, ATM machines, laptops etc. Consider the following statements
regarding touch screens:
1. Commonly there are two types of touch screens- resistive and capacitive.
2. Capacitive touch screens are multilayered with inner most and outer most layers
behaving as a conductor.
3. Resistive screens can detect more than one touch at once at different spots.
Which of the above statements are correct?
a) 1 and 2
b) 2 and 3
c) 1 and 3
d) All of the above
Q.103) Solution (a)
There are several types of touch screen technologies available, but of these, two are most
common. The first is the Resistive touch screen which works a bit like transparent
keyboard overlaid on top of the screen. There's a flexible upper layer of conducting
polyester plastic bonded to a rigid lower layer of conducting sheet and these are separated
by an insulating membrane. When the screen is pressed at a specific spot, it forces the
polyester to touch the glass and complete a circuit just like pressing the key on a
keyboard. A chip inside the screen figures out the coordinates of the place which is touched.
The other is the capacitive type screen. These screens are made from multiple layers. The
inner layer conducts electricity and so does the outer layer. Effectively the screen behaves
like two electrical conductors separated by an insulator in other words, a capacitor.
When the user brings his finger up to the screen, it alters the electrical field by a certain
amount that varies according to where the finger is. Capacitive screens can be touched in
more than one place at once.

www.iasbaba.com

47

Current Affairs- Subject wise compilation of 60 Day Plan- 2016


Q.104) Recently Zika virus caused a havoc in South American and central American
countries. Few cases have also been registered in USA. Which of the following statements
are correct about Zika virus?
1. It is disease caused by virus from infected Aedes mosquito.
2. Zika virus can be sexually transmitted.
3. Virus can pass through the fetus of a pregnant woman and can infect the brain cells of
the fetus.
4. The symptoms are similar to other arbovirus infections such as dengue, and include
fever, skin rashes, conjunctivitis, muscle and joint pain, malaise, and headache.
Select the code from the following:
a) 1,3 and 4
b) 1 and 3
c) 3 and 4
d) All of the above
Q.104) Solution (d)
Zika virus disease is caused by a virus transmitted primarily by Aedes mosquitoes.
People with Zika virus disease can have symptoms including mild fever, skin rash,
conjunctivitis, muscle and joint pain, malaise or headache. These symptoms normally last
for 2-7 days.
Sexual transmission of Zika virus has been documented in several different countries. To
reduce the risk of sexual transmission and potential pregnancy complications related to Zika
virus infection, the sexual partners of pregnant women, living in or returning from areas
where local transmission of Zika virus occurs should practice safer sex (including using
condoms) or abstain from sexual activity throughout the pregnancy.
Q.105) A two foot long but discovered in China has been declared as the worlds longest
insect. Which of the following species does this bug belongs to?
a) A Mantis
b) A Stick Insect
c) A centipede
d) A titan beetle
Q.105) Solution (b)
A bug, measuring over half-a-metre long, discovered in southern China has been declared
the world's longest insect.
A stick insect measuring 62.4 centimetres found two years ago in the southern province of
Guangxi has broken the record for length amongst the world's 807,625 known insects, the
official Xinhua agency said, citing the Insect Museum of West China.

www.iasbaba.com

48

Current Affairs- Subject wise compilation of 60 Day Plan- 2016

Q.106) Consider the following statements:


1. A black hole is a place in space where gravity pulls so much that even light cannot get out.
2. Density of black hole is very high.
3. Black holes are made when the fuel of a star gets over and it becomes a white dwarf.
Which of the above statements are correct?
a) 1 and 2
b) 2 and 3
c) 1 and 3
d) All of the above
Q.106) Solution (a)
A black hole is a place in space where gravity pulls so much that even light cannot get out.
The gravity is so strong because matter has been squeezed into a tiny space. This can
happen when a star is dying.
Because no light can get out, people can't see black holes. They are invisible. Space
telescopes with special tools can help find black holes. The special tools can see how stars
that are very close to black holes act differently than other stars.
Stellar black holes are made when the center of a very big star falls in upon itself, or
collapses. When this happens, it causes a supernova. A supernova is an exploding star that
blasts part of the star into space. Scientists think supermassive black holes were made at the
same time as the galaxy they are in.
Small stars become white dwarfs when there fuel gets over.
Q.107) Hepatitis is the medical term for inflammation of the liver. Consider the following
regarding this
1. There are five types of viruses responsible for Hepatitis
2. Intake of toxic substances, alcohol and certain drugs are some of the causes of Hepatitis
3. Hepatitis B is transmitted through exposure to infective blood, semen, and other body
fluids.
Select the correct code
a) 1 and 2
b) 2 and 3
c) 1 and 3
d) 1, 2 and 3
Q.107) Solution (d)

www.iasbaba.com

49

Current Affairs- Subject wise compilation of 60 Day Plan- 2016


http://www.thehindu.com/news/cities/bangalore/hepatitis-b-virus-more-prevalent-thanhiv-in-donor-blood/article8725933.ece
http://www.thehindu.com/news/cities/chennai/its-a-long-road-to-recovery-for-hepatitis-cpatients-at-stanley/article8553352.ece
http://www.who.int/features/qa/76/en/
http://www.thehindu.com/sci-tech/health/two-million-people-coinfected-with-hivhepatitis-c-globally/article8341900.ece
Q.108) Consider the following statements with respect to Maglev Technology
1. Maglev trains does not have engine.
2. It does not emit greenhouse gases.
3. It is generally preferred for short distance commutation.
Which of the above given statements is/are correct?
a) 2 only
b) 1 & 2 only
c) 2 & 3 only
d) None of the above
Q.108) Solution (b)
Maglev Technology is generally used for Mono rails which doesnt operates based upon
engines. Moreover the Magnetic levitation doesnt use any kind of fossil fuels so it wont
release any of the greenhouse gases.
Monorail system is preferred for long distance commutation only metro rail is preferred
for short distance commutation.
Q.109) Consider the following statements about Bio Digester Technology
1. Its a green technology developed by CSIR(Council of Scientific & Industrial Research)
2. It uses anaerobic digestion method.
3. Process is carried out by using Bacteria and Fungi.
Which of the above given statements is/are correct?
a) 2 only
b) 1 & 2 only
c) 2 & 3 only
d) 1,2 & 3
Q.109) Solution (a)
Bio-digester technology is been carried out only with the help of bacteria and not by
using fungi. Moreover the same question can be asked in the perspective of Dark
Fermentation.

www.iasbaba.com

50

Current Affairs- Subject wise compilation of 60 Day Plan- 2016

It is developed by DRDO

Q.110) Consider the following statements with respect to Green Diesel


1. Green Diesel is generally referred to as Biodiesel.
2. It is produced through Hydro cracking technology.
3. It possesses same chemical properties as petroleum based diesel.
Which of the above given statements is/are not correct?
a) 1 only
b) 2 only
c) 1 & 2 only
d) 1, 2 & 3
Q.110) Solution (a)
Green diesel is entirely different from the Bio diesel.
The major difference between them is the process of making. Biodiesel is processed by
using transesterification, while the green diesel is processed by fractional distillation like
fossil origin by Hydrocracking technology.
Moreover the chemical composition between them is also different.
Green diesel possesses same chemical properties as petroleum based diesel.
Q.111) Consider the following
1. Kolkata Metro Rail Corporation
2. Chennai Metro Rail Limited
3. Rapid Metro Rail Gurgaon
Which of the above metro rail corporation works under Indian Railways?
a) 1 only
b) 2 and 3 only
c) 1 and 2 only
d) 1 and 3 only
Q.111) Solution (a)
Chennai Metro rail is operated by the Ministry of urban development. Rapid metro rail
Gurgaon is Indias first fully privately financed metro and the first metro system in the
country to auction naming rights for its stations.
Q.112) Nirbhaya Fund is set up under which of the following Ministry?
a) Ministry of Health and Family Welfare
b) Ministry of Women and Child Development
c) Ministry of Finance

www.iasbaba.com

51

Current Affairs- Subject wise compilation of 60 Day Plan- 2016


d) Minister of Home
Q.112) Solution (c)
Ministry of Finance has constituted Nirbhaya Fund with a corpus of 1000 crores
Q.113) Connecting link between ape and man is
a) Cro Magnon Man
b) Australopithecus
c) Neanderthal Man
d) Lemur
Q.113) Solution (b)
Explanation:
Australopithecus shows characters of both apes and man.
Q.114) Consider the following statements regarding project Loon:
1. It is an ambitious project of Facebook to provide network to rural and remote areas.
2. Helium balloons are used to carry electronic payloads and are placed in atmosphere over
a place where network towers are not available.
3. The balloons will be placed in Stratosphere.
Which of the above statements are correct?
a) 1 only
b) 2 and 3
c) 1 and 2 only
d) All of the above
Q.114) Solution (b)
Project Loon the project of Google to provide network connection in rural and remote
areas where network connection is not available.
Facebook is going to start a similar project, where it will be using drones rather than helium
balloons.
Q.115) Consider the following statements regarding Impacting Research Innovation and
Technology (IMPRINT) Project?
1. It is an initiative of Ministry of Human Resource Development and Pan IIT + IISc.
2. The project aims at finding engineering and scientific solutions to major social and
developmental needs like healthcare, energy, ICT, sustainable habitat etc.
Which of the above statements are incorrect?
a) 1 only

www.iasbaba.com

52

Current Affairs- Subject wise compilation of 60 Day Plan- 2016


b) 2 only
c) Both 1 and 2
d) Neither 1 nor 2
Q.115) Solution (d)
http://imprint-india.org/
Q.116) India will be getting its first High Speed Train from a Spanish Company called
Taglo. The company is globally known for faster and lighter trains. Consider the following
statements:
1. The train will be tested on Delhi-Mumbai Railway line.
2. It can be run at a speed of around 300 km/hr without any significant upgrade of railway
tracks.
3. Currently the fastest train of India is Gatiman Express which runs between Delhi and Agra.
Which of the above statements are correct?
a) 1 and 2
b) 2 and 3
c) 1 and 3
d) All of the above
Q.116) Solution (c)
The company has offered lighter and safer trains, which can be run at speeds of about 160
km an hour and without any major upgrade of existing tracks. The fastest train on Indian
Railways system, Gatimaan Express, runs at 160 km an hour between Delhi and Agra.
*Note: Question has been corrected for factual error. Taglo is a high speed train not a
bullet train.
Q.117) Consider the following statements with respect to Colombia
1. Quito is the capital of Colombia
2. Quito is one of the closest places on earth where equator passes through
Select the correct answer
a) 1 only
b) 2 only
c) Both
d) None
Q.117) Solution (b)
Bogota is the capital of Colombia
http://www.worldatlas.com/aatlas/newart/locator/equator.htm

www.iasbaba.com

53

Current Affairs- Subject wise compilation of 60 Day Plan- 2016


Q.118) 38th parallel is found between which of the following two countries
a) North Korea and South Korea
b) South Korea and China
c) North Korea and China
d) China and Japan
Q.118) Solution (a)
38th parallel is between North Korea and South Korea.
http://www.reuters.com/article/us-northkorea-missile-idUSKCN0Z72YF
Q.119) Consider the following statements with respect to Shanghai Cooperation
Organisation
1. It consists of all members of Central Asia including China and Russia
2. It is headquartered at Shanghai
Select the correct answer
a) 1 only
b) 2 only
c) Both
d) None
Q.120) Solution (d)
Turkmenistan is not the member of central Asia
The headquarters of SCO is located in Beijing
http://indianexpress.com/article/india/india-news-india/indias-sco-membership-willstrengthen-regions-security-pm-modi-in-tashkent-2873303/
Q.121) Consider the following statements with respect to Net Neutrality
1. It is the principle that ISPs and governments should treat all data on the Internet equally
not discriminating or charging differently by user, content site platform application, type
of attached equipment and modes of communication
2. It creates a level playing field for all web services and websites, having access to the
same connection speed
Select the incorrect option
a) 1 only
b) 2 only
c) Both
d) None
Q.121) Solution (d)

www.iasbaba.com

54

Current Affairs- Subject wise compilation of 60 Day Plan- 2016


http://www.thehindu.com/business/Industry/trai-issues-preconsultation-paper-on-netneutrality/article8672081.ece
Q.122) Consider the following statements about Rajiv Rinn Yojana
1. The RRY is a central sector scheme which provides an interest subsidy on housing loans
to EWS and LIG households
2. It replaces the Interest Subsidy Scheme for Housing the Urban Poor (ISHUP)
Select the correct option
a) 1 only
b) 2 only
c) Both
d) None
Q.122) Solution (c)
Rajiv Rinn Yojana (RRY) is an instrument to address the housing needs of the EWS/LIG
segments in urban areas, through enhanced credit flow. It is also formulated to channelize
institutional credit to the poorer segments of the society and increasing home ownership in
the country along with addressing housing shortage. RRY has been formulated by modifying
the Interest Subsidy Scheme for Housing the Urban Poor (ISHUP) piloted in the 11th Plan
period with enhanced scope and coverage. RRY is a Central Sector Scheme applicable in all
the urban areas of the Country.
http://mhupa.gov.in/User_Panel/UserView.aspx?TypeID=1496
Q.123) Consider the following statements about an Indian city:
1. It is known as a city of thousand temples.
2. The city has been an important centre for advanced studies in Jainism and Buddhism in
the past.
3. It is situated on the banks of R. Vegavathy.
Identify the city that matches the description stated above:
a) Puducherry
b) Thanjavur
c) Kanchipuram
d) Madurai
Q.123) Solution (c)
Kanchipuram (A PRASAD City)
Silk City: Handloom industry (weaving of silk sarees)
Pallavas ruled from Kanchipuram (the region around Kanchi known as Tondaimandalam)

www.iasbaba.com

55

Current Affairs- Subject wise compilation of 60 Day Plan- 2016

Architecture style: Dravidian (under rulers such as Mahendravarman and


Narsimhavarman)
Ekambareswarar temple (Hosts one of the tallest temple towers Vimana in India)
HQ: Kanchi matha
Founder: Hindu saint Adi Shankaracharya
Consolidated the doctrine of Advaita Vedanta
Discussed the unity of the tman and Nirguna Brahman "brahman without
attributes"
Explained the key difference between Hinduism and Buddhism Hinduism asserts
"Atman (Soul, Self) exists", while Buddhism asserts that there is "no Soul, no Self"
Introduced the Pacyatana form of worship the simultaneous worship of five
deities Ganesha, Surya, Vishnu, Shiva and Devi (all deities were but different forms
of the one Brahman, the invisible Supreme Being)
UNESCO Heritage Status:
Rock-cut temples known as Rathas on banks of Mamallapuram
Kailasanatha temple
Vaikuntha Perumal temple
Q.124) Identify the Report based on the following observations:
1. The report states that the cost of adapting to a changing climate will be four to five
times higher than what was estimated two years ago.
2. It has highlighted Indias intense and frequent heat waves, and has stated that the
annual costs associated with additional demand for cooling (in India) would escalate
further.
3. The Report includes a more in-depth review of national-level cost estimates (bottom-up
studies), and global-level, sector specific estimates, while providing additional globallevel model estimates (top-down estimates)
4. It is an annual report and was first published in the year 2014 by UNEP.
Choose the correct answer:
a) The Energy Report & Living Planet Report
b) Adaptation Gap Report
c) Report on Regular Resources
d) Global Environment Outlook
Q.124) Solution (b)
Adaptation Gap Report by UNEP
Adaptation Gap: The adaptation gap can be defined generically as the difference
between the level of adaptation actually implemented and a societally set target or goal,
which reflects nationally determined needs related to climate change impacts, as well as
resource limitations and competing priorities

www.iasbaba.com

56

Current Affairs- Subject wise compilation of 60 Day Plan- 2016

Adaptation Cost: Costs of planning, preparing for, facilitating, and implementing


adaptation measures, including transaction costs

I. The Energy Report & Living Planet Report: By WWF (World Wildlife Fund)
II. Report on Regular Resources: UNICEF (United Nations Childrens Emergency Fund)
III. Global Environment Outlook: UNEP (United Nations Environment Programme)
UNEP
HQ: Nairobi, Kenya
Earthwatch: For an efficient surveillance of the environment
UNEP members decided to protect biodiversity (80s) Drafted an agreement and
presented it during the Earth Summit (Rio-Brazil, 1992) Signatures led to what you
so fondly study today about Convention on Biological Diversity (CBD)
CBD = Cartagena + Aichi + Nagoya
Cartagena Protocol on Biosafety
Aichi Targets
Nagoya Protocol on Genetic Resources
Q.124) Consider the following statements about the sport Jallikattu
1. It is a bull taming sport and is organised during Pongal.
2. It is native to the state of Tamil Nadu and has been in practice since Sangam age.
3. Eruthattam is another name for Jallikattu.
Choose the incorrect statement/s from the options given below:
a) Only 1
b) Both 1 and 3
c) Both 2 and 3
d) Only 3
Q.124) Solution (d)
Eruthattam is a bull-chasing sport whereas Jallikattu is a bull-taming sport.
Q.125) Recently National Mineral Exploration Policy was cleared. Consider the following
statements about (NMEP)
1. Of Indias entire Obvious Geological Potential (OGP) area, identified by GSI, only 10 per
cent has been explored and mining is undertaken in 1.5-2 per cent of this area.
2. The revenue-sharing of the government and the private entities would strictly be in the
form of lump-sum, to be paid throughout the period of mining lease with nontransferable rights.
3. Mining leases (ML) and prospecting license-cum-mining lease (PL-cum-ML) will be
granted only through an auction process.

www.iasbaba.com

57

Current Affairs- Subject wise compilation of 60 Day Plan- 2016

Choose the incorrect statement/s from the option given below:


a) Only 2
b) Only 3
c) Both 1 and 2
d) Both 2 and 3
Q.125) Solution (a)
Both 1 and 3 are correct about National Mineral Exploration Policy (NMEP)
The revenue-sharing could be either in the form of lump-sum or an annuity, to be paid
throughout the period of mining lease with transferable rights.
Annuity payment: An annuity is a series of equal payments at regular intervals
(regular deposits to a savings account, monthly home mortgage payments, etc.)
Lump sum Payment: A single payment made at a particular time (atonego)
http://pib.nic.in/newsite/PrintRelease.aspx?relid=146629
Q.126) Identify the other name coined for Annual Meeting of the New Champions
a) Summer Dove
b) Human Capital Summit
c) Summer Davos
d) Economic Champions Summit
Q.126) Solution (c)
Summer Davos: The tenth Annual Meeting of the New Champions, or the Summer Davos
took place in Tianjin, China.
Context: Human Capital Index (HCI)
Theme of the forum: The Fourth Industrial Revolution and its Transformational Impact
HCI measures: A countrys ability to nurture, develop and deploy talent for economic
growth
India: 105th position globally
Q.127) Identify the goddess who is also referred to as the Menstruating Goddess
a) Goddess Kaali
b) Goddess Durga
c) Devi Kamakhya
d) Goddess Saraswati
Q.127) Solution (c)
Kamakhya Devi Temple: (ONLY PRASAD)

www.iasbaba.com

58

Current Affairs- Subject wise compilation of 60 Day Plan- 2016


Faith: Centre of Tantric and shakti cults of Hinduism; amongst the 51 Shakti Peethas related
to the cult of Sati
Location: Nilachal Hills, overlooking river Brahmaputra in Assam
Q.128) Identify from the following the cave that hosts the famous Kailasa Temple
a) Ajanta Caves
b) Karan Chaupar Cave
c) Junnar Cave
d) Ellora Cave
Q.128) Solution (d)
Q.129) Choose from among the following the Financial Regulators of India
1. RBI
2. CAG
3. SEBI
4. SIDBI
Choose the correct option/s from the following:
a) 1, 3 and 4
b) Only 1
c) Both 1 and 3
d) All of the above
Q.129) Solution (c)
Q.130) Which is the nodal agency for the National Bamboo Mission?
a) Ministry of New & Renewable Energy
b) Ministry of Agriculture
c) Ministry of Environment, Forest & Climate Change
d) Federation of Green Energy
Q.130) Solution (b)
National Bamboo Mission
Launch: 2006-07
Called: An evergreen plant
3-day world conference: Indore (MP)
Q.131) Identify the Ministry which has launched the programmes HAPIS & CHAMAN:
a) Department of Nuclear Supplies
b) Integrated watershed development authority

www.iasbaba.com

59

Current Affairs- Subject wise compilation of 60 Day Plan- 2016


c) Ministry of Environment, Forest and Climate Change
d) Ministry of Agriculture
Q.131) Solution (d)
CHAMAN:
Coordinated Programme on Horticulture Assessment and Management using
geoiNformatics
Remote sensing + Sample Survey To assess data the area and production of major
horticulture crops
HAPIS:
Horticulture Area and Production Information System
Web portal for online submission of district level dataareas & production of
horticulture crops
Q.132) Centre for Advanced Animal Diagnostics and Services on Animal Health and
Diseases (ADSAHD) to prevent cattle from exotic diseases, is being developed in which of
the following regions?
a) North eastern Region
b) Haryana, Punjab and Rajasthan
c) Uttar Pradesh, Madhya Pradesh and Bihar
d) Andhra Pradesh and Telangana
Q.132) Solution (a)
The North Eastern Region of India, owing to its unique geographical location sharing five
international borders, bears constant threat of exotic trans-boundary diseases of our
valuable livestock. This programme is aimed at strengthening regional infrastructure and
capabilities for developing latest diagnostics and organizing rigorous surveillance for the
highly contagious and ravaging diseases so that forecasting model on disease outbreaks in
the region can be developed for a formidable defence to guard the territories.
Q.133) Consider the following Joint Military Exercises and participating countries
1. IBSAMAR :: India Brazil and Argentina
2. Garuda Shakti :: India and Sri Lanka
3. Yudh Abhyas :: India and Nepal
4. Nomadic Elephant :: India and Bhutan
Which of the following is incorrectly matched?
a) 1,2 and 3
b) 2 and 3
c) 1,2 and 4

www.iasbaba.com

60

Current Affairs- Subject wise compilation of 60 Day Plan- 2016


d) All of the above
Q.133) Solution (d)
IBSAMAR :: India, Brazil and South Africa
Garuda Shakti :: India and Indonesia
Yudh Abhyas :: India and USA
Nomadic Elephant :: India and Mongolia
Q.134) Consider the following statements:
1. It is used as an ingredient in food preparation and is also utilized as feed for poultry and
livestock.
2. It is a feedstock for biofuel.
3. It is used for starch production
4. It is also used as a raw material in distilleries.
Select the correct answer based on the above statements:
a) Jatropha
b) Sugar
c) Maize
d) Rice straw
Q.134) Solution (c)
It is used for starch production has high starch content (60%)
Q.135) Which Indian city is known as the shrimp capital of India?
a) Vishakhapatnam
b) Goa
c) Kolkata
d) Nellore
Q.135) Solution (d)
Nellore in Andhra Pradesh is known as the shrimp capital of India.
Q.136) Where is Shani Shingapur temple located?
a) West Bengal
b) Kerala
c) Maharashtra
d) Andhra Pradesh
Q.136) Solution (c)

www.iasbaba.com

61

Current Affairs- Subject wise compilation of 60 Day Plan- 2016


In news: Shani Shingapur (Maharashtra) & Sabrimala Temple (Kerala) facing challenges
related to the restrictions on entry of women in their premises.
Q.137) Where is China planning to build its first African naval base?
a) Somalia
b) Djibouti
c) Tanzania
d) Kilwa
Q.137) Solution (b)
Djibouti
1. A country located in the Horn of Africa
2. Located near the world's busiest shipping lanes, controlling access to the Red Sea and
Indian Ocean

Q.138) Vidya Lakshmi Portal would serve the purpose of


a) Integrating various government incentives for scholarship on one platform for easy
access
b) Being a single window for students to access information & submit applications for
educational loans to banks and for availing government scholarships
c) Being a single window for the interaction between teachers and parents
d) Helping students participate in the Apprenticeship Scheme of the government
Q.138) Solution (b)
In news: Banks have been asked by the Finance Ministry to integrate with Vidya Lakshmi
Portalto tackle the menace of NPAs in the education sector
Q.139) In which country did Jasmine Revolution take place?
a) Kenya
b) Libya
c) Tunisia
www.iasbaba.com

62

Current Affairs- Subject wise compilation of 60 Day Plan- 2016


d) Sri Lanka
Q.139) Solution (c)
In News: National Dialogue Quartet was awarded the 2015 Noble Prize for "its decisive
contribution to the building of a pluralistic democracy in Tunisia in the wake of the Tunisian
Revolution of 2011"
Jasmine Revolution
An intensive campaign of civil resistance, including a series of street demonstrations
high unemployment, food inflation, corruption, a lack of political freedoms like freedom
of speech and poor living conditions
Led to a thorough democratization of the country and to free and democratic
electionsthe most dramatic wave of social and political unrest in Tunisia in three
decades.
Q.140) Which organization publishes the Global Money Laundering Report?
a) World Bank
b) IMF
c) Bank for International Settlements
d) Financial Action Task Force
Q.140) Solution (d)
Self explanatory
Q.141) Identify the first mission amongst the following to have successfully landed a fully
operational spacecraft on Mars:
a) Viking I
b) Voyager I
c) Voyager II
d) Apollo II
Q.141) Solution: a
Viking I 1976
Obtain high resolution images of the Martian surface
Study the characteristics and composition of the Martian surface & atmosphere
Also, search for the presence of life at the planet
Q.142) Pellet guns are very much in news in the state of Jammu & Kashmir. Identify the
substance from the following, which makes up the pellets in a pellet gun
a) Granite
b) Lead
c) Cadmium

www.iasbaba.com

63

Current Affairs- Subject wise compilation of 60 Day Plan- 2016


d) Mercury
Q.142)Solution: b
Pellet Guns
Usage: A form of non-lethal crowd control methods employed by the police and military
Non-lethal: Can penetrate soft tissues
Usage areas: Kashmir Valley + Areas affected by Left Wing Extremism (LWE)
Q.143) Consider the following statements:
1. In a preferential trade agreement, the tariff between the countries in the trading bloc, is
completely abolished.
2. India-Mercosur Preferential Trade Agreement (PTA) was formally referred to as the
Bangkok Agreement.
Choose the option indicating the incorrect statement/s
a) Only 1
b) Only 2
c) Both 1 and 2
d) None of the above
Q.143) Solution (c)
Preferential Trade Agreement (PTA)
A trading bloc providing preferential access to certain products amongst the
participating countries
Tariffs are reduced (and not abolished)
1st Stage: Economic Integration
Bangkok Agreement: Asia-Pacific Trade Agreement (APTA) initiative of the United
Nations Economic and Social Commission for Asia and the Pacific (ESCAP: Regional
development arm of the United Nations for the Asia-Pacific region)
India-Mercosur Preferential Trade Agreement (PTA)
A trading bloc in Latin America comprising Brazil, Argentina, Uruguay, Venezuela and
Paraguay
Third largest integrated market after the European Union (EU) and North American Free
Trade Agreement (NAFTA)
Let us revise Economic Survey regarding Preferential Trade Agreements
Forms of PTAs
A. Partial Scope Agreement (PSA): allows for trade between countries on a small number of
goods

www.iasbaba.com

64

Current Affairs- Subject wise compilation of 60 Day Plan- 2016


B. Free Trade Agreement (FTA): A preferential arrangement in which members reduce
tariffs on trade among themselves, while maintaining their own tariff rates for trade with
non-members
C. Customs Union (CU): A free-trade agreement in which members apply a common
external tariff (CET) schedule to imports from non-members.
D. Common Market (CM): A customs union where movement of factors of production is
relatively free amongst member countries.
E. Economic Union (EU): A common market where member countries coordinate macroeconomic and exchange rate policies.
India & FTAs
Increased trade has been more on the import than export side India maintains relatively
high tariffs and hence had larger tariff reductions than its FTA partners
Outside Asia: Chile (2006) and MERCOSUR (2004)
Within Asia: Sri Lanka (1998), Afghanistan (2003), Thailand (2004), Singapore (2005), Bhutan
(2006), Nepal (2009), Korea (2009), Malaysia (2011) and Japan (2011)
Regional trade agreements: South Asian Free Trade Agreement (SAFTA, 2004) and the
India-Association of Southeast Asian Nations Agreement (ASEAN, 2010)
Biggest trade impact: ASEAN FTA
Benefitted on both sides of trade flows
Greatest reduction in Indian import tariffs
Studies indicate:
Differences in the coverage of products and degree of integration across recent FTAs
Give rise not only to beneficial trade creation but also to trade diversion
A significant increase in imports through the Most Favoured Nation (MFN) route, and
recommends relaxation of structural and regulatory factors to promote exports
One of the biggest barriers to trade according to the study is distance costs (transport
costs, costs related to language, currency, policy etc.)
Need for a better way to evaluate the benefits achieved and the loopholes present
Study needs to be conducted on parameters determining trade in the absence of an
FTA
More work is needed to enrich our analysis and extend it to services.
Refer:
Trans-Pacific Partnership (TPP) & India
The new Great Game in Asia
RCEP
The RCEP Effect on India
INDIA-ASEAN TRADE TIES

www.iasbaba.com

65

Current Affairs- Subject wise compilation of 60 Day Plan- 2016


Q.144) Consider the following
1. International Day for Older Person (IDOP) is celebrated every year on 1st October
2. The recent theme of IDOP was Leaving No One Behind: Promoting a Society for All
3. 2015, marks the golden jubilee of IDOP
Select the correct code
a) 1 and 2
b) Only 1
c) 1, 2 and 3
d) Only 3
Q.144) Solution (b)
The International Day of Older Persons is observed on October 1 each year.
On December 14, 1990 the United Nations General Assembly voted to establish October 1
as the International Day of Older Persons as recorded in Resolution 45/106. The holiday was
observed for the first time on October 1, 1991
Annual Themes
2011: The Growing Opportunities & Challenges of Global Ageing
2012: Longevity: Shaping the Future
2013: The future we want: what older persons are saying
2014: Leaving No One Behind: Promoting a Society for All
2015: Sustainability and Age Inclusiveness in the Urban Environment
Q.145) Consider the following regarding Trident Juncture and select the correct code?
a) It is the largest military exercise launched by NATO
b) It is a trilateral exercise of India, Indonesia and Japan
c) It is a bilateral military exercise between Srilanka and India
d) It is a trilateral military exercise of NATO, EU and USA
Q.145) Solution (a)
North Atlantic Treaty Organization (NATO) has launched largest exercise Trident Juncture
since 2002 with troops from 30 states, including non-NATO in Italy, Spain and Portugal.
30 nations will participate in the drills which are taking place at sea, in the air and across the
territory of three European states.
Q.146) Mission Indradhanush aims to cover which of the following diseases?
1. Measles
2. AIDS
3. Cancer
4. Polio

www.iasbaba.com

66

Current Affairs- Subject wise compilation of 60 Day Plan- 2016


5. TB
6. Whooping Cough
Select the correct code
a) 1, 3, 4 and 5
b) 3, 4, 5 and 6
c) 1, 4, 5 and 6
d) 1, 2, 3, 4 and 5
Q.146) Solution (c)
Mission Indradhanush: for vaccination against 7 preventable disease by 2020.
The diseases are Diphtheria, Whooping cough, Tetanus, Polio, Tuberculosis, Measles and
Hepatitis B.
The aim is to cover all the children who have been left out or missed out for immunization.
Q.147) Consider the following about Nobel Prize, 2015
1. Nobel Prize in Physics has been given for the discovery of neutrino oscillations, which
shows that neutrinos have mass
2. Nobel Prize in Chemistry has been given for the discoveries concerning a novel therapy
against infections caused by roundworm parasite.
3. Nobel Prize in Medicine has been given for mechanistic studies of DNA repair.
Select the correct code
a) 1 and 2
b) Only 2
c) Only 1
d) 1, 2 and 3
Q.147) Solution (c)
The Nobel Prize in Physics 2015
Takaaki Kajita and Arthur B. McDonald
"for the discovery of neutrino oscillations, which shows that neutrinos have mass"
The Nobel Prize in Chemistry 2015
Tomas Lindahl, Paul Modrich and Aziz Sancar
"for mechanistic studies of DNA repair"
The Nobel Prize in Physiology or Medicine 2015
William C. Campbell and Satoshi mura
"for their discoveries concerning a novel therapy against infections caused by roundworm
parasites"
Youyou Tu
"for her discoveries concerning a novel therapy against Malaria"

www.iasbaba.com

67

Current Affairs- Subject wise compilation of 60 Day Plan- 2016

Q.148) Consider the following statements regarding Compost


1. Organic ingredients intended for composting can alternatively be used to generate
biogas through aerobic digestion
2. Compost is used to control soil erosion and sediment run off.
Select the correct code
a) Only 1
b) Only 2
c) Both
d) None
Q.148) Solution (b)
Compost is rich in nutrients. It is used in gardens, landscaping, horticulture, and agriculture.
The compost itself is beneficial for the land in many ways, including as a soil conditioner, a
fertilizer, addition of vital humus or humic acids, and as a natural pesticide for soil. In
ecosystems, compost is useful for erosion control, land and stream reclamation, wetland
construction, and as landfill cover. Organic ingredients intended for composting can
alternatively be used to generate biogas through anaerobic digestion.
Topsoil loss is a serious ecological issue. The use of compost to control sediment run-off and
fight erosion is a relatively new technology, now being adopted by local authorities,
developers, farmers, and other major disturbers of soil as another tool to reduce topsoil
loss.
http://www.thehindu.com/todays-paper/tp-national/firms-must-buy-compost-frommunicipal-waste/article8230689.ece
Q.149) Consider the following regarding Biochar
1. It is made up of biomass by a process called combustion
2. It is an unstable solid formed by charcoal
3. It helps in increasing acidity of the soil when the PH of the soil is high
Select the correct code
a) 1 and 2
b) 1, 2 and 3
c) Only 3
d) None
Q.149) Solution (d)

www.iasbaba.com

68

Current Affairs- Subject wise compilation of 60 Day Plan- 2016


Bio-char reduces the acidity of the soil, protects the plants from diseases, promotes growth
of friendly micro organisms, and reduces the loss of micro nutrients apart from increasing
water retain-ability.
http://www.thehindu.com/business/agri-business/biochar-compost-tea-are-the-newrecipe/article7830057.ece
https://en.wikipedia.org/wiki/Biochar
Q.150) Consider the statements regarding the use of Antibiotics
1. Flu or cold can be treated by using antibiotics
2. Antibiotic resistance is caused when body becomes resistant to antibiotics
3. Antibiotic resistance is caused by genetic mutation and by acquiring resistance from
another bacterium
Select the correct code
a) 1, 2 and 3
b) 2 and 3
c) Only 3
d) 1 and 2
Q.150) Solution (c)
Because cold or flu is caused by a virus, while antibiotics can only cure infections caused by
bacteria. (You might need antibiotics only if your flu gets worse and results in a secondary
bacterial infection like bronchitis or pneumonia.)
One should understand that it is the bacteria which become resistant to antibiotics, not the
body. Through indiscriminate antibiotics use, you are helping create drug-resistant bacteria,
which then cause difficult-to-treat infections in others.
Some bacteria are naturally resistant to certain types of antibiotics. However, bacteria may
also become resistant in two ways: 1) by a genetic mutation or 2) by acquiring resistance
from another bacterium.
http://www.thehindu.com/news/national/kerala/no-longer-a-cureall/article7907429.ece
Q.151) National Tribal Advisory Council to be set up is to be chaired by?
a) President of India
b) Prime Minister of India
c) Ministry of Tribal Affairs
d) Prime Minister Office (PMO)
Q.151) Solution (b)

www.iasbaba.com

69

Current Affairs- Subject wise compilation of 60 Day Plan- 2016


A National Tribal Advisory Council under the chairmanship of Prime Minister will be set up
for real time monitoring of the ongoing programmes and schemes aimed at welfare of tribal
people and development of tribal areas. The Council will meet once or twice a year
Q.152) The cases of HIV infected blood transfusion have seen a rise. Consider the
following
1. It is mandatory to screen the blood donated for traces of HIV, Hepatitis C, malaria and
syphilis before transfusion
2. NACO is an autonomous body under Department of Health and Family Welfare,
Government of India which is primarily responsible for controlling spread of HIV
infection
3. India has the highest number of population living with HIV in the world as per UN report
Choose the correct code
a) Only 2
b) 2 and 3
c) 1 and 2
d) 1, 2 and 3
Q.152) Solution (c)
India has the third largest number of people living with HIV in world (2.1 million). Top two
are South Africa and Nigeria. [UNAIDS Gap report]. Around 36% of Indian adults with the
virus have access to antiretroviral treatment, the report said. An annual report by Indias
National AIDS Control Organization noted that the transgender population was emerging as
a risk group with high vulnerability and high levels of HIV.
Q.153) Operation Hill Vijay is being undertaken to declare Parasnath jungles Naxal-free.
Consider the following regarding famous Parasnath Hill
1. Parasnath hill (Shikharji) is located towards the eastern end of the Chota Nagpur Plataea
2. It is a famous Jain pilgrimage site
3. Bhagwan Parshwanath was the last Tirthankara
Select the correct code
a) Only 1
b) 1 and 2
c) 2 and 3
d) 1 and 3
Q.153) Solution (b)
He was not the last Tirthankara.
Learn about Operation Vijay Hill

www.iasbaba.com

70

Current Affairs- Subject wise compilation of 60 Day Plan- 2016


http://www.thehindu.com/news/national/jharkhand-dgp-supervises-operation-hillvijay/article8673129.ece
Q.154) Consider the following regarding Global Slavery Index 2016
1. India has the distinction of having the highest number of people in the world trapped in
modern slavery
2. 2016 Global Slavery Index is released by Australia based human right group, Walk Free
Select the correct code
a) Only 1
b) Only 2
c) Both
d) None
Q.154) Solution (c)
http://www.business-standard.com/article/current-affairs/india-tops-global-slavery-indexwith-18-35-million-people-enslaved-116053100436_1.html
Q.155) Consider the following
1. Rhizobium
2. Clostridium
3. Azotobacter
4. Pseduomonas
Which of the above is/are free living nitrogen bacteria present in the soil?
a) 1 and 2 only
b) 2 and 3 only
c) 1 and 3 only
d) 1,2 and 3
Q.155) Solution (b)
Rhizobium is a present in the legumes of a plant.
Pseduomonas is denitrifying bacteria
Q.166) Consider the following statements with respect to GSLV Mark 3
1. It features an Indian cryogenic technology
2. It is used to launch only remote sensing satellite in geostationary orbit
3. It is currently used to launch satellites weighing more than 3 tons
Select the incorrect option
a) 1 and 3 only

www.iasbaba.com

71

Current Affairs- Subject wise compilation of 60 Day Plan- 2016


b) 2 and 3 only
c) 1 and 2 only
d) 1,2 and 3
Q.166) Solution (b)
It is used to launch communication satellite and it is currently under development
Q.167) Consider the statements
1. Moons gravity at Earth is much stronger than Suns
2. Lunar tidal force is stronger than solar tidal force
Select the correct code
a) Only 1
b) Only 2
c) Both
d) None
Q.167) Solution (b)
Tides are caused by tidal forces, and the answer to your question lies in the definition of a
tidal force. A tidal force is related to gravity, but it isn't the same thing. It's really the
difference between the strength of gravity at two locations.
The gravitational attraction between two objects (say the Earth and the Moon) decreases
with distance. This means that the Moon's gravity pulls most strongly on the side of the
Earth closest to the Moon and least strongly on the side of the Earth farthest from the
Moon. Tidal forces on the side of Earth closest to the Moon pull material (mostly water)
toward the Moon. Tidal forces on the other side of Earth actually pull material away from
the Moon. The resulting deformation of Earth looks the same when the moon is at opposite
sides of its orbit, like full moon and new moon or first quarter and third quarter. That's why
tides around the equator are higher during both a new moon and a full moon.
The Sun also affects the Earth's tides. However, tidal forces due to the Sun are about half as
strong as those due to the Moon. This seems strange, because the Sun's gravity at Earth is
much stronger than the Moon's. But remember that tides concern the difference between
gravity's pull at opposite sides of the Earth. The radius of the Earth is a very small fraction of
the distance between the Sun and the Earth, about 0.005%. As a result, the difference
between the Sun's gravitational pull on either end of the Earth is small. In contrast, the
radius of the Earth is about 1.7% of the distance between the Earth and the Moon. So even
though the Moon's gravity isn't as strong as the Sun's, lunar tidal forces are stronger than
solar tidal forces, so lunar tides are stronger than solar tides.

www.iasbaba.com

72

Current Affairs- Subject wise compilation of 60 Day Plan- 2016


Q.168) Consider the following statements with respect to National Pension Scheme
1. National pension scheme was launched in the year 2015
2. NRIs may subscribe to the NPS as an investment option under FEMA 1999
3. NPS has been provided for all citizens of the country including the unorganized sector
workers on voluntary basis.
Select the correct option with respect to NPS
a) 1 and 2 only
b) 2 and 3 only
c) 1 and 3 only
d) All
Q.168) Solution (b)
The National Pension System (NPS) was launched on 1 January, 2004 with the objective
of providing retirement income to all the citizens.
RBI took the decision in consultation with the Union Government to enable National
Pension System (NPS) as an investment option for NRIs under Foreign Exchange
Management Act (FEMA), 1999.
NRIs may subscribe to the NPS governed and administered by the Pension Fund
Regulatory and Development Authority (PFRDA)
NPS aims to institute pension reforms and to inculcate the habit of saving for retirement
amongst the citizens.
Initially, NPS was introduced for the new government recruits (except armed forces).
With effect from 1 May, 2009, NPS has been provided for all citizens of the country
including the unorganized sector workers on voluntary basis.
Q.169) The Government of India scheme Vikalp refers to
a) It aims to provide confirmed accommodation to waitlisted passengers in alternate trains.
b) It aims to ensure 100 percent attendance in tribal areas.
c) The scheme aims to provide free electricity in rural areas
d) The scheme aims to ensure safety of women passengers in public transport through the
use of ICT
Q.169) Solution (a)
Indian Railways has launched Alternate Train Accommodation Scheme (ATAS) called as
Vikalp to provide confirmed accommodation to waitlisted passengers in alternate trains.
Q.170) Consider the following statements with respect to Olive Ridely turtles
1. They are also known as Pacific Ridely sea turtles.
2. Olive Ridley sea turtle is placed under Schedule - I of Indian Wildlife (Protection) Act,
1972 (amended 1991).

www.iasbaba.com

73

Current Affairs- Subject wise compilation of 60 Day Plan- 2016


3. It is endangered as per IUCN red list
Select the correct option
a) 1 and 2 only
b) 2 and 3 only
c) 1 and 3 only
d) All
Q.170) Solution (a)
Also known as the Pacific Ridley Sea Turtle, it is a medium-sized species of sea turtle
found in warm and tropical waters, primarily in the Pacific and Indian Oceans.
Gahirmatha Beach in Kendrapara district of Odisha, a part of the Bhitarkanika Wildlife
Sanctuary
Bhitarkanika Mangroves were designated a Ramsar Wetland of International Importance
in 2002
Olive Ridley sea turtle has found place in Schedule - I of Indian Wildlife (Protection) Act,
1972 (amended 1991).
All the species of sea turtles in the coastal water of Orissa are listed as "vulnerable" as
per IUCN Red Data Book.
The Sea Turtles are protected under the 'Migratory Species Convention' and CITES
(Convention of International Trade on Wildlife Flora and Fauna). India is a signatory
nation to all these conventions
Q.171) Which of the following statements regarding H1N1 is/are correct?
1. It is transmitted by an Influenza A virus subtype H1N1
2. The H stands for hemagglutinin. And N for neuraminidase- both proteins on the outer
layer of the virus
3. Swine influenza is a respiratory disease that occurs in pigs that is caused by the Influenza
A virus.
4. It is a Non-contagious disease
Select the correct answer using the code given below:
a) 1 ,2 and 3 only
b) 2 and 4 only
c) 1, 2 and 4 only
d) All the above
Q.171) Solution (a)
It is transmitted by an Influenza A virus subtype H1N1.The H stands for hemagglutinin.
And N for neuraminidase- both proteins on the outer layer of the virus

www.iasbaba.com

74

Current Affairs- Subject wise compilation of 60 Day Plan- 2016

It is a pandemic outbreak Means Disease outbreak occurs over a wide geographic area
and affects a very high proportion of the population.
Swine influenza is a respiratory disease that occurs in pigs that is caused by the Influenza
A virus.
Its a highly contagious disease that can easily spread from person to person.

Q.172) Consider the following statement with respect to TRAI (Telecom Regulatory
Authority of India)
1. TRAI is an apex body to regulate telecom services, including fixation/revision of tariffs
for telecom services which were earlier vested in the State government
2. Recently TRAI was accorded the status of Non banking financial company
Select the correct option
a) 1 only
b) 2 only
c) Both
d) None
Q.172) Solution (d)
The Telecom Regulatory Authority of India (TRAI) was, thus, established with effect from
20th February 1997 by an Act of Parliament, called the Telecom Regulatory Authority of
India Act, 1997, to regulate telecom services, including fixation/revision of tariffs for
telecom services which were earlier vested in the Central Government.
It is not a non banking financial company
Q.173) Virtual reality is making headlines nowadays. Consider the following statement
with respect to the term Virtual reality
Virtual reality is the creation of a virtual environment presented to our senses in such a
way that we experience it as if we were really there.
Medicine is one of the biggest beneficiaries with the development of virtual reality.
It is also called floating multimedia
Select the correct option
a) 1 and 2 only
b) 2 and 3 only
c) Only 1
d) All
Q.173) Solution (a)
Click here to know more about virtual reality

www.iasbaba.com

75

Current Affairs- Subject wise compilation of 60 Day Plan- 2016

This question can be asked in Mains from S&T part of GS 3, from prelims perspective
know the basics.

Q.174) The kind of unemployment when the economy experiences many ups and downs is
known as
a) Frictional unemployment
b) Structural unemployment
c) Cyclical unemployment
d) Seasonal unemployment
Q.174) Solution (c)
http://study.com/academy/lesson/three-types-of-unemployment-cyclical-frictionalstructural.html
Cyclical Unemployment
Over time, the economy experiences many ups and downs. That's what we call cyclical
unemployment because it goes in cycles. Cyclical unemployment occurs because of
these cycles. When the economy enters a recession, many of the jobs lost are
considered cyclical unemployment.
For example, during the Great Depression, the unemployment rate surged as high as
25%. That means one out of four people were willing and able to work, but could not
find work! Most of this unemployment was considered cyclical unemployment.
Eventually, unemployment came down again. As you can see, at least part of
unemployment can be explained by looking at the cycles, or the ups and downs of the
economy.
Q.175) Consider the following statements with respect to SAARC countries
1. SAARC is an economic and political organization of eight countries in Southern Asia that
includes Bangladesh, Bhutan, India, Mauritius, Nepal, Pakistan and Sri Lanka
2. The headquarters of SAARC is located in Kathmandu, Nepal
Select the correct option
a) 1 only
b) 2 only
c) Both
d) None
Q.175) Solution (b)
Mauritius is not a member of SAARC
Q.176) Consider the following statements with respect to Intended Nationally
Determined Contributions

www.iasbaba.com

76

Current Affairs- Subject wise compilation of 60 Day Plan- 2016


1. Intended Nationally Determined Contributions (INDCs) is a term used under the United
Nation Environment Programme (UNEP) for reductions in greenhouse gas emissions
2. As part of INDC India aims to reduce the Emissions Intensity of its GDP by 33 to 35 Per
Cent by 2030 from 2005 Level
Select the correct option
a) 1 only
b) 2 only
c) Both
d) None
Q.176) Solution (b)
Intended Nationally Determined Contributions (INDCs) is a term used under the United
Nations Framework Convention on Climate Change (UNFCCC) for reductions in greenhouse
gas emissions
http://pib.nic.in/newsite/PrintRelease.aspx?relid=128403
Q.177) Consider the following statements with respect to Renewable energy and India
1. India has the target to produce 1,00,000 MW of solar power by 2022.
2. India allows 49 % FDI in renewable energy sector to promote domestic industries as part
of Make in India campaign
Select the correct option
a) 1 only
b) 2 only
c) Both
d) None
Q.177) Solution (a)
India allows 100 % FDI in renewable energy sector
Q.178) Consider the following
1. Prithvi
2. Agni
3. BrahMos
4. Akash
Arrange the above missiles in decreasing order of their range
a) 1234
b) 2134
c) 2143

www.iasbaba.com

77

Current Affairs- Subject wise compilation of 60 Day Plan- 2016


d) 1243
Q.178) Solution (b)
Agni missile has the highest range of up to 8000 kms
Prithvi missile has the range up to 600 kms
BrahMos missile has the range up to 290 kms
Akash missile has the range up to 30 kms
Q.179) Consider the following statements with respect to Insider trading
a) Transactions of shares/security by someone having access to confidential information of
a listed company.
b) The trading of shares in between the promoters of a listed company to stop a possible
takeover.
c) The trading of virtual commodities on floor of commodity stock exchanges on the last
working days of the weeks.
d) None of the above
Q.179) Solution (a)
Insider trading is the buying or selling of a security by someone who has access to material,
non public information about the security.
Q.180) Bollgard technology is associated with which of the following crop?
a) Wheat
b) Rice
c) Cotton
d) Sun flower
Q.180) Solution (c)
It is associated with cotton
http://indianexpress.com/article/india/india-news-india/genetically-modified-technologyfiasco-in-the-name-of-the-farmer-2829660/
Q.181) Consider the following statement/s
a) Stapled Visa doesnt leave permanent trace on ones passport
b) China had issued stapled visa to residents of Indian J&K and Arunanchal Pradesh
c) Both correct
d) Both incorrect
Q.181) Solution (c)
http://www.business-standard.com/article/pti-stories/arunachal-mp-wants-to-visit-chinabut-not-on-stapled-visa-116050201229_1.html

www.iasbaba.com

78

Current Affairs- Subject wise compilation of 60 Day Plan- 2016

Q.182) Consider the following statements with respect to Medecins Sans Frontieres
(MSF)
1. It is also called by the name Doctors without Borders
2. It was formed aftermath of World War 2
Select the correct answer
a) 1 only
b) 2 only
c) Both
d) None
Q.182) Solution (a)
Mdecins Sans Frontires (MSF) or Doctors Without Borders, is an international
humanitarian-aid non-governmental organization (NGO) and Nobel Peace Prize laureate,
best known for its projects in war-torn regions and developing countries facing endemic
diseases. It was founded in France in response to the Biafran War. The organization is
known in most of the world by its localized name or simply as MSF; in Canada and the
United States the name Doctors without Borders is commonly used.
Q.183) The method of removing pollutants with the help of living organism like Bacteria is
known as
a) Bio magnification
b) Bio Remediation
c) Leaching
d) Bio Accumulation
Q.183) Solution (b)
Bio remediation is the use of microorganisms to degrade the environmental contaminants in
to less toxic forms.
Q.184) Consider the following statements with respect to Dispersion of light
1. The sequence VIBGYOR is in the order of increasing frequency.
2. The red light used in the traffic light is visible even in dense fog, because it has lowest
frequency in the VIBGYOR colour spectrum.
Select the correct option from code given below
a) 1 only
b) 2 only
c) Both
d) None

www.iasbaba.com

79

Current Affairs- Subject wise compilation of 60 Day Plan- 2016

Q.184) Solution (b)


The sequence VIBGYOR is in the order of decreasing frequency or increasing wave
length.
The light which has lowest frequency (Red) will bend slightly whereas light which has
more frequency will bend more (Violet)
Source 10th Ncert - 11th chapter
Q.185) Which of the following statements is/are correct regarding the Dedicated Freight
Corridors (DFC) project?
1) The project aims to reduce the unit cost of transportation and speed up the freight train
in India.
2) The Eastern DFC is funded by Japan while the Western DFC is funded by World Bank.
3) The freight trains in DFC will run completely on electricity, reducing the emission of
Green house gases.
Select the correct answer using the code given below:
a) 2 and 3 only
b) 1 and 3 only
c) 1 and 2 only
d) 1 only
Q.185) Solution (b)
Explanation:
The primary aim of the project is to reduce the unit transportation cost and speed up
the freight trains resulting in higher productivity.
The Eastern DFC extends from Dankuni near Kolkata to Ludhiana in Punjab, while the
Western DFC extends from the Jawahar Lal Nehru Port (JNPT) in Mumbai to Dadri near
Delhi.
The Western DFC is being funded by the Japan International Cooperation Agency (JICA)
up to 77 per cent of the total cost.
The Ludhiana to Mughalsarai section (1183 km) of the Eastern DFC is being funded by
the World Bank up to 66 per cent of the project cost.
The trains will run completely on electrcity and thus reduces the green house gas
emissions. Once implemented DFC will make Indian freight transport cheapest in the
world as they run on a Broad-gauage with double-stack using electric locomotives, which
no other country has achieved.
Q.186) Match List-I with List-II and select the correct answer using the code given below
the Lists :
www.iasbaba.com

80

Current Affairs- Subject wise compilation of 60 Day Plan- 2016


List-I (Hydroelectric power station)
A. Srisailam
B. Sabarigiri
C. Hirakud
D. Sileru

List-II (Location in the map)

a) A B C D : 3 1 4 2
b) A B C D : 3 4 1 2
c) A B C D : 2 4 1 3
d) A B C D : 2 1 4 3
Q.186) Solution (b)
Explanation:
Self-explanatory factual question
Q.187) According to the International Energy Agency (IEA), which region will be a big
source of crude oil imports for India as countrys oil demand is set to increase to five
million barrels per day by 2021?
a) Gulf region
b) The United States of America (USA)
c) Middle East region
d) Central Asia region
Q.187) Solution (c)
Explanation:
Self-explanatory factual question
Q.188) Consider the following
1. French
2. Arabic
3. Spanish
4. Chinese
Which of the following is an official language of United Nations?
a) 1 and 2 only
b) 2 and 3 only
c) 1, 3 and 4 only
d) 1, 2 3 and 4
Q.188) Solution (d)
http://www.un.org/en/sections/about-un/official-languages/

www.iasbaba.com

81

Current Affairs- Subject wise compilation of 60 Day Plan- 2016

Q.189) Consider the following


1. Argentina
2. Brazil
3. Marshall Islands
4. Nicaragua
Which of the countries are the members of MERCOSUR common market?
a) 1 and 2 only
b) 2, 3 and 4 only
c) 1, 3 and 4 only
d) All
Q.189) Solution (a)
https://www.google.co.in/search?client=opera&q=official+languages+of+united+nations&s
ourceid=opera&ie=UTF-8&oe=UTF-8#q=mercosur
Q.190) Energy statistics report is released by which of the following ministry
a) Ministry of Statistics and Programme
b) Ministry of Power
c) Ministry of New and Renewable energy
d) Ministry of Finance
Q.190) Solution (a)
It is released by Ministry of Statistics and Programme
http://mospi.nic.in/Mospi_New/upload/Energy_stats_2015_26mar15.pdf
Q.191) Consider the following
1. To study Pluto
2. To analyse Kupier belt
3. Space probe to study Solar flare
Select the correct option with respect to New Horizon mission
a) 1 and 2 only
b) 2 and 3 only
c) 1 and 3 only
d) All
Q.191) Solution (a)
https://www.nasa.gov/mission_pages/newhorizons/overview/index.html

www.iasbaba.com

82

Current Affairs- Subject wise compilation of 60 Day Plan- 2016


Q.192) Consider the following
1. Downs syndrome
2. Turner syndrome
3. Klinefelter syndrome
Which of the above chromosomal disorder is caused by the presence of extra
chromosomes?
a) 1 and 2 only
b) 2 and 3 only
c) 1 and 3 only
d) All
Q.192) Solution (c)
1) Downs syndrome and Klienfelter syndrome are two chromosomal disorders that have
extra set of chromosomes
2) Klinefelter syndrome - A genetic condition in which a male is born with an extra copy of
the X chromosome
3) Down syndrome is a chromosome disorder caused by the presence of an extra copy of
chromosome 21
Q.193) For quite some time, Nutrifarms are given a lot of boost by the Government of
India. Consider the following statements regarding the objective of Nutri farms:
1. Promoting the use of organic fertilizers
2. Introducing new crop varieties that is rich in Micro nutrients
Select the correct option
a) 1 only
b) 2 only
c) Both
d) None
Q.193) Solution (b)
http://www.downtoearth.org.in/news/nutrifarms-to-fight-hidden-hunger-40483
Q.194) Consider the following
1. Solid waste management
2. Digital literacy
3. E Gram connectivity
Which of the following services are included under Shyama Prasad Mukerji Rurban
mission?

www.iasbaba.com

83

Current Affairs- Subject wise compilation of 60 Day Plan- 2016


a) 1 and 2 only
b) 2 and 3 only
c) 1 and 3 only
d) All
Q.194) Solution (d)
To ensure an optimum level of development, fourteen components have been suggested as
desirable for the cluster, which would include; Skill development training linked to economic
activities, Agro Processing/Agri Services/Storage and Warehousing, Digital Literacy,
Sanitation, Provision of piped water supply, Solid and liquid waste management, Village
streets and drains, Street lights, Fully equipped mobile health unit, Upgrading school /higher
education facilities, Inter-village road connectivity, Citizen Service Centres- for electronic
delivery of citizen centric services/e-gram connectivity, Public transport., LPG gas
connections.
Q.195) Recently PRINCE 40, major utility equipment was making rounds in print media.
PRINCE 40 refers to
a) Solar Pump
b) Solar cooker
c) Solar Light
d) None of the above
Q.195) Solution (b)
This piece was in Kurukshetra magazine, PRINCE 40 refers to community solar cookers that
can be used for cooking large meals like mid day meals
Q.196) Which of the following is/are NOT correct about NMSA, one of the missions under
the NAPCC?
1) NMSA was aimed at transforming Indian agriculture into a climate resilient
production system through suitable adaptation and mitigation measures in the
domain of crops and animal husbandry.
2) National Mission on Food Security and the National Horticulture Mission are part of
NMSA
Select the appropriate code:
a) Only 1
b) Only 2
c) Both 1 and 2
d) Neither 1 nor 2
Q.196) Solution (b)

www.iasbaba.com

84

Current Affairs- Subject wise compilation of 60 Day Plan- 2016

Solution to this question is modified and corrected. Please refer below and update the
same.

Doubts: Baba ji plz clarify ques 24 option b.. It is not correct NFSM is separate, not a part of
NMSA..?????
Clarification:
There is a new umbrella scheme The Krishonnati Yojana (Refer IYB, 2016)
It is provided that both NMSA and NFSM are included in this scheme.
Q.197) Global energy architecture performance index report (GEAPI) is released by which
of the following international institutions.
a) World economic forum
b) United Nations development programme
c) United Nations environment programme
d) International renewable energy agency
Q.197) Solution (a)
World economic forum releases Global energy architecture performance index report
(GEAPI).
It explored the energy architecture of 126 countries based on their ability to provide
energy access across three dimensions of the "energy triangle" affordability,
environmental sustainability, security and access.
India has been ranked at 90 out of 126 countries.
Q.198) Which among the following statements is/are not correct?
1) Infant mortality rate (IMR) is the number of deaths of children under one year of age per
1000 live births.
2) Neonatal death the death of a baby before or during birth after 28 weeks of gestation.
Choose the appropriate code:
a) 1 only
b) 2 only
c) Both
d) None
Q.198) Solution (b)
Infant mortality: refers to deaths of young children, typically those less than one year of
age.
It is measured by the infant mortality rate (IMR), which is the number of deaths of
children under one year of age per 1000 live births.
www.iasbaba.com

85

Current Affairs- Subject wise compilation of 60 Day Plan- 2016

Stillbirth: the death of a baby before or during birth after 28 weeks of gestation
(according to WHO definition).
Neonatal death: the death of a baby within the first 28 days of life. (so option 2 is wrong)

Q.199) India is progressing on acquisition and exploration of the Farzad-B gas oilfields. In
which country does this oil field belong to?
a) Papua New Guinea
b) Iran
c) Turkmenistan
d) UAE
Q.199) Solution (b)
Link:
http://www.thehindu.com/business/Industry/farzadb-gas-project-deal-on-iran-gasfield-may-be-sealed-by-oct/article8687362.ece
Q.200) Which country is likely to set up an International Maritime Judicial Centre?
a) Japan
b) USA
c) China
d) India
Q.200) Solution (c)
Link:
http://www.business-standard.com/article/news-ians/china-to-build-internationalmaritime-judicial-centre-116031300070_1.html
Q.201) Though India has one of the largest coal reserves in the world, but the extraction of
Coal Bed Methane (CBM) is less than optimal because
1) India lacks in CBM related services.
2) Lack of infrastructure and CBM related technology.
3) Low gas prices in domestic market.
4) Most of CBM reserves are in tribal areas.
Select the correct answer using the code given below.
(a) 1, 2 and 3 only
(b) 1, 3 and 4 only
(c) 2 and 4 only
(d) 1, 2, 3 and 4
Q.201) Solution (d)
All the statements are correct factual question

www.iasbaba.com

86

Current Affairs- Subject wise compilation of 60 Day Plan- 2016


Q.202) Pattachitra is
a) A cloth-based scroll painting of Odisha
b) A traditional dance form of Odisha
c) Block painting of Buddhism faith
d) Paintings done on dry leaves and preserved
Q.202) Solution (a)
Pattachitra paintings:
Depict stories of Jagannath and of the Vaishnava Sect
Has earned a GI tag
Q.203) What are Mithrim Montes?
a) Dust coming from beyond our solar system
b) Mission to explore the subsurface oceans of Saturns icy moons
c) Mountainous ridges hosting Titans tallest peak
d) Experimental aircrafts to test the green aviation technology by the industry
Q.203) Solution (c)
Q.204) What do you mean by Google Tax?
a) A tax council to act as an advisory body with a research unit initiated by the web giant
Google
b) An equalisation levy to ensure that the online businesses which are global in nature are
taxed effectively
c) A special tax to be levied on foreign companies having their headquarters in India
d) The procedure enabling electronic filing of documents
Q.204) Solution (b)
http://www.dnaindia.com/money/report-google-tax-to-be-levied-from-june-1-here-s-allyou-need-to-know-about-it-2218380
Q.205) Consider the following statements:
1) Deposit amount on bank balance sheet (account book) is put under liability category.
2) Lent money is put under asset category
Select the correct option/s:
a) Only 1
b) Only 2
c) Both
d) None

www.iasbaba.com

87

Current Affairs- Subject wise compilation of 60 Day Plan- 2016


Q.205) Solution (c)
Deposit amount on bank balance sheet is put under liability category as banks have to
pay that amount back with interest.
Lent money is put under asset category as it generates income for the bank in the form
of interest.
Q.206) Who has headed the panel on restructuring of the Indian Railways?
a) Mr. Debroy
b) Mr. Khakodkar
c) Mr. Prakash Javdekar
d) Mr. Ashok Mehta
Q.206) Solution (a)

www.iasbaba.com

88

Current Affairs- Subject wise compilation of 60 Day Plan- 2016

Q.207) The ordinance making power of president lies with which of the following articles
of Indian constitution
a) Article 122
b) Article 123
c) Article 124
d) Article 125
Q.207) Solution (b)
Self explanatory
Q.208) Anuradhapura is an ancient Buddhist town found in which of the following
country
a) Thailand
b) Sri Lanka
c) Cambodia
d) Nepal
Q.208) Solution (b)
Anuradhapura is a ancient Buddhist site in Sri Lanka which was famous during the time
of Ashoka. Ashoka sent his emissarys to propagate Buddhism in Sri Lanka. Buddhas
relics can also be found in this place. PM modi visited Anuradhapura when he made a
trip to Sri Lanka last time.
www.iasbaba.com

89

Current Affairs- Subject wise compilation of 60 Day Plan- 2016

Q.209) Consider the following


1. Algae prepare food by the process of photosynthesis
2. Plants can synthesise not only carbohydrates but also proteins and fats
3. All plants are autotrophs
Which of the statements is true with respect to plants and algae
a) 1 and 2 only
b) 2 and 3 only
c) 1 and 3 only
d) All the above
Q.209) solution (a)
Not all plants are autotrophs. Plants like nephanthesis, picture plants are dependent on
other organisms for food and their survival.
Q.210) Consider the following
1. Copper Anaemia
2. Vitamin B1 Beri Beri
3. Vitamin C Scurvy
Which of the above mineral deficiency and its disorder is rightly matched
a) 1 and 2 only
b) 2 and 3 only
c) 1 and 3 only
d) All the above
Q.210) Solution (b)
Anaemia is caused by deficiency of iron and not by copper.
Q.211) Which of the following statements with respect to e Sahayog is correct
a) It is launched by the ministry of heavy industries to facilitate production of capital goods
b) It is launched by ministry of consumer affairs to facilitate new start ups in the area of
supply chain management
c) Launched by ministry of finance to facilitate taxpayers
d) Launched by law ministry to facilitate e filing of cases
Q.211) Solution (c)
Q.212) Consider the following countries
1. Myanmar

www.iasbaba.com

90

Current Affairs- Subject wise compilation of 60 Day Plan- 2016


2. Vietnam
3. China
4. Philippines
5. Malaysia
Mekong river flows through which of the above given countries?
a) 1,2,3 &4
b) 1,2,3 and 5 only
c) 1,2 and 3 only
d) 2&4 only
Q.212) Solution (c)
Refer atlas, it does not flow in Philippines and Malaysia
Q.213) Consider the following
1. Rashtriya Swasthya Bima Yojana (RSBY) insures people working in organised sector only
2. As per this scheme organised sector worker and his family (unit of five) will be covered
with total sum of Rs. 30,000/- per family per annum
Select the correct option
a) 1 only
b) 2 only
c) Both
d) None
Q.213) solution (d)
Rashtriya Swasthya Bima Yojana (RSBY) insures people working in unorganised sector
only
As per this scheme unorganised sector worker and his family (unit of five) will be
covered with total sum of Rs. 30,000/- per family per annum
Q.214) Consider the following
1. Sikkim is the first organic state in India
2. Indias highest peak Kanchenjunga is located in the state of Sikkim
3. River Teesta flows through Sikkim
Select the correct option with respect to the state of Sikkim
a) 1, 2 & 3
b) 1 & 2 only
c) 2 & 3 only
d) 1 & 3 only

www.iasbaba.com

91

Current Affairs- Subject wise compilation of 60 Day Plan- 2016

Q.11) solution (a)


Doubt- Isn't India's highest peak K2??
Officially it should be since we still claim Pok or China occupied Kashmir (where exactly K2
lies) as our part...
Clarification
We are aware of Wrong depiction of the map of India could soon land violators behind
bars, with a maximum term of seven years and a fine up to Rs 100 crore
Hail Mogambo (India)- K2 is the highest peak of India
So to hell with NCERT now :P Let UPSC take the call
Hence the answer is (d)
Q.215) Consider the following statements regarding Zika Virus
1. It is named after the Zika forest in Ethiopia
2. The mosquitoes that spread dengue and chikungunya viruses are the same affecting Zika
virus
3. It can be spread through sexual contact
Select the correct code
a) 1 and 2
b) 2 and 3
c) Only 2
d) 1, 2 and 3
Q.215) Solution (b)
Q.216) Consider the following statements with respect to Spaceward bound programme
1. Spaceward bound programme is a project by NASA
2. The objective is to train the next generation of space explorers.
3. India has joined hands with NASA in this experiment to conduct microbiological
expeditions in places like Ladakh that has extreme climatic conditions
Select the correct option
a) 1 & 2 only
b) 2 & 3 only
c) 1 & 3 only
d) All

www.iasbaba.com

92

Current Affairs- Subject wise compilation of 60 Day Plan- 2016


Q.216) Solution (d)
An expedition to Ladakh to study the similarities of certain parts of the regions
topography and microbial life to Martian surroundings.
India is part of Spaceward Bound programme for the first time.
Ladakh offers a high UV exposed, dry ecosystem with Mars analogue topological
features.
The Space ward Bound is a NASA project.
It educates future space explorers and funds expeditions to places with extreme climate
conditions.
The objective is to train the next generation of space explorers.
Q.217) Corruption perception index is a report released by which of the following
international organisations
a) World bank
b) Transparency international
c) United Nations development programme
d) Amnesty international
Q.217) solution (b)
It is released by Transparency International
Q.218) Nai Manzil is a government of India scheme related to development of which of
the following
a) Skill development
b) Recapitalisation of banks
c) Health
d) Tourism
Q.218) solution (a)
Under the scheme girls from minority communities will be imparted three month skill
development training in seven identified sectors relevant to the region
Q.219) Mobile academy, IT initiative of government of India is related to which of the
following option
a) Its a mobile-based application aimed at providing training services to 9 million ASHAs.
b) It will be an IT-enabled tool to help tobacco users to quit tobacco.
c) It is an audio-based mobile service that delivers weekly audio messages to families
about pregnancy, child birth and child care.
d) It is a Mobile based application for filing quick IT returns
Q.219) Solution (a)
www.iasbaba.com

93

Current Affairs- Subject wise compilation of 60 Day Plan- 2016


Its a mobile-based application aimed at providing training services to 9 million ASHAs. This
will aid in enhancing their inter-personal skills. Once registered, ASHAs can access the 240minute course via their mobile phones.
Q.220) Consider the following
1. Argentina
2. Brazil
3. Uruguay
4. Paraguay
Which of the above countries does not belong to MERCOSUR Trade Bloc?
a) All belong to MERCOSUR
b) 1 and 2 only
c) 2 and 3 only
d) 3 and 4 only
Q.220) Solution (a)
http://commerce.nic.in/trade/international_ta_indmer.asp
Q.221) Consider the following statements with respect to Free Trade agreement
1. Goods
2. Services
3. IPR
4. Investment
Which of the above is included in FTA (Free trade agreement) between two countries?
a) 1 and 2 only
b) 2 and 4 only
c) 1 and 3 only
d) All
Q.221) Solution (d)
FTAs are arrangements between two or more countries or trading blocs that primarily
agree to reduce or eliminate customs tariff and non tariff barriers on substantial trade
between them.
FTAs normally cover trade in goods (such as agricultural or industrial products) or trade
in services (such as banking, construction, trading etc.).
FTAs can also cover other areas such as intellectual property rights (IPRs), investment,
government procurement and competition policy, etc.
Q.222) Consider the following

www.iasbaba.com

94

Current Affairs- Subject wise compilation of 60 Day Plan- 2016


1. Norway
2. Switzerland
3. Sweden
4. Iceland
Which of the above countries is a member of The European Free Trade association?
a) 1 and 2 only
b) 1,2 and 4 only
c) 2 and 4 only
d) 2 and 3 only
Q.222) Solution (b)
EFTA was in news yesterday as India is pushing to work out the modalities concerning to
finalising free trade Agreement with EFTA
To know more about EFTA http://www.efta.int/about-efta/european-free-tradeassociation

Q.223) Consider the following research institutes


1. National institute of Ocean technology is located in the state of Goa
2. National institute of Oceanography is located in Chennai
Select the correct statement with respect to location and research institute is concerned
a) 1 only
b) 2 only
c) Both
d) None
Q.223) Solution (d)
Location and research institute is interchanged
Q.224) Consider the following statements with respect to SAFTA
1. SAFTA or South Asian free trade agreement come under the ambit of ASEAN
(Association of South East Asian nations)
2. Pakistan is not a member of SAFTA
Select the correct statements
a) 1 only
b) 2 only
c) Both
d) None

www.iasbaba.com

95

Current Affairs- Subject wise compilation of 60 Day Plan- 2016

Q.224) Solution (d)


It comes under the ambit of SAARC
Pakistan is a member of SAFTA
It was signed during 2004 SAARC summit

Q.225) Consider the following


1. Durand line separates India and Pakistan
2. Siegfried line separates Germany and France
3. Mannerheim line separates Russia and Finland
Which of the above are correctly matched?
a) 1 and 3 only
b) 1 and 2 only
c) 2 and 3 only
d) All
Q.225) Solution (c)
Durand line is between Pakistan and Afghanistan
Q.226) Consider the following with respect to Upgrading Skills and Training in Traditional
Arts/Crafts Development (USTTAD) Scheme
1. It aims for capacity building of traditional artisans and craftsman belonging to minorities
community
2. The scheme is implemented by Ministry of Skill Development and Entrepreneurship
Select the correct answer
a) 1 only
b) 2 only
c) Both
d) None
Q.226) Solution (a)
It aims for capacity building of traditional artisans and craftsman belonging to minorities
community, it is implemented by Ministry of Minorities Affairs
Q.227) Consider the following statements with respect to first Jain Council
1. It was convened at Pataliputra in 3rd century BC by Sthalabahu
2. The first Jain council resulted in the compilation of 12 Angas

www.iasbaba.com

96

Current Affairs- Subject wise compilation of 60 Day Plan- 2016


Select the correct answer
a) 1 only
b) 2 only
c) Both
d) None
Q.227) Solution (c)
The first Jain Council was convened at Pataliputra by Sthalabahu,in the beginning of the
3rd century B.C. It resulted into the compilation of 12 Angas replacing the lost 14 purvas.
The second Jain Council was held at Valabhi in 5th century A.D. It resulted into he final
compilation of 12 Angas and 12 Upangas.
Q.228) Consider the following
1. Uzbekistan
2. Turkmenistan
3. Kazakhstan
Which of the above countries are bordering Aral Sea?
a) 1 and 2 only
b) 2 and 3 only
c) 1 and 3 only
d) All
Q.228) Solution (c)
Turkmenistan does not border Aral Sea.
Q.229) Consider the following
1. Lake Baikal
2. Lake Rukwa
3. Lake Balkhash
Which of the above lakes are found in Asia?
a) 1 and 2 only
b) 2 and 3 only
c) 1 and 3 only
d) All
Q.229) Solution (c)
L Rukwa is found in Africa
L Baikal in Russia
L Balkhash in Kazakhstan

www.iasbaba.com

97

Current Affairs- Subject wise compilation of 60 Day Plan- 2016

Q.230) Consider the following


1. Brown field projects
2. Greenfield projects
3. Stalled projects
Which of the above kind of projects is/are covered under National Investment and
Infrastructure Fund?
a) All
b) 1 only
c) 2 and 3 only
d) 1 and 3 only
Q.230) Solution (a)
The objective of NIIF is to maximise economic impact through infrastructure
development in viable projects both Greenfield and brownfield, including stalled
projects, mainly in the core infra sector.
NIIF has been structured as a fund of funds and set up as Category II Alternate
Investment Fund (AIF) under the Securities and Exchange Board of India (SEBI)
Regulations. Total corpus of the fund is Rs. 40000 Crore. The government will invest
Rs.20,000 crores into it from budget while the remaining Rs. 20,000 crores are expected
to come from private investors. Government stake has been fixed at 49%. This stake
structure (49% government, 51% private) will help NIIF to be seen with characters of
both sovereign fund as well as private sector.
http://arthapedia.in/index.php%3Ftitle%3DNational_Investment_and_Infrastructure_Fund_
(NIIF)
Q.231) Consider the following with respect to Vidyanjali scheme
1. The scheme is being implemented under the overall aegis of the Sarva Shiksha Abhiyan
2. The programme has been envisaged to bring together people willing to volunteer their
services at schools which really need them
3. It is implemented by Ministry of Home affairs
Select the correct statements
a) 1 and 2 only
b) 2 and 3 only
c) 1 and 3 only
d) All
Q.231) Solution (a)

www.iasbaba.com

98

Current Affairs- Subject wise compilation of 60 Day Plan- 2016

The Honble Prime Minister during the review of Sarva Shiksha Abhiyan on 22nd June,
2015 desired that the Ministry of Human Resource Development may explore the
possibility of young professionals being encouraged to volunteer their services in
Government schools. The Ministry of Human Resource Development recognizes the
unique place of volunteers in schools. School volunteers are mentors, confidantes and
communicators. Vidyanjali is an initiative to enhance community and private sector
involvement in Government run elementary schools across the country under the
overall aegis of the Sarva Shiksha Abhiyan. Through this initiative people from the Indian
Diaspora, retired teachers, retired government officials including retired defence
personnel, retired professionals and women who are home makers can volunteer at a
school that requests for one.
Vidyanjali will also cover initiatives under the Corporate Social Responsibility (CSR) and
Public Private Partnership (PPP), with Public Sector Undertaking (PSU) Companies,
private corporate and others.
The scheme is implemented by Ministry of Human Resource and Development.
http://vidyanjali.mygov.in/index.php/frontend/guideline
Q.232) Consider the following statements
1. Food produced from organic farming has high nutritional values
2. Organic farming promotes the use of crop rotations and cover crops
3. Use of pesticides are strictly prohibited under Organic farming
Select the incorrect code
a) 1 and 2
b) Only 1
c) 1 and 3
d) Only 3
Q.232) Solution (c)
Statement 1- Not a proved fact
Statement 3- No, some naturally occurring pesticides are used
http://www.business-standard.com/article/b2b-connect/organic-farming-a-boon-or-abane-115082800513_1.html
Q.233) Consider the following differences between HRIDAY and PRASAD Schemes by the
Government
1. PRASAD is under Tourism Ministry while HRIDAY is under Urban Development Ministry.
2. HRIDAY cities are chosen based on it elements governing faith, spirituality and religion
whereas PRASAD cities are chosen based on their heritage status.
Select the correct option exhibiting the correct difference/s between HRIDAY & PRASAD:

www.iasbaba.com

99

Current Affairs- Subject wise compilation of 60 Day Plan- 2016


a) Only 1
b) Only 2
c) Both 1 and 2
d) None of the above
Q.233) Solution (a)
HRIDAY cities are chosen based on their heritage status (focused on Infrastructure)
PRASAD cities are chosen based on it elements governing faith, spirituality and religion
(Improving the tourism ecosystem as a whole)
Cities: There are twelve cities under each scheme with 10 being common cities amongst
them. The different cities are
PRASAD: Kamakhya in Assam and Kedarnath in Uttarakhand
HRIDAY: Badami in Karnataka and Warangal in Telangana
Q.234) Consider the following statements:
1. The accompanying melody to this dance form is simply to express adoration for the
Supreme Being.
2. The final dance known as tillana, finds its origin in the tarana of Hindustani Music.
3. It heavily leans on abhinaya and is conducted by the Nattuvanar.
Choose the correct dance-form described above:
a) Kuchipudi
b) Bharatnatyam
c) Kathakali
d) Sattriya
Q.234) Solution (b)
Bharatnatyam (Ekaharya)
Elaborate dance procedure to please God
Source: Abhinaya Darpana by Nandikesvara
Sculpted: On the gopurams of the Chidambaram temple
Q.235) Which of the following statement/s is/are true?
1. FIIs are restricted from participating in commodities trading at exchanges.
2. A committee headed by Shanta Kumar suggested the possible merger of FMC with
SEBI, citing the reason that the structure of the FMC is not fully suited to the
challenges of an emerging market and needed to be overhauled.

www.iasbaba.com

100

Current Affairs- Subject wise compilation of 60 Day Plan- 2016


Choose the correct option:
a) Only 1
b) Only 2
c) Both 1 and 2
d) None of the above
Q.235) Solution (d)
Recently Reserve Bank (RBI) removed restriction imposed on foreign institution
investors or registered foreign portfolio investors on buying equity shares in the
commodity bourse.

Link:
http://www.financialexpress.com/markets/indian-markets/multi-commodityexchange-rallies-9-as-rbi-allows-fiis-to-buy-shares/263329/

A committee headed by Habibullah suggested the possible merger of FMC with SEBI,
citing the reason that the structure of the FMC is not fully suited to the challenges of an
emerging market and needed to be overhauled

Q.236) Which committee was asked to submit a report on the IPL?


a) Meiyappan Committee
b) Mudgal Committee
c) Lodha Committee
d) Kundra Committee
Q.236) Solution (b)
The Justice Mukkul Mudgal Committee was constituted to conduct an independent inquiry
over allegations of corruption, betting and spot-fixing in IPL matches. The report was
submitted in 2014.
Lodha Committee: BCCI Reforms
Q.237) Which of the following would be the first preferred category of preference in order
to acquire weapons as mentioned in the DPP 2016?
a) Fast-track Clearance Method
b) Superior Technology Accreditation Method
c) IDDM Method
d) Lowest Bidder Method
Q.237) Solution (c)
IDDM Method Indigenously Designed Developed & Manufactured
Minimum 40% of the product should be manufactured in India to qualify for this
category
Or constitutes minimum 60% indigenous content on cost basis of the contractual value

www.iasbaba.com

101

Current Affairs- Subject wise compilation of 60 Day Plan- 2016


Fast-track Clearance Method would now be also used by the Defence Acquisition Council
(earlier the usage was limited to the armed forces only)
DPP, 2016
A governing manual for defence procurement
Based on recommendations of: Former Home Secretary Dhirendra Singh
Objectives:
To ensure transparency & fast-track acquisition tap the best domestic technical
intellect invest in R&D Retain ownership of the applied technology
Give a push to the Make in India initiative Promote indigenisation
Q.238) River Mandakini is a tributary of
a) R. Ganga
b) R. Chandra
c) R. Bhagirathi
d) R. Alaknanda
Q.238) Solution (d)
R. Mandakini:
Originates from a Chorabari glacier near Kedarnath
Joins Alaknanda at Rudraprayag
R. Alaknanda
One of the two headstreams of the Ganga (Other: R. Bhagirathi)
Tributaries: R. Mandakini, R. Nandakini and R. Pindar
Highest Point: Nanda Devi
Mythology:
Believed to have split off from the celestial Ganges when Goddess Ganga descended from
heaven
Lake Satopanth on the Satopanth Glacier: Named after the Hindu trinity Lord Brahma, Lord
Vishnu, Lord Shiva
Panch Prayag: Several rivers in the Garhwal region merge with the Alaknanda at Panch
Prayag or 'holy confluence of rivers'
1. Vishnuprayag, where the Alaknanda is met by the Dhauliganga River
2. Nandaprayag, where it is met by the Nandakini River
3. Karnaprayag, where it is met by the Pindar River
4. Rudraprayag, where it is met by the Mandakini River
5. Devprayag, where it meets the Bhagirathi River and officially becomes the Ganges
Kedarnath: (Only PRASAD)
One of the four Chota Char Dham sites presence of one of the twelve Jyotirlingas of
Kedar or Lord Shiva
www.iasbaba.com

102

Current Affairs- Subject wise compilation of 60 Day Plan- 2016

Himalaya: Kumaon-Garhwal
River: R. Mandakini
News: 2013s devastating flood

Q.239) Which of the following hills host the Lomash Rishi cave, Sudama cave and Karan
Chaupar caves?
a) Amarkantak
b) Barabar
c) Nagarjuni
d) Pir Panjal
Q.239)Solution (b)
Barabar Caves
One of the oldest existing rock cut temples in India
Situated on the bank of the Phalgu river, Gaya in Bihar; in the twin hills of Barabar (4)
and Nagarjuni (3)
By Emperor Asoka and his grandson Dasratha
For Ajivika SectBelief in determinismdeeply ascetic sect (considered by many to be a
separate belief)
Lomas Rishi is one of the oldest rock-cut chambers in India
Q.240) This particular mineral has been discovered by the Curiosity rover on Mars
suggesting the fact that Mars has hosted explosive volcanoes in the past
a) Coal
b) Methane
c) Tridymite
d) Sphalerite
Q.240) Solution (c)
Tridymite is a high-temperature polymorph of Silica (SiO2)
Q.241) Consider the following statements:
1. One of only five elemental metals that is liquid at or near room temperature.
2. It is a hazardous material as a metal and its radioisotopes present a high health risk if
released into the environment.
3. It is a by-product of nuclear power and is highly soluble in water, making it ideal for
measuring the release of radioactive material into the ocean
Identify the element:
a) Caesium
b) Uranium
www.iasbaba.com

103

Current Affairs- Subject wise compilation of 60 Day Plan- 2016


c) Lead
d) Arsenic
Q.241) Solution (a)
http://www.thehindu.com/news/international/pacific-ocean-radiation-nears-prefukushimalevel/article8808697.ece?w=alauto
Q.242) This project will allow the government to track the account details and identify the
tax evaders
a) Pariksha
b) Insight
c) E-sahyog
d) Suvidha
Q.242) Solution (b)
Link: http://articles.economictimes.indiatimes.com/2015-07-28/news/64957817_1_taxevaders-data-analytics-nab-tax
Project Insight:
Insight is the Finance ministry's flagship project, aimed at widening the tax base by
catching tax evaders using technology. The ministry has floated a tender worth over Rs
150 crore to buy data analytics software and the infrastructure that goes along with it, a
person with knowledge of the matter said. The ministry has also attached a nondisclosure agreement to the tender to keep exact details on the project from getting
leaked.
The project will track the Permanent Account Numbers being quoted on financial
transactions and tally them with income tax filings.
e-Sahyog:
By Income tax department

To provide an online portal to help taxpayers resolve the mismatches in their returns, if
any

Q.243) SAMOA Pathway is concerned with which of the following?


a) Promotion of development of environmental friendly sports facilities
b) Convention on Biological Diversity
c) Small Island Developing States
d) Sustainable Pastoralism
Q.243) Solution (c)
Small Island Developing States Accelerated Modalities of Action (Samoa Pathway)

www.iasbaba.com

104

Current Affairs- Subject wise compilation of 60 Day Plan- 2016


Read More http://www.sids2014.org/index.php?menu=1537
Q.244) Which of the following crops are affected by the early blight disease?
1. Tomato
2. Potato
3. Rice
4. Coconut
Select the correct code
a) 1 and 2
b) Only 2
c) 3 and 4
d) None of the above
Q.244) Solution (a)
Alternaria solani is a fungal pathogen, that produces a disease in tomato and potato plants
called early blight. The pathogen produces distinctive "bullseye" patterned leaf spots and
can also cause stem lesions and fruit rot on tomato and tuber blight on potato.
Read More https://en.wikipedia.org/wiki/Alternaria_solani
Coconut
trees
hit
by
leaf
blight
disease
(In
News)
http://www.thehindu.com/news/national/tamil-nadu/coconut-trees-hit-by-leaf-blightdisease/article8695857.ece

Management of potato early blight (In News) http://www.thehindu.com/scitech/management-of-potato-early-blight/article7326203.ece


Q.245) Some hand pumps are being painted red in rural areas. What is the reason behind
doing this?
a) It indicates arsenic contamination in the aquifer
b) To protect the hand pump from rusting and corrosion
c) It indicates that the hand pumps is funded under NRDWP
d) To distinguish between hand pumps set up under Accelerated Rural Water Supply
Programme (ARWSP)
Q.245) Solution (a)
RED COLORED hand pumps indicate ARSENIC contamination in the aquifer.
Q.246) Consider the following Tiger Reserves and States
1. Pakke :: Arunachal Pradesh
2. Anamalai :: Kerala

www.iasbaba.com

105

Current Affairs- Subject wise compilation of 60 Day Plan- 2016


3. Periyar :: Tamil Nadu
4. Tadoba :: Maharashtra
Which of the following is incorrectly matched?
a) 1 and 2
b) 2 and 3
c) 1,2 and 4
d) None of the above
Q.246) Solution (b)
1. Pakke :: Arunachal Pradesh
2. Anamalai :: Tamil Nadu
3. Periyar :: Kerala
4. Tadoba :: Maharashtra
Q.247) According to Economic Survey 15-16, India is the biggest exporter of water. How is
this export being done?
a) Water is being exported from the Himalayas
b) Exports of crops that have high amount of water embedded in them
c) Breweries are taking up all the ground water and exporting it throughout the world
d) None of the above
Q.247) Solution (b)
In 2010, India exported about 25 cu km of water embedded in its agricultural exports. This is
equivalent to the demand of nearly 13 million people, says the survey.
Read More https://www.thethirdpole.net/2016/02/28/government-underlines-indiaswater-crisis/
In News http://www.livemint.com/Opinion/bPPHFHv19qBaA5qrPa6SuN/India-is-thebiggest-virtual-exporter-of-water.html
Q.248) Which city listed below is the capital of South Sudan?
a) Juba
b) Yambio
c) Bor
d) Pibor
Q.248) Solution (a)
Juba: Is witnessing escalated violence due to clashes between anti and pro-government
forces
Indian Step: Operation Sankat Mochan; For the safe evacuation of Indians
BBC-South Sudan conflict explained

www.iasbaba.com

106

Current Affairs- Subject wise compilation of 60 Day Plan- 2016

Q.249) Where will the jointly developed naval base by India and Seychelles be located?
a) Djibouti
b) Assumption Island
c) Somalia
d) Reunion Island
Q.249) Solution (b)
Once ready, the naval base to be built by the defence forces of India, and Seychelles will
help India exercise greater control over the Indian Oceans western region all the way to the
piracy-prone eastern African coastline.
http://indiatoday.intoday.in/education/story/naval-base-in-seychelles/1/557245.html

www.iasbaba.com

107

Current Affairs- Subject wise compilation of 60 Day Plan- 2016

Q.250) Which of the following statements are not correct about the Lakshadweep islands?
1. They are all coral islands.
2. Rainfall is very scarce
3. They are the extended part of the continent.
4. They have Indias only active volcano, i.e. Barren Island.
Select the code from below:
a) 1,2 and 4
b) 1,2 and 3
c) 2,3 and 4
d) All of the above
Q.250) Solution (c)
Lakshadweep islands have a coral origin.
Barren island lies in Andaman and Nicobar islands.

www.iasbaba.com

108

Current Affairs- Subject wise compilation of 60 Day Plan- 2016

Q.251) Consider the following statements:


1. Weather is the average atmospheric condition of an area over a considerable time.
2. Climate is a short term condition of temperature, pressure and humidity.
Which of the above statements are correct?
a) 1 only
b) 2 only
c) Both 1 and 2
d) Neither 1 nor 2
Q.251) Solution (d)
Climate is the average atmospheric condition of an area over a considerable time.
Q.252) Consider the following:
Nomads Place
1. Bindibu Australia
2. Bedouin Arabia
3. Tuaregs Sahara
4. Bushmen Kalahari
Which of the above are correctly matched?
a) 2 only
b) 1,2 and 4
c) 2,3 and 4
d) All of the above
Q.252) Solution (d)
Self Explanatory
Q.253) Consider the following statements regarding Gobi Desert:
1. Gobi desert has a scarcity of water as snow which falls there evaporates directly through
sublimation.
2. Bactrian Camel survives in Gobi desert by directly eating ice/snow which most of the
other animals cant do.
Which of the above statements are correct?
a) 1 only
b) 2 only
c) Both 1 and 2
d) Neither 1 nor 2

www.iasbaba.com

109

Current Affairs- Subject wise compilation of 60 Day Plan- 2016

Q.253) Solution (c)


Gobi desert is a cold desert in Mongolia. Precipitation mostly occurs in the form of snowfall.
Although the quantity of snowfall is good enough, the land does not get the water as it
evaporates directly through sublimating i.e. without getting converted into liquid water.
This is the reason why there is scarce vegetation.
Bactrian camel has adapted to this environment. They can directly eat snow and store
water.
Q.254) Consider the following statement regarding Mediterranean climatic region:
1. During summers, the trade winds are offshore and there is practically no rainfall.
2. They have open woodlands, of which Cork Oak trees are the best known. Cork oaks are
valued for their thick barks used for making corks of wine bottles.
3. Animal farming is an important activity because of soft nutritious grass.
Which of the above statements are correct?
a) 1 and 2
b) 2 and 3
c) 1 and 3
d) All of the above
Q.254) Solution (a)
During summers the ITCZ shifts northwards. The trade winds blow offshore the
Mediterranean region. Hence practically there is no summer rainfall in this region.
Conditions in Mediterranean do not suit grass, because most of the rain comes in the cool
season when growth is slow. Even if the grass survives they are wiry and bunchy that they
are not suitable for animal farming. Cattle rearing is thus unimportant in Mediterranean.
(G.C. Leong)
Q.255) Which among the following is correct about Indian monsoon rainfall?
a) The Indian summer monsoon rainfall is influenced by a system of oscillating sea surface
temperatures known as the Indian Ocean Dipole (IOD) in which the western Indian
Ocean becomes alternately warmer and then colder than the eastern part of the ocean.
b) A positive IOD occurs when the sea surface temperatures are greater than normal in the
Arabian Sea and less than normal in the tropical eastern Indian Ocean. When the reverse
is the case, a negative IOD is said to have developed.
c) A positive IOD leads to greater monsoon rainfall and more active (above normal rainfall)
monsoon days while negative IOD leads to less rainfall and more monsoon break days
(no rainfall).
d) All of the above

www.iasbaba.com

110

Current Affairs- Subject wise compilation of 60 Day Plan- 2016


Q.255) Solution (d)
Refer
:
http://www.thehindu.com/sci-tech/science/the-dipole-factor-in-summermonsoon-rainfall/article8828899.ece
Q.256) Drachma is the currency of
a) Australia
b) Denmark
c) Greece
d) Netherland
Q.256) Solution (c)
Let us revise some prelims-centric points about Greece
Officially: The Hellenic Republic
Location: South-eastern Europe
Strategically located at the crossroads of Europe, Asia, and Africa
Situated on the southern tip of the Balkan Peninsula
Shares borders with Turkey to the northeast (In News)
Capital: Athens
Seas: The Aegean Sea lies to the east of the mainland, the Ionian Sea to the west, and the
Mediterranean Sea to the south Greece has the longest coastline on the Mediterranean
Basin
Highest peak: Mount Olympus
Philosophy: SocratesAristotlePlato
http://iasbaba.com/2015/08/iasbabas-mains-issues-greece-crisis-de-mystified/

www.iasbaba.com

111

Current Affairs- Subject wise compilation of 60 Day Plan- 2016

Q.257) Consider the following components:


1. Aadhaar penetration
2. Basic bank account penetration
3. Banking Correspondents (BC) density
Choose the correct option that forms the indicator of JAM Preparedness Index
a) 1 and 2
b) 2 and 3
c) 1 and 3
d) All of the above
Q.257) Solution (d)
JAM Preparedness Index (Economic Survey)
Utility to measure states preparedness to implement
a) DBT in urban areas
b) DBT in rural areas
c) BAPU
JAM TrinityJan Dhan, Aadhaar, Mobile
Urban penetration is better than rural penetration
Major constraint: Last-mile financial inclusion of getting money from banks into peoples
hands especially in rural areas

www.iasbaba.com

112

Current Affairs- Subject wise compilation of 60 Day Plan- 2016


Penetration less than 50 per cent: Nagaland (48.9), Mizoram (38.0), Meghalaya (2.9) and
Assam (2.4; LOWEST)
Penetration above 75 per cent in: MP & Chhattisgarh
Consider:
First-mile (beneficiary identification),
Middle-mile (distributor opposition)
Last-mile (beneficiary financial inclusion)

Solutions:
Development of Banking Correspondent
Development of Mobile Money Networks
Activate models like BAPUBiometrically Authenticated Physical Uptake
Significant savings and efficiency gains can be achieved by transferring funds directly
from the state/central government to the worker rather than layer by layer (Centre
State District Block Panchayat), with leakages along the way
PAHAL scheme DBT in LPG
Transferring LPG subsidies via DBT
Reduced leakages by 24 per cent but occurrence of the exclusion of few genuine
beneficiaries
Identifies beneficiaries by linking households LPG customer numbers with Aadhaar
numbers to eliminate ghost and duplicate households from beneficiary rolls
Expenditure Information Network (EIN) development of IT systems and strong
coordination
BAPUBiometrically Authenticated Physical Uptake
Beneficiaries verify their identities through scanning their thumbprint on a POS (Point of
Sale) machine while buying the subsidised product
Activated in the Krishna district in Andhra Pradesh, with significant leakage reductions
Note:
97 per cent of LPG is consumed by the richest 30 per cent of households

www.iasbaba.com

113

Current Affairs- Subject wise compilation of 60 Day Plan- 2016

MGNREGS is one of the governments largest schemes, and forms 41 per cent of DBT
expenditure
National Food Security Act (NFSA) provides for subsidised grain to households but a cash
transfer maternal entitlement to mothers.
Jan Dhan is monitored at household level, while Aadhaar is an individual identifier
With DBT, subsidies are transferred to beneficiaries in cash whereas with BAPU,
beneficiaries certify their identity using Aadhaar and then physically take the subsidised
goods like today

Aadhaar
Brief:
Seeks to provide unique identification numbers to each individual in a country
Collection of demographic and biometric information
Usage of technology expansion of opportunities
A.P. Shah Committee report Safeguards regarding privacy w.r.t. Aadhaar; National
Security is the only ground on which a Competent Authority can share this information
Linked with: DBT Jan Dhan Yojna; help in financial inclusion drive and providing easy
access to loans.
E-commerce: Merchants are not allowed to collect Aadhaar information from general
public for printing Aadhaar card amount to a criminal offence punishable with
imprisonment
SC
Aadhaar not mandatory to receive benefits of the social welfare schemes
Prohibited the sharing the Aadhaar information with any agency
Government Identity Schemes
The NPR Scheme: 2004 Amendment of the Citizenship Act; Central govt. has the power to
compulsorily register citizens for an Identity Card
UIDAI: Set up through an executive notification.
Aadhaar (Targeted Delivery of Financial and Other Subsidies, Benefits and Services) Bill,
2016
Passed by Parliament
To provide efficient, transparent, and targeted delivery of subsidies, benefits and
services
Provides for the establishment of the Unified Identification Authority of India
Provides for establishment, operation and maintenance of the Central Identity Data
Repository
Aadhaar number cannot confer right of or proof of citizenship of domicile

www.iasbaba.com

114

Current Affairs- Subject wise compilation of 60 Day Plan- 2016


Is Aadhaar a Money Bill?
Govt. has challenged SC for examination of Speakers authority on passage of Adhaar as a
Money bill owing to a plea filed under Article 32 (Right to Constitutional Remedies) of the
Constitution (violation of the Basic Rule of Law enshrined in the Constitutionamendments
moved by Rajya Sabha MPs were rejected by Lok Sabha)
Centre has asked state govt. to link Aadhar with caste & domicile certificates issued to
school students
To expedite the delivery of grants and scholarship schemes to the deserving students
and avoid the unusual delays and harassment faced by these students in obtaining these
certificates separately
The responsibility for issuance of residency and caste certificates rests with the States
and Union territories
Q.258) Global Hunger Index Report is published by
a) FAO
b) IMF
c) International Food Policy Research Institute
d) World Economic Forum
Q.258) Solution (c)
Q.259) This navy vessel is the first open-ocean voyage of the Navys all-woman crew
a) Vaidehi
b) Mhadei
c) Padmavati
d) Rudramma
Q.259) Solution (b)
Indian Naval Sailing Vessel (INSV) Mhadei: From Goa to Port Louis in Mauritius
Q.260) Consider the following schemes:
1. Hamari Dharohar
2. KIRAN
3. USTAAD
4. Nai Manzil
Select the correct option that depicts schemes designed for the minority communities of
India
a) 2 and 3
b) 1, 2 and 3
c) 1, 2 and 4

www.iasbaba.com

115

Current Affairs- Subject wise compilation of 60 Day Plan- 2016


d) 1, 3 and 4
Q.260) Solution (d)
Hamari Dharohar: A scheme to preserve rich heritage of minority communities of India
under the overall concept of Indian Culture
Upgrading the Skills and Training in Traditional Arts/Crafts for Development (USTAAD):
To boost the skill of craftsmen, weavers and artisans who are already engaged in the
traditional ancestral work.
Nai Manzil in J&K: Girls from minority communities will be imparted three month skill
development training in seven identified sectors relevant to the regiontraining in
saffron processing, food processing, embroidery, computers IT (both software and
hardware), Tourism/hospitality, electronics and plumbing.
KIRAN (Knowledge Involvement in Research Advancement through Nurturing): To
address various issues related with women scientists (e.g. unemployment, relocation
etc.)
Q.261) Consider the following statements with respect to Reusable launch vehicle(RLV)
1. It is co developed by ISRO and DRDO
2. The RLV will be a dummy flight, i.e. The RLV will not be a powered flight
Select the correct option
a) 1 only
b) 2 only
c) Both
d) None
Q.261) Solution (b)
http://www.thehindu.com/sci-tech/science/indias-first-reusable-launch-vehicleexperiment/article8634050.ece
Q.262) Consider the following
1. Gwadar port is the only port in Iran that has the direct access to ocean located in close
proximity to Gulf of Oman
2. The Gwadar port will help India to bypass Pakistan and open up a route to land-locked
Afghanistan and Central Asia
3. With Gwadar port India hopes to compete with the Chinese, who are building Chabahar
port, in Pakistani Baluchistan.
Select the correct answer
a) 1 and 2 only
b) 2 and 3 only

www.iasbaba.com

116

Current Affairs- Subject wise compilation of 60 Day Plan- 2016


c) 1 and 3 only
d) None of the above
Q.262) Solution (d)
Gwadar port is in Pakistan and it is developed by Chinese and Chabahar port is in Iran
and it is developed in India.
Q.263) Consider the following statements with respect to Pradhan Mantri Jhan Dhan
Yojana
1. The scheme is run by Dept of Financial services Ministry of Finance
2. Anybody above the age of 10 can open the bank account with zero balance
3. The account holder can take loan benefit of Rs 50000 from the bank after six months
from opening the account.
Select the correct answer
a) 1 and 2 only
b) 2 and 3 only
c) 1 and 3 only
d) All the above
Q.263) Solution (a)
Click http://pmjandhanyojana.co.in/
The account holder can take loan benefit(over draft) of Rs 5000 from the bank after
six months from opening the account.
Q.264) Consider the following statements with respect to Kalaripayattu
1. It is a type of dance form originated in Kerala
2. It got developed during the time of Sangam age.
Select the correct one
a) 1 only
b) 2 only
c) Both
d) None
Q.264) Solution (b)
Kalaripayattu is a martial art form
Q.265)Jagor is a folk dance observed in which of the following states of India
a) Gujarat
b) Chattisgarh
c) Jharkhand

www.iasbaba.com

117

Current Affairs- Subject wise compilation of 60 Day Plan- 2016


d) Goa
Q.265) Solution (d)
Jagor, the traditional folk dance-drama, is performed by the Hindu Kunbi and Christian
Gauda community of Goa, to seek the Devine Grace for protection and prosperity of the
crop. Literal meaning of Jagor is jagran or wakeful nights. The strong belief is that the
night long performance awakens the deities once a year and they continue to remain awake
throughout the year guarding the village.
Q.266) Consider the following statements
1. Oligotropic lakes have more oxygen present in the hypolimnion (Bottom layer) than
eutrophic lakes
2. The total number of plants and animal species in eutrophic lakes is more than
oligotropic lakes.
Select the correct answer using the codes given below
a) 1 only
b) 2 only
c) Both
d) None
Q.266) Solution (a)
The total number of plants and animal species in eutrophic lakes is less than oligotropic
lakes cos of lack of oxygen.
Oxygen is required by all respiring plants and animals and it is replenished by
photosynthesis of green plants, as the oxygen levels are already low because of
population explosion and further oxygen taken up by microorganism which feed off the
dead algae during decomposition process.

Eutrophication or more precisely hypertrophication, is the ecosystem's response to the


addition of artificial or natural nutrients, mainly phosphates, through detergents,
fertilizers, or sewage, to an aquatic system. One example is the "bloom" or great
increase of phytoplankton in a water body as a response to increased levels of nutrients.
Negative environmental effects include hypoxia, the depletion of oxygen in the water,
which may cause death to aquatic animals.

Q.267) Consider the following statements with respect to National youth policy 2014
1. The National youth policy 2014 replaces National youth policy 2003 to reap the benefits
of Demographic dividend.
2. The National youth policy defines the age of youth from 18 to 29 years.

www.iasbaba.com

118

Current Affairs- Subject wise compilation of 60 Day Plan- 2016


Select the correct answer using the codes given below.
a) 1 only
b) 2 only
c) Both
d) None
Q.267) Solution (a)
The national youth policy defines the age of youth from 15 to 29 years.
President of India mentioned this in a speech Youth and Nation-Building on Jan 19th
2016.
Q.268) Consider the following statements with respect to Conference of Parties
1. It is organised by United Nations framework convention on climate change.
2. 2015 edition of COP was held in Nairobi Kenya.
3. It is conducted annually.
Select the correct answer using the codes given below.
a) 1 and 3 only
b) 2 only
c) 2 and 3 only
d) 3 only
Q.268) Solution (a)
The COP 21st edition was held in Paris.
The WTO ministerial conference was held in Nairobi Kenya.
The conference negotiated the Paris Agreement, a global agreement on the reduction of
climate change, the text of which represented a consensus of the representatives of the
196 parties attending it.
Q.269) Consider the following statements with respect to Sukanya samridhi yojana
1. It was launched as a part of Beti Bachao Beti padhao campaign.
2. The scheme is related to Girls health.
3. The account will remain operative for 21 years from the date of opening or marriage of
the girl child attaining 18 years of age, whichever is earlier.
Select the correct answer using the code given below.
a) 1 only
b) 2 and 3 only
c) 1 and 3 only
d) All the above

www.iasbaba.com

119

Current Affairs- Subject wise compilation of 60 Day Plan- 2016


Q.269) Solution (c)
Sukanya Samruddi Yojana
The Scheme is a small savings instrument that focuses on making girls financially secure.
It was launched on Jan 2015 by PM as part of Beti Bhacho,Beti Padhao
Sukanya samridhi account can be opened in the name of the girl child any time from her
birth till she attains the age of 10 with minimum deposited of RS 1000.
A maximum deposit of 1,50,000 can be deposited in a financial year and the account
holder girl child will be able to operate the account herself on attaining the age of 10
years.
The account will remain operative for 21 years from the date of opening or marriage of
the girl child after attaining the age 18 years of age, whichever is earlier.
The provision of not allowing withdrawal from the account till the girl child attains the
age of 18 years has been kept to prevent early marriage.
Q.270) Kalileh-wa-Dimneh is a Persian translation of which of the following Indian
works
a) Mahabharata
b) Ramayana
c) Panchtantra
d) Bhagavatgeeta
Q.270) Solution (c)
Kalileh-wa-Dimneh is a Persian translation of Panchatantra. PM Modi mentioned this at a
http://pib.nic.in/newsite/PrintRelease.aspx?relid=145578
Q.271) Consider the following
1. Diesel vehicles emit more carbon monoxide then petrol cars.
2. Diesel cars emit more nitrogen oxide air pollutants than petrol cars
Which of the following is/are the prime reason for banning of diesel cars?
a) 1 only
b) 2 only
c) Both
d) None
Q.271) Solution (b)
Diesel cars produce more nitrogen oxide air pollutants than petrol cars. The physical and
chemical conditions that exist inside any suchdiesel engines under any conditions differ
considerably from spark-ignition engines, because, by design, diesel engine power is not
controlled by the air/fuel mixture (as in most gasoline engines), but rather it is directly

www.iasbaba.com

120

Current Affairs- Subject wise compilation of 60 Day Plan- 2016


controlled by the fuel supply. For instance, diesel engines generally produce 28 times less
carbon monoxide than gasoline engines, as diesels burn their fuel in excess air even at full
load.
However, the lean-burning nature of diesel engines and the high temperatures and
pressures of the combustion process result in significant production of gaseous nitrogen
oxides (NOx), an air pollutant that constitutes a unique challenge with regard to their
reduction.[not verified in body] Total nitrogen oxides from petrol cars have decreased by
around 96% through adoption of exhaust catalytic converters as of 2012, while diesel cars
still produce nitrogen oxides at a similar level to those bought a decade and a half ago under
real world tests; hence, diesel cars emit around 20 times more nitrogen oxides than petrol
cars.
Q.272) The conservation of Galapagos Islands was in news recently. Consider the
following statements with respect to Galapagos Islands
1. Galapagos Islands are located off coast of Peru
2. It is a UNESCO world heritage site
3. Galapagos Islands passes through Equator
Select the correct option
a) 1 and 2 only
b) 2 and 3 only
c) 1 and 3 only
d) All
Q.272) Solution (b)
It is located off coast of Ecuador and it passes through equator. It was recognised as the
world heritage site in 1979.
Q.273) Recently a Cancer-causing chemical found was in bread samples from Delhi has
alarmed the authorities. Consider the following statements
1. Potassium bromate / Iodate are carcinogenic chemicals that are used while making
confectionary items.
2. Potassium Iodate/Bromate was found to cause tumour of the kidney and thyroid, and
cancer of the abdominal lining in laboratory animals.
Select the correct option
a) 1 only
b) 2 only
c) Both
d) None

www.iasbaba.com

121

Current Affairs- Subject wise compilation of 60 Day Plan- 2016

Q.273) Solution (c)


http://www.thehindu.com/todays-paper/tp-national/cancercausing-chemical-found-inbread-samples-from-delhi/article8638091.ece
Q.274) Consider the following
1. Mahanadi and Godavari
2. Krishna and Pennar
3. Ken and Betwa
4. Narmada and Krishna
Which of the following are the proposed inter basin water transfer links
a) 2, 3 and 4 only
b) 1, 3, and 4 only
c) 1, 2 and 3 only
d) All
Q.274) Solution (c)
http://www.nih.ernet.in/rbis/india_information/interlinking.htm
Q.275) Consider the following statements regarding Indias Foreign Trade Policy 2015-20
1. It aims at increasing Indias exports in goods and services to 900 billion by 2019-20.
2. Previous reward schemes of goods and services for export have been merged into two
schemes- Merchandise Export from India Scheme (MEIS) and Service Export from India
Scheme (SEIS).
3. It seeks to raise Indias share in World exports from 2% to 3.5%
Select the correct option
a) 1 only
b) 2 only
c) 1 and 2 only
d) All
Q.275) Solution (d)
http://www.dnaindia.com/money/report-11-features-of-the-new-trade-policy-you-mustknow-2073819
Q.276) Map of Africa

www.iasbaba.com

122

Current Affairs- Subject wise compilation of 60 Day Plan- 2016

Identify the countries located in the Map


a) 1-Niger ,2-Somalia, 3-Burkino Faso ,4-Alegeria
b) 1-Somalia ,2-Niger, 3-Algeria, 4-Burkino Faso
c) 1-Algeria, 2-Niger,3-Somalia,4-Burkino Faso
d) 1-Burkino Faso,2-Somalia,3-Algeria,4-Niger
Q.276) Solution (c)
Refer Atlas
Q.277) Consider the following statements regarding Amaravathi.
1. It was the capital of Satavahanas who ruled from 2nd century BCE to 3rd century CE
2. It is located on the banks of river Godavari.
3. Amaravati has been chosen as one of the sites for Heritage City Development and
Augmentation Yojana (HRIDAY) scheme of Government of India.
Choose the correct answer using the codes given below
a) 1 and 2 only
b) 1 and 3 only
c) 2 and 3 only
d) All the above
Q.277) Solution (b)
Amaravathi is an important historic town, and served as the capital of the Satavahana
kingdom in ancient days.
Sri Amaralingeswara Swamy temple is located at the Amararama Pancharama Kshetra
site in this village, which makes it a holy town for the Hindus. It is also a historic Buddhist

www.iasbaba.com

123

Current Affairs- Subject wise compilation of 60 Day Plan- 2016

site, and the Amaravati Mahachaitya stupa was built here between the 2nd century BCE
and the 3rd century CE.
Designated as a historic site by the Government of India, Amaravati has been chosen as
one of the sites for Heritage City Development and Augmentation Yojana (HRIDAY)
scheme of Government of India.
It is located on the banks of river Krishna.

Q.278) As per Ease of Doing Business Rank 2015, India ranked 130 out of 189 countries.
Which of the following parameters are used in calculating ease of doing business rank?
1. Starting a business
2. Getting electricity
3. Trading across borders
4. Getting credit
5. Protecting minority investors
Select the correct answer using the code given below.
a) 1, 2, 3 and 4 only
b) 1, 3 and 4 only
c) 1, 4 and 5 only
d) All the above
Q.278) Solution (d)
Starting a business
Dealing with construction permits
Getting electricity
Registering property
Getting credit
Protecting minority investors
Paying taxes
Trading across borders
Enforcing contracts
Q.279) Match list 1 with list 2 and choose the correct answer using the codes given below
1. BrahMos- A. Anti tank guided missile
2. AkashB. Stealth cruise missile
3. PrithviC. Surface to air missile
4. NAGD. Short range ballistic missile
Code
1234
a) B A C D

www.iasbaba.com

124

Current Affairs- Subject wise compilation of 60 Day Plan- 2016


b) A B D C
c) B C D A
d) D B C A
Q.279) Solution (C)
Self explanatory
Q.280) Consider the following
1. Jasrota
2. Kishtwar
3. Dachigam
Which of the above wild life sanctuary/national park is/are found in Jammu & Kashmir?
a) 1&2 only
b) 2&3 only
c) 1&3 only
d) All
Q.280) Solution (d)
All are found in the state of Kashmir
Q.281) Consider the following
1. Uttarakhand
2. Himachal Pradesh
3. Kerala
Montane type of climate is found in which of the following states?
a) 1 and 2
b) 2 and 3
c) 1 and 3
d) All
http://www.greaterkashmir.com/news/front-page/-ready-to-shift-sheep-farm-fromdachigam/220054.html
Q.281) Solution (a)
The elevation at which one habitat changes to another varies across the globe,
particularly by latitude. The upper limit of montane forests, the forest line or timberline,
is often marked by a change to hardier species that occur in less dense stands.
Uttarakhand, Arunachal Pradesh, Himachal Pradesh, Jammu and Kashmir are some of
the states where montane climate is observed

www.iasbaba.com

125

Current Affairs- Subject wise compilation of 60 Day Plan- 2016


Q.282) Consider the following
1. Spoon billed sandpiper
2. Ganges shark
3. Red panda
Which of the listed above species are critically endangered?
a) 1 and 2
b) 2 and 3
c) 1 and 3
d) All
Q.282) Solution (a)
Spoon billed sandpiper, Ganges shark, large tooth saw fish, hawksbill turtle are some of
the critically endangered species.
Red panda which is found in the state of Sikkim is endangered
http://www.bbc.com/news/uk-england-gloucestershire-36524786
Q.283) Global investment trend monitor report is released by which of the following
international organisations
a) WTO
b) UNCTAD
c) IMF
d) ADB
Q.283) Solution (b)
UNCTAD releases global investment trend monitor report
http://unctad.org/en/pages/publications/Global-Investment-Trends-Monitor-(Series).aspx
Q.284) Consider the following statement
1. Green climate fund was created under United Nations framework convention on climate
change
2. Global environment facility is a financial mechanism to help developing countries to
mitigate climate change and was created during recently held COP 21
Select the correct answer using the codes given below
a) 1 only
b) 2 only
c) Both
d) None

www.iasbaba.com

126

Current Affairs- Subject wise compilation of 60 Day Plan- 2016


Q.284) Solution (a)
The Global Environment Facility (GEF) was established as a pilot programme for
environmental protection. The current project cycle is GEF-6 over the years 2014-18. In
1992, when the Biodiversity and Climate Change Conventions were adopted at Rio de
Janeiro, the GEF was adopted as a financial mechanism for helping developing countries
meet their financing needs for achieving their climate change goals. As of November
2015, the GEF has directly invested a total of US$14.5 billion in 3946 projects in 167
countries, out of which US$4.2 billion is in 1010 projects for climate change mitigation.
Till date, India has received US$516.6 million of GEF grant, of which US$324.69 million is
for climate change mitigation projects while US$10 million is for climate change
adaptation projects.
The GCF was established as an operating entity of the financial mechanism of the
UNFCCC in 2011 and is expected to be a major channel for climate finance from
developed to developing countries.
Source Economic survey
Q.285) Palmyra, the ancient city was recently in news. In which of the following
countries is it located?
a) Syria
b) Iraq
c) Saudi Arabia
d) Israel
Q.285) Solution (a)
Factual question.
A questions is expected. Do read about it.
Q.286) International Intellectual Property Index is compiled by which of the following
organizations?
a) IMF
b) WTO
c) World Bank
d) US Chamber of Commerce
Q.286) Solution (d)
Q.287) Consider the following statements with respect to Nai Manzil
1. The scheme is launched by Skill Development Ministry
2. The scheme focuses on education and skill development of BPL and APL families
Select the correct option

www.iasbaba.com

127

Current Affairs- Subject wise compilation of 60 Day Plan- 2016


a) 1 only
b) 2 only
c) Both
d) None
Q.287) Solution (d)
The scheme is launched by Ministry of Minorities and it focuses on skill development of
minorities communities.
http://pib.nic.in/newsite/PrintRelease.aspx?relid=134694
Q.288) Shishu, Kishore, Tarun are terms associated with which of the following
government of India scheme
a) Loans issued under Start Up India
b) Development stages of Smart City Mission
c) Loans issued under MUDRA Scheme
d) Age based vaccines under Mission Indradhanush
Q.288) Solution (c)
The above stages are under MUDRA Scheme
http://pib.nic.in/newsite/PrintRelease.aspx?relid=118005
Q.289) Global financial literacy survey is a report released by which of the following
organization?
a) WTO
b) IMF
c) ADB
d) S&P rating services
Q.289) Solution (d)
It is released by S&P rating services
http://www.livemint.com/Money/2l9bZsVo7eempRU9yLvPAL/SP-survey-More-than-70Indians-fare-badly-in-financial-li.htm.
Q.290) Gender Development Index is a report released by which of the following
organizations?
a) UNDP
b) UNICEF
c) IMF
d) ILO

www.iasbaba.com

128

Current Affairs- Subject wise compilation of 60 Day Plan- 2016


Q.290) Solution (a)
It is released by UNDP
http://www.thehindubusinessline.com/economy/india-fares-poorly-in-un-humandevelopment-index/article6244720.ece.
Q.291) Consider the following statements with respect to Bilateral Totalization
Agreement (BTA)
1. It ensures that the tax charged for the exclusive purpose of social security in one country
is excluded from taxation
2. India has a BTA with USA but not Double Taxation Avoidance Agreement (DTAA)
3. USA has a BTA with only two Asian countries Japan and South Korea
Which of the following statements is/are correct?
a) Only 1
b) 1 and 2
c) 1 and 3
d) Only 3
Q.291) Solution (c)
A Totalisation agreement is a bilateral treaty between two countries that integrates the
social security laws of two countries. The purpose of the totalisation agreements is to
eliminate double social security taxation on citizen/residents of India who are posted to
foreign countries like US and second, to allow workers who divide their careers between
India and a foreign country to continue to be covered under Indias social security system.
India has a DTAA with USA but not BTA.
The US currently has Totalisation Agreements with 24 countries, the only two Asian
countries being South Korea and Japan.
India has signed social security agreements with Belgium, France, Germany, Switzerland,
Luxembourg and the Netherlands in the recent past.
Q.292) Match List I with List II and
the Lists:
List I
A. Sardar Vallabhbhai Patel
B. Pandit Deendayal Upadhyaya
C. Atal Bihari Vajpayee
D. Dr. B R Ambedkar

select the correct answer using the code given below


List II
1. Good Governance Day
2. Antyodaya Diwas
3. National Unity Day

A-B-C-D

www.iasbaba.com

129

Current Affairs- Subject wise compilation of 60 Day Plan- 2016


a) 1-( )-3-2
b) 3-2-1-( )
c) ( )-3-2-1
d) 2-3-( )-1
Q.292) Solution (b)
Sardar Vallabhbhai Patel National Unity Day
Pandit Deendayal Upadhyaya Antyodaya Diwas
Atal Bihari Vajpayee Good Governance Day
Q.293) Consider the following statements with respect to Rat Hole Mining.
1. It is mainly used for iron ore and coal mining in India
2. It is more prevalent in the state of Jharkhand
3. National Green Tribunal has issued a blanket ban on Rat Hole Mining
Which of the statements is/are incorrect?
a) Only 2
b) 1 and 2
c) 2 and 3
d) All of the above
Q.293) Solution (b)
Rat Hole Mining is concerned with only Coal Mining in the state of Meghalaya. NGT has
issued a blanket ban on the same.
Know More http://www.theatlantic.com/video/index/389607/the-horrors-of-rat-holemining/
In News http://www.thehindu.com/todays-paper/tp-in-school/why-are-miners-switchingoccupations/article7955682.ece
Q.294) Which of the following statements is not true about Mural Painting?
1) Murals are large works executed on the walls of solid structure
2) Murals are mainly found in natural caves and rock-cut chambers
3) Themes of Murals - Buddhist, Jain and Hindu religions
Select the correct code
a) 1 only
b) 1 and 3 only
c) 2 only
d) None of the above
Q.294) Solution (d)

www.iasbaba.com

130

Current Affairs- Subject wise compilation of 60 Day Plan- 2016


Explanation:
Recently, INTACH (Indian National Trust for Art, Culture and Heritage) found 400yr old
murals belonging to Nayaka period in a mantapa called Chitra Sabhai (painted using
plant pigments and natural dyes)
Nayaka Paintings (extension of Vijayanagara style) depicts episodes of Mahabharata and
Ramayana and also Krishna-leela (some stories also include Shiva and Vishnu)
Excerpt from The Hindu article
Above all the given statements are correct
Q.295) Consider the following statements:
1. NDRF has been established under Disaster Management Act, 2005.
2. It functions under the aegis of Union Ministry of Defence.
3. It is the only dedicated disaster response force of the world.
Select the incorrect option/s:
a) Both 1 and 3
b) Only 3
c) Only 2
d) Both 2 and 3
Q.295) Solution (c)
NDRF functions under the aegis of Union Ministry of Home Affairs.
Doubts: Babaji, in Q 24, NDRF is not the only disaster response team in the world no... US,
Japan and all have their own disaster response teams. Shouldn't the answer be D instead of
C.
Clarification: http://ndrfandcd.gov.in/cms/Ndrf.aspx
Q.296) Identify the correct function of the Treaty of Pelindaba:
a) Controls uranium supply from key mineral hubs in Africa
b) Forced Germany to accept complete responsibility for initiating World War I
c) Allowed Italy, Romania, Hungary, Bulgaria, and Finland to resume their responsibilities
as sovereign states in international affairs and to qualify for membership in the United
Nations post the end of World War II
d) Ended the War of 1812 between the U.S. and Britain
Q.296) Solution (a)
Treaty of Pelindaba:
Also known as the African Nuclear Weapon Free Zone Treaty
www.iasbaba.com

131

Current Affairs- Subject wise compilation of 60 Day Plan- 2016


Prohibits the research, development, manufacture, stockpiling, acquisition, testing,
possession, control or stationing of nuclear explosive devices in the territory of parties to
the Treaty and the dumping of radioactive wastes in the African zone by Treaty parties.
In News: Will not be allowed to supply uranium to India as India is not a member of the NPT
Q.297) The ethnic group Dinka and Nuer belong to which country?
a) South Africa
b) South Sudan
c) Botswana
d) Reunion Islands
Q.297) Solution (b)
In News: Crisis in South Sudan
Q.298) Recently we have started hearing more about Sleeper Cells in the country. What
are they?
a) A special initiative by the Railways to host mothers with infants during their journey in a
safe manner.
b) Underground groups of trained terrorists
c) Special investigative teams set-up by CCIT in the country to oversee safety
d) A special group in Kashmir lending its support to Burhan and his ideology
Q.298) Solution (b)
http://www.thehindu.com/news/national/is-may-have-set-up-sleeper-cells-in-india-iraqienvoy/article8845195.ece
Q.299) Identify the country/countries that had opposed Indias proposal for a
Comprehensive Convention on International Terrorism (CCIT)
1. European Union
2. U.S.A
3. Organisation of Islamic Countries
4. Latin America
Choose the correct option from the following:
a) Only 1
b) 2, 3 and 4
c) 1, 3 and 4
d) 3 and 4
Q.299) Solution (b)

www.iasbaba.com

132

Current Affairs- Subject wise compilation of 60 Day Plan- 2016


http://www.firstpost.com/india/sponsors-of-terrorism-must-be-made-accountable-saysindias-un-ambassador-syed-akbaruddin-2868750.html
Q.300) Consider the following statements:
1. Indias per capita consumption is less than the world average.
2. Indias per capita consumption is the highest among the BRICS nation.
Choose the correct option from the following:
a) Only 1
b) Only 2
c) Both 1 and 2
d) None of the above
Q.300) Solution (a)
Indias per capita consumption 1,070 kWh (2015-16)
1. Less than the world average
2. Lowest among the BRICS nation
Q.301) Consider the following statements:
1. UGC is a non-statutory body.
2. It was first formed in 1945 to oversee the work of two Central Universities of Aligarh and
Calcutta.
3. T.S.R. Subramanium Committee has recommended the law that has set up the higher
education regulator UGC be allowed to lapse.
Choose the incorrect option from the following:
a) Only 1
b) Both 1 and 2
c) Only 3
d) Only 2
Q.301) Solution (b)
UGC is a statutory body under Ministry of Human Resource Development
It was first formed in 1945 to oversee the work of three Central Universities of Aligarh,
Banaras and Delhi.
T.S.R. Subramanium Committee has recommended the law that has set up the higher
education regulator UGC be allowed to lapseUGC A Silent Spectator (19562016)
Q.302) Which sector will adopt the Shinkansen Technology as part of the assistance
offered to India by Japan?
a) Railways

www.iasbaba.com

133

Current Affairs- Subject wise compilation of 60 Day Plan- 2016


b) Roadways
c) Insurance
d) Yamuna Action Plan (YAP)
Q.302) Solution (a)
For the Bombay-Ahmedabad high-speed train project
http://www.forbes.com/forbes/welcome/#19ad6d035173
Q.303) Which country is the largest producer of sesame?
a) India
b) Japan
c) China
d) Pakistan
Q.303) Solution (a)
India: Largest producer as well as largest exporter of sesame seeds.
Q.304) Which organization has released the Energy and Air Pollution, World Energy
Outlook Special Report?
a) IMF
b) World Bank
c) UNEP
d) International Energy Agency
Q.304) Solution (d)
http://www.thehindu.com/data/premature-deaths-will-rise-minus-air-quality-normsiea/article8840532.ece
Q.305) Identify the wetland based on the following statements:
1. The Irrawaddy dolphin (Orcaella brevirostris) is the flagship species here.
2. It was the first Indian wetland of international importance under the Ramsar Convention.
3. Nalbana Island is the core area of this wetland.
Select the wetland that is best suited to the above description
a) East Calcutta Wetlands
b) Deepor Beel
c) Chilika Lake
d) Bhoj Wetland
Q.305) Solution (c)
Chilika Lake

www.iasbaba.com

134

Current Affairs- Subject wise compilation of 60 Day Plan- 2016

A brackish water lagoon; largest coastal lagoon in India and the second largest lagoon in
the world
Spread: Puri, Khurda and Ganjam districts of Odisha
Where: At the mouth of R. Dayaflowing into the Bay of Bengal
Birds: Hosts over 160 species of birds
Home to the only known population of Irrawaddy dolphins in India (Critically
endangered)

Q.306) Identify the country based on the following statements:


1. It was named in honour of King Philip II of Spain.
2. It is the seventh-most populated country in Asia and the 12th most populated country in
the world.
3. It hosts the headquarters of the Asian Development Bank
Select the country that is best suited to the above description
a) Brunei
b) Taiwan
c) Malaysia
d) Philippines

www.iasbaba.com

135

Current Affairs- Subject wise compilation of 60 Day Plan- 2016


Q.306) Solution (d)
Philippines
Situated on the Pacific Ring of Fire; prone to earthquakes and typhoons
Endowed with abundant natural resources and some of the world's greatest biodiversity
An archipelago composed of about 7,500 islands
Bordered by the Philippine Sea to the east, the South China Sea to the west, and the
Celebes Sea to the south
The Philippines is an active member of the
East Asia Summit (EAS),
Asia-Pacific Economic Cooperation (APEC),
Latin Union,
Group of 24,
Non-Aligned Movement
Also seeking to strengthen relations with Islamic countries by campaigning for
observer status in the Organisation of Islamic Cooperation
https://amti.csis.org/arbitration-101-philippines-v-china/

www.iasbaba.com

136

Current Affairs- Subject wise compilation of 60 Day Plan- 2016

The Permanent Court of Arbitration specified


China owes the Philippines and other countries more than $177 billion in rent and
damages for Chinas South China Sea fiasco there was no legal basis for China to claim
historic rights to resources in areas falling within its nine-dash line, which is based on a
vague map that emerged in the 1940s
The virtual line overlaps with waters also claimed by the Philippines, Brunei, Malaysia,
Taiwan and Vietnam
Mischief Reef is a low-water elevation and within Philippines exclusive economic zone
(Philippines indisputable legal rights to the reef)
China did not participate in the proceedings at the Permanent Court of Arbitration and
rejected the Tribunals decision

www.iasbaba.com

137

Current Affairs- Subject wise compilation of 60 Day Plan- 2016

www.iasbaba.com

138

Current Affairs- Subject wise compilation of 60 Day Plan- 2016


Q.307) The government of India scheme known as Anubhav refers to
a) Speedy redressal of Infrastructural projects
b) A platform for retiring employees
c) A new communication satellite on the anvil
d) None of the above
Q.307) Solution (b)
http://pensionersportal.gov.in/anubhav
Q.308) Recently Ministry of Water resources and Ganga rejuvenation appointed Ganga
task force for which of the following purposes?
a) To lay down guidelines to promote tourism in areas surrounding Ganga river
b) To ensure that industry and civilians do not pollute the river
c) To remove silt and other pollutants from Ganga river
d) To protect Ganges river dolphins
Q.308) Solution (b)
Ganga Task force is a bunch of Jawans who will be deployed on the banks of river Ganga to
ensure that industry and civilians do not pollute the river.
Q.309) Consider the following statements with respect to Indian rhino vision 2020
1. The Indian Rhino vision aims to increase the number one horned rhinoceros to 3000 by
2020.
2. Kaziranga national park houses nearly 75% of world rhinoceros
3. WWF (WWF-India and WWF AREAS) and the International Rhino Foundation (IRF) are
the two major partnering of Indian Rhino vision 2020
Select the incorrect statements
a) 2 only
b) 2 and 3 only
c) 1 and 3 only
d) All
Q.309) Solution (a)
https://www.savetherhino.org/asia_programmes/irv2020_india/indian_vision_2020_in_de
pth
Q.310) Consider the following statements with respect to Ancient Nalanda University
1. The university taught/helped propagates Hinayana school of Buddhism.
2. It was started by a ruler known as Kumaragupta.

www.iasbaba.com

139

Current Affairs- Subject wise compilation of 60 Day Plan- 2016


Select the correct answer with respect to Nalanda university
a) 1 only
b) 2 only
c) Both
d) None
Q.310) Solution (b)
All students at Nalanda studied Mahayana as well as the texts of the eighteen (Hinayana)
sects of Buddhism. Their curriculum also included other subjects such as the Vedas, logic,
Sanskrit grammar, medicine and Samkhya
Q.311) Consider the following
1. Defence of the body
2. Metabolic activities
3. Damage repair
Which of the following are the primary functions of proteins?
a) 1 and 2 only
b) 2 and 3 only
c) 1, 2 and 3 only
d) 1 and 3 only
Q.311) Solution (c)
Q.312) Recently it is found that sea ice cover surrounding Antarctica has been slightly
increasing in sharp contrast to drastic loss of ice occurring in the Arctic region. What can
be the possible reason for the same?
1. The topography of Antarctica and the depth of the ocean surrounding it
2. Topography of Antarctica and depth of ocean influence the winds and ocean currents to
drive the formation and evolution of Antarcticas sea ice cover and help sustain it.
Select the correct code
a) Only 1
b) Only 2
c) Both
d) None
Q.312) Solution (c)
The recent research revealed that as sea ice forms and builds up early in the sea ice growth
season, it gets pushed offshore and northward by winds, forming a protective shield of
older, thicker ice that circulates around the continent.

www.iasbaba.com

140

Current Affairs- Subject wise compilation of 60 Day Plan- 2016

The persistent winds, which flow down slope off the continent and are shaped by
Antarcticas topography, pile ice up against the massive ice shield, enhancing its thickness.
This band of ice, which varies in width from roughly 100 to 1,000 km, encapsulates and
protects younger, thinner ice in the ice pack behind it from being reduced by winds and
waves. The team also used QuikScat radar data to classify the different types of Antarctic
sea ice.
Older, thicker sea ice returns a stronger radar signal than younger, thinner ice does. They
found the sea ice within the protective shield was older and rougher (due to longer
exposure to wind and waves), and thicker (due to more snow accumulation).
As the sea ice cover expands and ice drifts away from the continent, areas of open water
form behind it on the sea surface, creating ice factories conducive to rapid sea ice growth,
http://www.thehindu.com/sci-tech/energy-and-environment/why-sea-ice-cover-aroundantarctica-isrising/article8634112.ece?utm_source=RSS_Feed&utm_medium=RSS&utm_campaign=RSS_
Syndication
Q.313) Which of the following are not affected by Solar Storm/flare?
1. Short wave radio communication
2. Lon range radio communication
3. Chromosphere
4. Ionosphere
5. Astronauts
Select the appropriate code
a) 1 and 2
b) 2 and 4
c) 1, 4 and 5
d) None
Q.313) Solution (d)
All are affected by Solar flare
The soft X-ray flux of X class flares increases the ionization of the upper atmosphere, which
can interfere with short-wave radio communication and can heat the outer atmosphere and
thus increase the drag on low orbiting satellites, leading to orbital decay.
X-rays and UV radiation emitted by solar flares can affect Earth's ionosphere and disrupt
long-range radio communications. Direct radio emission at decimetric wavelengths may
disturb the operation of radars and other devices that use those frequencies

www.iasbaba.com

141

Current Affairs- Subject wise compilation of 60 Day Plan- 2016


Solar flares affect all layers of the solar atmosphere (photosphere, chromosphere, and
corona)
The radiation risks posed by solar flares are a major concern in discussions of a manned
mission to Mars, the Moon, or other planets. Energetic protons can pass through the human
body, causing biochemical damage, presenting a hazard to astronauts during interplanetary
travel. Some kind of physical or magnetic shielding would be required to protect the
astronauts. Most proton storms take at least two hours from the time of visual detection to
reach Earth's orbit. A solar flare on January 20, 2005 released the highest concentration of
protons ever directly measured, giving astronauts as little as 15 minutes to reach shelter.
Q.314) Consider the following statements
1. Seawater is utilised to produce a solar fuel that is hydrogen peroxide (H2O2)
2. Chlorine in seawater is mainly responsible for enhancing the photo-catalytic activity
Select the correct code
a) Only 1
b) Only 2
c) Both
d) None
Q.314) Solution (c)
http://www.thehindu.com/sci-tech/new-method-can-generate-power-fromseawater/article8632993.ece?utm_source=RSS_Feed&utm_medium=RSS&utm_campaign=
RSS_Syndication
Q.315) Consider the following
1. Itai Itai
2. Yokakaichi Asthama
3. Minimata
Select the above ailments with their causing pollutants
a) 1 Mercury, 2 Cadmium, 3 Sulphur oxide
b) 1 Cadmium, 2 Sulphur oxide, 3 Mercury
c) 1 Sulphur oxide, 2 Cadmium, 3 Mercury
d) 1 Sulphur oxide, 2 Mercury, 3 Cadmium
Q.315) Solution (b)
Q.316) Consider the following
1. Transfat are saturated fatty acids

www.iasbaba.com

142

Current Affairs- Subject wise compilation of 60 Day Plan- 2016


2. They are formed during the process of addition of hydrogen atoms to oil
3. Trans fat are naturally found in nature
Which of the following statements is/are false with respect to Transfat?
a) 1 and 2 only
b) 1 and 3 only
c) 1 only
d) None
Q.316) Solution (c)
https://en.wikipedia.org/wiki/Trans_fat
Transfats are unsaturated and they are naturally found in Nature in milk, meat etc
Q.317) Consider the following conventions and select the wrongly matched convention
with its intent
a) Stockholm convention deals with persistent organic pollutants
b) Basel convention deals with the control of hazardous wastes and their disposal
c) Montreal conventions deals with preventing the use of substances that deplete ozone
layer
d) Rotterdam convention are dedicated to the protection of ozone layer
Q.317) Solution (d)
Rotterdam convention aims to promote shared responsibility and cooperative efforts
among parties in the international trade of certain hazardous chemicals in order to protect
human health and the environment from potential harm.
Q.318) Consider the following
1. India lies in close proximity with ring of fire
2. The Indian plate is moving northwards diverging with Eurasian plate
3. The Indian plate is moving northwards converging with Eurasian plate
4. Presence of active Volcanoes along Himalayan range
Which of the following statements does not explain the possible reason for presence of
earth quake in India?
a) 1 and 2 only
b) 2 and 3 only
c) 1, 2 and 4 only
d) All
Q.318) Solution (c)

www.iasbaba.com

143

Current Affairs- Subject wise compilation of 60 Day Plan- 2016


Indian plate is converging with Eurasian plate.
Q.319) Consider the following
1. Tiger
2. One horned rhinoceros
3. Elephant
4. Peacock
Which of the following animals are found in the ten rupees Indian currency note?
a) 1,2&4 only
b) 2,3&4 only
c) 1,3&4 only
d) 1,2&3 only
Q.319) Solution (d)
Peacock is not found
Q.320) The recent New Delhi Declaration is related to which of the following themes
a) Satellite coordination to monitor human induced greenhouse gases
b) To curb violence against women in tribal areas
c) To enhance scientific temper among school going students
d) To aim 100 % adult literacy by 2020
Q.320) Solution (a)
http://thewire.in/2016/06/04/isro-other-world-space-agencies-unite-to-monitorgreenhouse-gas-emissions-40777/
Q.321) Consider the following
a) Chimmonoy Wild life sanctuary
b) Parambikulam wild life sanctuary
c) Eravikilam national park
Which of the above wild life sanctuary/national park is not located in Kerala?
a) 1&2 only
b) 3 only
c) 2&3 only
d) All are present in Kerala
Q.321) Solution (d)
Refer atlas

www.iasbaba.com

144

Current Affairs- Subject wise compilation of 60 Day Plan- 2016


Q.322) GPS-Aided Geo Augmented Navigation (GAGAN) system will provide augmentation
service over which of the following areas?
1. Bay of Bengal
2. Middle East
3. South East Asia
4. Africa
5. Russia
6. Scandinavia
Choose the correct code
a) 1,2,3 and 4
b) 2,4,5 and 6
c) 3,4,5 and 6
d) 1 and 3 only
Q.322) Solution (a)
GAGAN will provide augmentation service for the GPS over the country, the Bay of Bengal,
South East Asia and Middle East and up to Africa
Read More http://www.thehindu.com/news/national/satellitebased-navigation-systemlaunched/article7418634.ece
Q.323) Consider the following statements with respect to Kaladan Multi-modal Transit
Transport Project
1. It is a project between India, Bangladesh and Myanmar
2. The project consists of only naval routes
3. It will reduce the need to transport goods through the Chickens neck
Which of the following is incorrect?
a) Only 1
b) Only 2
c) 1 and 2
d) 2 and 3
Q.323) Solution (c)
The project consists of road and naval routes
It will reduce the need to transport good through the narrow Siliguri corridor, also
known as Chicken's Neck
Siliguri Corridor https://en.wikipedia.org/wiki/Siliguri_Corridor
Kaladan
Multi-modal
Transit
Transport
Project

https://en.wikipedia.org/wiki/Kaladan_Multi-modal_Transit_Transport_Project

www.iasbaba.com

145

Current Affairs- Subject wise compilation of 60 Day Plan- 2016


Q.324) Consider the following statements regarding Channapatana handcraft
1. Channapatana toys are special type of wooden toys
2. They are protected by GI tag
3. Channapatana handcraft can be traced back to the reign of Tipu Sultan
4. It flourished with the assistance of Persian Art form
Select the correct code
a) 1, 2 , 3 and 4
b) Only 1
c) 1, 3 and 4
d) 1, 2 and 3
Q.324) Solution (a)
http://www.thehindu.com/news/cities/bangalore/bid-to-restore-channapatna-craft-toits-original-glory/article7941995.ece
Q.325) In a major boost to the sustainable development of the Eastern Ghats, with special
focus on its fragile environment, the United Nations University has sanctioned one more
Regional Centre of Expertise (RCE) to India. What are the RCEs in India?
1. RCE-Srinagar, working on western Himalayas
2. RCE-Guwahati on Eastern Himalayas
3. RCE-Gujarat on wetland ecosystems
4. RCE-TERI (Mumbai) on Youth empowerment and energy
5. RCE-Kodagu on traditional knowledge and tribal communities of Western Ghats.
6. RCE-Tirupati will work on Eastern Ghats, coastal communities, marine ecosystem,
biodiversity and sustainable development.
Select the correct code
a) 1, 2, 3, 4 and 6
b) 1, 2, 5 and 6
c) 2, 4 and 6
d) 1, 3, 4 and 5
Q.325) Solution (b)
RCE- Chandigarh on Wetland ecosystems
RCE- TERI (Goa)on Youth empowerment and energy
http://www.thehindu.com/news/national/andhra-pradesh/rcetirupati-to-focus-on-easternghats/article7983642.ece
Q.326) Consider the following with respect to Ganges River Dolphin
1. Ganges river dolphin is the national aquatic animal of India

www.iasbaba.com

146

Current Affairs- Subject wise compilation of 60 Day Plan- 2016


2. Ganges river dolphin is Vulnerable as per IUCN red list
3. Vikramshila Gangetic dolphin sanctuary is the only protected area for the endangered
Gangetic dolphins in Asia
Select the correct answer using the codes given below
a) 1 and 2 only
b) 2 and 3 only
c) 1 and 3 only
d) All
Q.326) Solution (c)
It is endangered as per IUCN red list
Vikramshila Gangetic dolphin sanctuary is the only protected area for the endangered
Gangetic dolphins in Asia
It is located in Bhagalpur District of Bihar, India. Only a few hundred dolphins remain in
India, of which half are found here. WWF-India and Aaranyak a NGO has been working
closely with various government departments to protect these blind river Dolphins of
India.
Q.327) Consider the following lakes
1. Lake Erie
2. Huron
3. Michigen
4. Ontario
5. Superior
Arrange the above lakes from West to East
a) 53124
b) 53134
c) 53214
d) 53241
Q.327) Solution (c)

www.iasbaba.com

147

Current Affairs- Subject wise compilation of 60 Day Plan- 2016

Q.328) Consider the following countries


1. Liechtenstein
2. Norway
3. Switzerland
Which of the above is a Non EU country?
a) 1 and 2 only
b) 2 and 3 only
c) 3 only
d) All
Q.328) Solution (d)
European countries that are not part of the European Union include Norway, Iceland,
Liechtenstein, Albania, Switzerland, Turkey, Russia, Macedonia andMontenegro. Of these,
two countries, Russia and Turkey, straddle Europe and Asia.
Q.329) Consider the following statement with respect to SHILP SAMPADA
1. The objective of this scheme is to upgrade the technical and entrepreneurial skill of
Backward Classes by way of providing training and financial assistance
2. The maximum loan limit under this scheme is Rs.1 Lakh.
3. The scheme is spearheaded by Ministry of Social justice and empowerment
Select the correct answer
a) 1 and 2 only
b) 2 and 3 only
c) 1 and 3 only
d) All

www.iasbaba.com

148

Current Affairs- Subject wise compilation of 60 Day Plan- 2016

Q.329) Solution (c)


http://nbcfdc.gov.in/en/shilp-sampada
Q.330) Which of the following properties of light is observed in the phenomena of
Rainbow Formation?
1. Dispersion
2. Refraction
3. Total internal reflection
4. Reflection
Select the correct answer using the options given below
a) 1,2,3 only
b) 2, 3, 4 only
c) 3 only
d) All of these
Q.330) Solution (d)
A rainbow is a natural spectrum appearing in the sky after a rain shower
It is caused by dispersion of sunlight by tiny water droplets, present in the atmosphere.
A rainbow is always formed in a direction opposite to that of the Sun.
The water droplets act like small prisms. They refract and disperse the incident sunlight,
then reflect it internally, and finally refract it again when it comes out of the raindrop
Due to the dispersion of light and internal reflection; different colours reach the
observers eye.
Source Ncert 10th std in 10 chap
Q.331) Consider the following statements with respect to cryogenic engine
1. In Cryogenic engine its fuel or oxidizer (or both) are gases liquefied and are stored at
very low temperatures.
2. Combination of liquid hydrogen (LH2) fuel and the liquid oxygen (LOX) oxidizer is one of
the most widely used fuels in the rocket
Select the correct option
a) 1 only
b) 2 only
c) Both
d) None
Q.331) Solution (c)

www.iasbaba.com

149

Current Affairs- Subject wise compilation of 60 Day Plan- 2016


http://www.isro.gov.in/gslv-d5-gsat-14/indigenous-cryogenic-engine-and-stage
Q.332) Consider the following statements with respect to GAGAN satellite
a) It is used for telemedicine in rural areas
b) It is used to provide live streaming of classes from top Indian universities
c) It is used to provide navigation facilities for civil aviation
d) It is used for rain and whether forecasting
Q.332) Solution (c)
http://www.isro.gov.in/applications/step-towards-initial-satellite-based-navigationservices-india-gagan-irnss
Q.333) Consider the following
1. Chandrayaan 1
2. Mars orbiter mission
3. Space capsule recovery experiment
Which of the above space missions are launched using PSLV launch vehicle?
a) 1 &3 only
b) 1&2 only
c) 2&3 only
d) All
Q.333) Solution (d)
The PSLV is one of world's most reliable launch vehicles. It has been in service for over
twenty years and has launched various satellites for historic missions like Chandrayaan-1,
Mars Orbiter Mission, Space Capsule Recovery Experiment, Indian Regional Navigation
Satellite System (IRNSS) etc. PSLV remains a favourite among various organisations as a
launch service provider and has launched over 40 satellites for 19 countries. In 2008 it
created a record for most number of satellites placed in orbit in one launch by launching 10
satellites into various Low Earth Orbits.
http://www.isro.gov.in/launchers/pslv
Q.334) Consider the following statements with respect to IRNSS (Indian regional
navigation satellite system)
1. It provide accurate position information service to users in India as well as the region
extending up to 1500 km from its boundary
2. IRNSS will provide two types of services namely, Standard Positioning Service (SPS) and
Restricted Service (RS) to all users
Select the correct answer

www.iasbaba.com

150

Current Affairs- Subject wise compilation of 60 Day Plan- 2016


a) 1 only
b) 2 only
c) Both
d) None
Q.334) Solution (a)
Restricted area service is for military and other defence related purposes
IRNSS is an independent regional navigation satellite system being developed by India. It
is designed to provide accurate position information service to users in India as well as
the region extending up to 1500 km from its boundary, which is its primary service area.
An Extended Service Area lies between primary service area and area enclosed by the
rectangle from Latitude 30 deg South to 50 deg North, Longitude 30 deg East to 130 deg
East.
IRNSS will provide two types of services, namely, Standard Positioning Service (SPS)
which is provided to all the users and Restricted Service (RS), which is an encrypted
service provided only to the authorised users. The IRNSS System is expected to provide a
position accuracy of better than 20 m in the primary service area.
http://www.isro.gov.in/irnss-programme
Q.335) Consider the following statements with respect to ISRO (Indian space research
organisation)
1. ISRO is the space agency of the Indian government headquartered at Sri Hari Kota
2. "Space technology in the Service of humankind." is the motto of ISRO
Select the correct option
a) 1 only
b) 2 only
c) Both
d) None
Q.3355) Solution (b)
ISRO is headquartered at Bengaluru
"Space technology in the Service of humankind." is the motto of ISRO
http://www.isro.gov.in/about-isro/genesis
Q.336) Where will the joint operations centre of the Islamic Military Alliance be
established?
a) Hyderabad
b) Islamabad
c) Riyadh
d) Cairo

www.iasbaba.com

151

Current Affairs- Subject wise compilation of 60 Day Plan- 2016

Q.336) Solution (c)


An intergovernmental military alliance of countries in the Muslim world united around
military intervention against ISIL and other counter-terrorist activities
Its primary objective is to protect the Muslim countries from all terrorist groups and
terrorist organizations irrespective of their sect and name
It would operate in line with the United Nations and Organization of Islamic Conference
(OIC) provisions on terrorism
Riyadh
The capital and largest city of the Kingdom of Saudi Arabia; also the capital of Riyadh
Province
Houses the largest all-female university in the world, the Princess Nora bint Abdul
Rahman University
Q.337) On 15 January 2016, gunmen armed with heavy weapons attacked the Cappuccino
restaurant and the Splendid Hotel in the heart of Ouagadougou wherein 28 people were
killed and many were injured. In which country does Ouagadougou lie?
a) Burkina Faso
b) Texas
c) Florida
d) France
Q.337) Solution (a)
Burkina Faso is a landlocked country in West Africa.
Q.338) Khankahs are
a) A Sufi convent
b) A retreat designed specifically for gatherings of Sufi brotherhood to meet, reside, study,
and assemble and pray together as a group in the presence of a Sufi master
c) Pillars carved in red sandstone
d) A type of pottery practiced during the Neolithic period
Q.338) Solution(b)
Q.339) Identify the State hosting Raas, Sankirtana and Thang-ta as its dance-forms
a) Meghalaya
b) Assam
c) Nagaland
d) Manipur

www.iasbaba.com

152

Current Affairs- Subject wise compilation of 60 Day Plan- 2016


Q.339) Solution (d)
Raas
Exhibits the transcendental love of Krishna and Radha and the Gopis devotion to the
Lord
Theme: Pangs of separation of the gopis and Radha from Krishna
Sankirtana
Accompanied with congregational singing
During all festivals and social functions
Thang-Ta: Martial dance form
Q.340) Consider the following statements:
1. INCOIS is an attached office of the Government of India, under the Ministry of Earth
Sciences.
2. Prime Ministers National Relief Fund consists of public contributions and budgetary
support in the 60:40 ratios respectively.
Choose the correct option/s:
a) Only 1
b) Only 2
c) Both 1 and 2
d) None of the above
Q.340) An annual Pitra-paksha mela takes place here
a) Bodhgaya
b) Varanasi
c) Puri
d) Gaya
Q.340) Solution (d)
Q.341) The Udayagiri-Khandagiri caves are a part of the heritage of which faith?
a) Hinduism
b) Buddhism
c) Jainism
d) Sati Cult
Q.341) Solution (c)
King: King Kharavela
Utility: Carved out as residential blocks for Jain monks

www.iasbaba.com

153

Current Affairs- Subject wise compilation of 60 Day Plan- 2016

Q.342) The assigned code name of Indias first nuclear bomb explosion in 1974 is:
a) Operation Blue Star
b) Operation Shakti
c) Smiling Buddha
d) Operation Crossroads
Q.342) Solution (c)
Operation Blue Star:
An Indian military operation which occurred between 3 June and 8 June 1984, ordered
by Prime Minister Indira Gandhi in order to establish control over the Harmandir Sahib
Complex in Amritsar, Punjab, and remove Jarnail Singh Bhindranwale and his armed
followers from the complex buildings.
The operation had two componentsOperation Metal, confined to the Harmandir
Sahib complex, and Operation Shop, which raided the Punjabi countryside to capture
any suspects
Roots: The Khalistan Movement was a political Sikh nationalist movement that wanted to
create an independent state for Sikh people, inside the current North-Western Republic of
India.
After-effects:
Led to the assassination of Prime Minister Indira Gandhi on 31 October 1984 by two of her
Sikh bodyguards triggering the 1984 anti-Sikh riots
Bombing of Ari India Flight 182 the largest mass murder in Canadian history and the
attack, considered to be a retaliation against India for the Operation Blue Star
Smiling Buddha: The first confirmed nuclear weapons test by a nation outside the five
permanent members of the United Nations Security Council
Operation Shakti: The second nuclear testPokhran-II was the series of five nuclear bomb
test explosions conducted by India at the Indian Army's Pokhran Test Range in May 1998
The Operation Crossroads series in July 1946, was the first post-war test series and one of
the largest military operations in U.S. history.
Q.343) Consider the following statements:
1. He was also known as Bacha Khan.
2. He founded the Khudai Khidmatgar ("Servants of God") movement in 1929.
3. He strongly opposed the All-India Muslim League's demand for the partition of India.
Identify the personality from the following options:
a) Badruddin Tyabji
b) Khan Abdul Gaffar Khan
c) Hakim Ajmal Khan
d) Maulana Abul Kalam Azad

www.iasbaba.com

154

Current Affairs- Subject wise compilation of 60 Day Plan- 2016

Q.343) Solution (b)


Khan Abdul Gaffar Khan:
He was also nicknamed as the "Frontier Gandhi" in British India
The Khudai Khidmatgar (Red Shirts) was founded on a belief in the power of Gandhi's
notion of Satyagraha, a form of active non-violence as captured in an oath.
He was one of the arrested during protests arising out of the Salt Satyagraha, 1930.
In 1962, Abdul Ghaffar Khan was named an "Amnesty International Prisoner of the Year"
Was awarded the Bharat Ratna, India's highest civilian award, in 1987
In News: Terror attack at Bacha Khan University
Badruddin Tyabji:
Was the first Indian solicitor
In 1902, he became the first Indian to hold the post of Chief Justice in Mumbai.
Was active in women's emancipation and worked to weaken the zenana system
A moderate Muslim; He along with Pherozshah Mehta, Kashinath Trimbak Telang and
others formed the Bombay Presidency Association in 1885
Hakim Ajmal Khan:
Was an Indian physician specialising in the field of South Asian traditional Unani
medicine as well as a Muslim Nationalist, politician and freedom fighter.
Participated in the Non-co-operation movement (Satyagraha)
Led the Khilafat Movement
Was one of the founders of the Jamia Millia Islamia University
Abul Kalam Azad:
The first Minister of Education in the Indian government
His contribution to establishing the education foundation in India is recognised by
celebrating his birthday as "National Education Day11th Nov" across India credited
with the establishment of the Indian Institutes of Technology and the foundation of the
University Grants Commission
The youngest person to serve as the President of the Indian National Congress (1923)
Q.344) Where did the Counter-Terrorism Conference-2016 take place?
a) New Delhi
b) Jaipur
c) Mumbai
d) Ahmedabad
Q.344) Solution (b)

www.iasbaba.com

155

Current Affairs- Subject wise compilation of 60 Day Plan- 2016


http://pib.nic.in/newsite/PrintRelease.aspx?relid=136023
Q.345) Vinicius is the mascot for Rio Olympics 2016. Consider the following statements
regarding Vinicius:
1. Vinicius is a mix of different Brazilian animals.
2. The name of the Olympic mascot pays tribute to Brazilian musician Vinicius de Moraes.
Which of the above statements are correct?
a) 1 only
b) 2 only
c) Both 1 and 2
d) Neither 1 nor 2
Q.345) Solution (c)
Vinicius
Name
The name of the Olympic mascot pays tribute to Brazilian musician Vinicius de Moraes.
Description
Vinicius is a mix of different Brazilian animals. His design takes inspiration from pop culture,
as well as video game and animation characters. Alongside his Paralympic Games colleague,
Vinicius represents the diversity of the Brazilian people and culture, as well as its exuberant
nature.
Q.346) Which of the following Indian lakes have saline water?
1. Wular
2. Chilka
3. Sambhar
4. Pangyong
5. Loktak
Select the code from below:
a) 1,2 and 3
b) 2,3 and 4
c) 3,4 and 5
d) 2 and 3
Q.346) Solution (b)
Wular lake- One of the largest fresh water lakes in Asia and the largest in India, is located in
Jammu and Kashmir.
Located in Bandipora district in the Indian state of Jammu and Kashmir.

www.iasbaba.com

156

Current Affairs- Subject wise compilation of 60 Day Plan- 2016


Chilka Lake-Spread over the Puri, Khurda and Ganjam districts of Odisha state on the east
coast of India.
Largest coastal lagoon in India and the second largest lagoon in the world.
Largest wintering ground for migratory birds on the Indian sub-continent. The lake is home
to a number of threatened species of plants and animals.
The lake is an ecosystem with large fishery resources. It sustains more than 150,000 fisher
folk living in 132 villages on the shore and islands.
Sambhar Lake -India's largest inland salt lake.
A bowl shape lake encircles historical Sambhar Lake Town located 96 km south west of the
city of Jaipur (Northwest India) and 64 km north east of Ajmer along National Highway 8 in
Rajasthan.The Salt Lake was owned by various rulers of Jaipur and Jodhpur before it was
leased to the British.
Situated 65 kilometers from Jaipur, the lake is also a major hub for the production of salt.
Bird lovers are also in for a treat here as Sambhar is the best place/town/sanctuary for bird
watching.
The main wildlife found at the Sambhar Wildlife Sanctuary includes flamingos, storks,
sandpipers and redshanks.
Pangong- It is an endorheic lake in the Himalayas situated at a height of about 4,350 m
(14,270 ft). It is 134 km (83 mi) long and extends from India to Tibet.
Approximately 60% of the length of the lake lies in Tibet.
The lake is 5 km (3.1 mi) wide at its broadest point.
During winter the lake freezes completely, despite being saline water.
Loktak Lake: Located near Moirang in Manipur state, India.
Largest freshwater (sweet) lake in North -East India.
Famous for the phumdis (heterogeneous mass of vegetation, soil, and organic matters at
various stages of decomposition) floating over it.
Keibul Lamjao the only floating national park in the world floats over it.
Q.347) Consider the following statements:
1. German Government has recently introduced an initiative One World, no Hunger.
2. NABARD has entered into collaboration with German Government for a special
programme on Soil Protection and Rehabilitation for Food Security.
Which of the above statements are correct?
a) 1 only
b) 2 only
c) Both 1 and 2
d) Neither 1 nor 2

www.iasbaba.com

157

Current Affairs- Subject wise compilation of 60 Day Plan- 2016

Q.347) Solution (c)


The National Bank for Agriculture and Rural Development (NABARD) has entered into
collaboration with German Government for a special programme on Soil Protection and
Rehabilitation for Food Security. The programme is part of the German governments
recently introduced special initiative One World, No Hunger initiative. As part of the
collaboration NABARD is anchoring two programmes viz.
Soil conservation and rehabilitation of degraded land programme in Madhya Pradesh
and Maharashtra.
Innovation of watershed development and climate change adaptation programmes in
five states Telangana, Andhra Pradesh, Gujarat, Maharashtra and Rajasthan.
Q.348) Consider the following statements:
1. Order of the Rising Sun is the highest civilian award of Japan.
2. Former bureaucrat and Parliamentarian, N K Singh, has been selected for this prestigious
award.
Which of the above statements are correct?
a) 1 only
b) 2 only
c) Both 1 and 2
d) Neither 1 nor 2
Q.348) Solution (c)
Former bureaucrat and parliamentarian Nand Kishore Singh has been selected for
prestigious Order of the Rising Sun, Gold and Silver, Japans highest civilian award. He was
chosen for this prestigious award for promoting economic, educational and cultural ties
between India and Japan. About Order of the Rising Sun It is a Japanese Government highest
civilian established in 1875 by Emperor Meiji. It is conferred in recognition of distinguished
accomplishments of an individual. The modern version of the order is being given to nonJapanese recipients from 1981.
Q.349) Consider the following statements:
1. Brazilian Selvas are not included in the list of Biodiversity hotspots of the world.
2. They do not satisfy the condition of having at least 1500 species of vascular plants
(>0.5% of the worlds total) as endemic.
Which of the above statements are incorrect?
a) 1 only
b) 2 only
c) Both 1 and 2

www.iasbaba.com

158

Current Affairs- Subject wise compilation of 60 Day Plan- 2016


d) Neither 1 nor 2
Q.349) Solution (b)
Selvas are not included in the list of Worlds Biodiversity Hotspots.
According to CI, to qualify as a hotspot a region must meet two strict criteria: it must
contain at least 1,500 species of vascular plants (> 0.5% of the world's total) as endemics,
and it has to have lost at least 70% of its original habitat.
Selvas do not satisfy the second condition. Although currently large scale destruction is
going on in those tropical rain forests.
Note: this is actually a criticism of the criteria of Hotspots. That in order to pay total
attention to conserve a biologically rich area, do we need to wait till it is 70% destroyed.
Q.350) Consider the following statements with respect to Missile technology control
regime (MTCR)
1. For signatories of missile technology control regime the maximum permissible payload
of a missile can be up to 1 ton
2. The range of missiles for signatories of MTCR should not exceed more than 300 Kms
Select the correct answer
a) 1 only
b) 2 only
c) Both
d) None
Q.350) Solution (b)
Q.351) Consider the following statements:
1. Colonies of snowflake coral have been recently documented off the coast of
Vishakhapatnam.
2. These snowflake coral (Carijoa riisei) are native species of India.
3. It inhabits reefs and underwater structures such as shipwrecks and piers, attaching itself
to metal, concrete and even plastic.
Select the correct option:
a) Only 3
b) 1 and 2
c) 2 and 3
d) None of the above
Q.351) Solution: (a)

www.iasbaba.com

159

Current Affairs- Subject wise compilation of 60 Day Plan- 2016


Friends of Marine Life (FML), a local NGO, has recorded the presence of several colonies of
the fast-growing alien species amid barnacle clusters on the rocky reef off the coast of
Kovalam in Thiruvananthapuram and Enayam, Kanyakumari.
A native of the tropical Western Atlantic and the Caribbean, C.riisei was first reported as an
invasive species from Hawaii in 1972. Since then, it has spread to Australia, Thailand,
Indonesia and the Philippines. It is considered an invasive species because of its capacity to
dominate space and crowd out other marine organisms. With its capacity to thickly settle
and occupy a variety of surfaces, C.riisei can destabilise the marine ecosystem
India: Gulf of Mannar, the Andaman and Nicobar Islands, Gulf of Kutch and Goa.
Q.352) Consider the following statements regarding the planets sixth mass biological
extinction event:
1. The number of rodents doubles and the abundance of the disease-causing ectoparasites
that they harbour decreases.
2. Megafauna face the highest rate of decline.
3. The number of invertebrate animals such as beetles, butterflies, spiders and worms has
increased by 20 per cent.
Select the incorrect option:
a) Only 3
b) Only 2
c) Both 1 and 3
d) All of the above
Q.352) Solution (c)
An era of Anthropocene defaunation the current die-off largely associated with human
activity
http://www.thehindu.com/sci-tech/energy-and-environment/biodiversity-loss-pushingearth-towards-sixth-mass-extinction/article6252886.ece
Q.353) Consider the following statements with respect to Swadesh Darshan scheme
1. The scheme aims for Integrated Development of Tourist Circuits around Specific
Themes.
2. Under this scheme 12 pilgrimage sites that were part of HRIDAY will be merged with
Swadesh Darshan scheme
Select the correct option
a) 1 only
b) 2 only
c) Both

www.iasbaba.com

160

Current Affairs- Subject wise compilation of 60 Day Plan- 2016


d) None
Q.353) Solution (a)
The second statement is the explanation of PRASAD scheme.
HRIDAY scheme works under Ministry of Urban development.
http://pib.nic.in/newsite/PrintRelease.aspx?relid=121258
Q.354) Consider the following
1. Amritsar
2. Amravati
3. Dwarka
4. Gaya
5. Kedarnath
6. Kamakhaya
Which of the following pilgrimage sites are included under HRIDAY SCHEME?
a) 1,2,3, and 5 only
b) 2,3,4 and 5 only
c) 1,2,3 and 4 only
d) All
Q.354) Solution (c)
All the places are included in the HRIDAY scheme except Kamakhaya and Kedarnath
Q.355) Indian Air force successfully test fired BrahMos missile from Pokran test range.
Consider the following statements with respect to BrahMos missile
1. It is a long range supersonic cruise missile
2. It consists of ram jet engine
3. It is co developed by India and Russia
Select the correct option
a) 2 and 3 only
b) 3 only
c) 1, 2 and 3 only
d) 1 and 2 only
Q.355) Solution (a)
It is a short range supersonic cruise missile
Q.356) Consider the following statements with respect to Sansad Adarsh Gram Yojana

www.iasbaba.com

161

Current Affairs- Subject wise compilation of 60 Day Plan- 2016


1. Strengthening of local democracy through strong and transparent Gram Panchayat is an
important objective of SAGY
2. As per the scheme each MP is required to develop three model villages by 2019
Select the incorrect statement
a) 1 only
b) 2 only
c) Both
d) None
Q.356) Solution (d)
http://pib.nic.in/newsite/PrintRelease.aspx?relid=110430
Q.357) Consider the following
1. Australia Group
2. Wassaner Arrangement
3. Missile Control Technology Regime
4. Nuclear Non Proliferation Treaty
India is a member of which of the above treaties?
a) 1 and 3 only
b) 1 and 2 only
c) 3 only
d) None
Q.357) Solution (d)
The Australia Group is an informal group of countries (now joined by the European
Commission) established in 1985 (after the use of chemical weapons by Iraq in 1984) to
help member countries to identify those exports which need to be controlled so as not
to contribute to the spread of chemical and biological weapons.
The Missile Technology Control Regime (MTCR) is an informal and voluntary partnership
among 34 countries to prevent the proliferation of missile and unmanned aerial vehicle
technology capable of carrying a 500 kg payload for at least 300 km.
The Wassenaar Arrangement is a multilateral export control regime (MECR) with 41
participating states including many former COMECON (Warsaw Pact) countries. The
Wassenaar Arrangement was established to contribute to regional and international
security and stability by promoting transparency and greater responsibility in transfers
of conventional arms and dual-use goods and technologies, thus preventing destabilizing
accumulations.
India, Pakistan, Israel, South Sudan, North Korea are countries that have not signed NPT

www.iasbaba.com

162

Current Affairs- Subject wise compilation of 60 Day Plan- 2016


Q.358) Consider the following statements with respect to Indian nuclear test and its spill
over effects.
1. Operation Shakti was the assigned code name of India's first successful nuclear bomb
test on 1974.
2. The NSG was founded in response to the Indian nuclear test in 1974
3. Nuclear Supplier Group is a multinational body concerned with reducing nuclear
proliferation by controlling the export and re-transfer of nuclear materials
Select the correct option
a) All
b) 1 and 2 only
c) 2 and 3 only
d) 1 and only
Q.358) Solution (c)
Smiling Buddha was the assigned code name of India's first successful nuclear bomb test
on 1974.
The NSG was founded in response to the Indian nuclear test in May 1974 and first met in
November 1975. The test demonstrated that certain non-weapons specific nuclear
technology could be readily turned to weapons development. Nations already
signatories of the Nuclear Non-Proliferation Treaty (NPT) saw the need to further limit
the export of nuclear equipment, materials or technology.
Q.359) Recently conservation of Indigenous cow breeds were in news. Consider the
following statements with respect to cows.
1. Cows normally tend to produce two types of milk known as A1 and A2
2. A2 milk is considered to be richer and helps human beings to fight various diseases.
3. Indigenous cow breed like Gir, Sahiwal, Gersy normally produce A2 type of milk
Select the incorrect answer using the codes given below
a) 1 and 3 only
b) 3 only
c) 1 and 2 only
d) All
Q.359) Solution (b)
Gersy is not indigenous cow breed.
Q.360) Global Burden on Food borne diseases is a report released by which of the
following international agencies

www.iasbaba.com

163

Current Affairs- Subject wise compilation of 60 Day Plan- 2016


a) FAO
b) WHO
c) UNDP
d) WTO
Q.360) Solution (b)
The report is released by WHO
Q.361) Consider the following FDI contributors of India for 2015-16
1. Mauritius
2. Singapore
3. USA
4. Netherlands
5. Switzerland
Arrange the above countries in decreasing order of FDI in 2015-2016?
a) 1 2 3 5 4
b) 2 1 4 3 5
c) 2 1 3 4 5
d) 1 2 4 3 5
Q.361) Solution (c)
Singapore at US$ 10.99 billion, followed by Mauritius (US$ 6.12 billion), USA (US$ 3.51
billion), Netherlands (US$ 2.15 billion)
http://www.ibef.org/economy/foreign-direct-investment.aspx
http://dipp.nic.in/English/Publications/FDI_Statistics/2016/FDI_FactSheet_JanuaryFebruary
March2016.pdf
Q.362) Consider the following
1. Measles
2. Japanese Encephalitis
3. Rotavirus
The Mission Indradhanush, targets to immunize all children against vaccine preventable
diseases, namely
a) 1, 2 and 3
b) 2 and 3 only
c) 1 only
d) None of the above

www.iasbaba.com

164

Current Affairs- Subject wise compilation of 60 Day Plan- 2016


Q.362) Solution (c)
The health ministry will soon revamp its flagship immunisation programme 'Mission
Indradhanush' to include four new vaccines. The mission, currently providing coverage
against seven life-threatening diseases, will soon also include vaccines for rotavirus, measles
rubella, inactivated polio vaccine biavalent and Japanese Encephalitis for adults
Q.363) Pradhan Mantri Krushi Sinchai Yojana, PMKSY aims to bring rationality and
investment in irrigation and bring water in every field. Consider the following
1. Accelerated Irrigation Benefit Programme (AIBP)
2. Integrated Watershed Management Programme (IWMP)
3. On Farm Water Management (OFWM)
Pradhan Mantri Krishi Sinchana yojana is formulated by amalgamating which of the above
schemes
a) 1 and 2 only
b) 2 and 3 only
c) 1 and 3 only
d) All
Q.363) Solution (d)
http://pib.nic.in/newsite/PrintRelease.aspx?relid=124298
Pradhan Mantri Krishi Sinchayee Yojana (PMKSY) has been formulated amalgamating
ongoing schemes viz. Accelerated Irrigation Benefit Programme (AIBP) of Ministry of Water
Resources, River Development & Ganga Rejuvenation; Integrated Watershed Management
Programme (IWMP) of Department of Land Resources; and On Farm Water Management
(OFWM) component of National Mission on Sustainable Agriculture (NMSA) of Department
of Agriculture and Cooperation.
Q.364) Consider the following
1. Pradhan Mantri Awas Yojana also known as Housing for all aims to provide quality
homes at affordable prices by 2022.
2. The scheme is eligible only for BPL families
Select the correct option with respect to PMAY (Pradhan Mantri Awas Yojana)
a) 1 only
b) 2 only
c) Both
d) None
Q.364) Solution (a)

www.iasbaba.com

165

Current Affairs- Subject wise compilation of 60 Day Plan- 2016


The scheme is not only eligible for BPL families but also low income groups, economically
weaker section and women.
Q.365) Consider the following
1. Tamil Nadu
2. Assam
3. Kerala
4. Karnataka
Coffee is grown in which of the above states
a) 1, 2 and 4 only
b) 4 only
c) 3 and 4 only
d) 1, 3 and 4 only
Q.365) Solution (d)
Refer page number 37 in Oxford atlas
Doubt:
Coffee not grown in Assam? Oxford atlas vs Coffee board of India
http://www.indiacoffee.org/cof...
[Wiki]Coffee is grown in 3 regions of India with
1. Traditional coffee growing region of South India - Karnataka, Kerala and Tamil Nadu
2. New areas developed in the non-traditional areas of Andhra Pradesh and Orissa in the
eastern coast of the country
3. Third region comprising the states of Assam, Manipur, Meghalaya, Mizoram, Tripura,
Nagaland and Arunachal Pradesh of Northeastern India, popularly known as Seven
Sister States of India"
Clarification: Yes Assam should be included in the answer . So question should be
Q.5) Consider the following
4. Tamil Nadu
5. Assam
6. Kerala
7. Karnataka
Coffee is grown in which of the above states
a) 1, 2 and 4 only
b) 4 only
c) 3 and 4 only

www.iasbaba.com

166

Current Affairs- Subject wise compilation of 60 Day Plan- 2016


d) 1, 2, 3 and 4 only
Q.5) Solution (d)
Q.366) Consider the following
1. Horn of Africa
2. Gulf of Mannar
3. Sri Lanka
Which of the above is/are biodiversity hotspot?
a) 1 and 2 only
b) 2 only
c) 1 and 3 only
d) 2 and 3 only
Q.366) Solution (c)
List of Biodiversity Hotspots
1. North and Central America: California Floristic Province, Madrean pine-oak woodlands,
Mesoamerica
2. The Caribbean: Caribbean Islands
3. South America: Atlantic Forest, Cerrado, Chilean Winter Rainfall-Valdivian Forests,
Tumbes-Choc-Magdalena, Tropical Andes
4. Europe: Mediterranean Basin
5. Africa: Cape Floristic Region, Coastal Forests of Eastern Africa, Eastern Afromontane,
Guinean Forests of West Africa; Horn of Africa; Madagascar and the Indian Ocean
Islands; Maputaland-Pondoland-Albany; Succulent Karoo
6. Central Asia: Mountains of Central Asia;
7. South Asia: Eastern Himalaya, Nepal; Indo-Burma, India and Myanmar; Western Ghats,
India; Sri Lanka
8. South East Asia and Asia-Pacific: East Melanesian Islands; New Caledonia; New Zealand;
Philippines; Polynesia-Micronesia; Southwest Australia; Sundaland; Wallacea;
9. East Asia: Japan; Mountains of Southwest China
10. West Asia: Caucasus; IranoQ.367) Consider the following statements with respect to defects of eye and there
correction
1. Myopia is a condition of long sightedness
2. People with myopia use convex lens
Select the correct option from code given below
a) 1 only

www.iasbaba.com

167

Current Affairs- Subject wise compilation of 60 Day Plan- 2016


b) 2 only
c) Both
d) None
Q.367) Solution (d)

In a myopic eye, the image of a distant object is formed in front of the retina and not at
the retina itself. This defect may arise due to
Excessive curvature of the eye lens, or
elongation of the eyeball. This defect can be corrected by using a concave lens of
suitable power.

Q.368) Domestic electric wiring is basically a


a) Series connection
b) Parallel connection
c) Combination of series and parallel
d) Series connection with each room and parallel connection elsewhere.
Q.368) Solution (b)
Parallel connection because electricity can be supplied equally to all the instruments in a
house and also when one circuit
Q.369) Global energy architecture performance index report (GEAPI) is released by which
of the following international institutions.
a) World economic forum
b) United Nations development programme
c) United Nations environment programme
d) International renewable energy agency
Q.369) Solution (a)
World economic forum releases Global energy architecture performance index report

It explored the energy architecture of 126 countries based on their ability to provide
energy access across three dimensions of the "energy triangle" affordability,

Click here to know more about the rep

Q.370) Consider the following

www.iasbaba.com

168

Current Affairs- Subject wise compilation of 60 Day Plan- 2016


1. Project Mausam is transnational initiative meant to revive its ancient maritime routes
and cultural linkages between countries of the Indian Ocean world
2. It was launched at 38th World Heritage Session at Doha in 2012
Select the correct option
a) 1 only
b) 2 only
c) Both
d) None
Q.370) Solution (a)
Project Mausam is transnational initiative meant to revive its ancient maritime routes

It was lau

Q.371) Consider the following


1. Temperature
2. Pressure
3. Medium
What are the factors that affect the speed of sound from above?
a) Only 3
b) 1 and 3
c) 2 and 3
d) 1, 2 and 3
Q.371) Solution (d)
Here are the factors that affect the speed of sound:
Medium
Medium has a huge effect of the speed of sound. When most people discuss the speed of
sound they are talking about the propagation of sound waves through the medium of Air.
For anyone who has gone underwater and listen to people talking above it is likely that one
would notice the muted an odd way that voices sound underwater. This is because the
medium of water greatly bends, distorts and changes the speed of sound wave.
There is a whole aspect of science that measure and defines the effect of different mediums
(gaseous and liquid) on the speed of sound. This is called Fluid Dynamics. Underwater
communication is possible if you understand how this wave propagation as well as another
important factor (pressure).
Because of elasticity of materials sound will, as a rule of thumb, generally travel faster in
solids than in liquids and faster in liquids than in gases.

www.iasbaba.com

169

Current Affairs- Subject wise compilation of 60 Day Plan- 2016


Temperature
Temperature has a large effect on the speed of sound. Not as much as the Medium does,
but far more than anything else. Temperature affects the speed of sound because
temperature can affect the elastic qualities of different mediums. At the very basics lower
temperatures will decrease the speed of sound while higher temperatures will increase the
speed of sound, all other factors being equal.
Pressure
Pressure is the final factor that has a significant impact on the speed of sound. The effect of
pressure on the speed of sound is due to the materials inertial properties. In short, the more
pressure that is applied to the material or medium the denser it becomes and the greater
the inertia becomes. This makes any interactions between particles slower. Therefore the
speed of sound throughout the medium is slowed due to the greater pressure.
Q.372) Consider the following regarding Aerogels
1. They are liquid having low density
2. They have high thermal conductivity
3. Metal Oxide aerogels are the most common type of aerogels
Select the incorrect code
a) 1 and 2
b) 2 and 3
c) Only 3
d) 1, 2 and 3
Q.372) Solution (d)
Aerogels are solid having low density and low thermal conductivity. Silica aerogel is the
most common type of aerogel, and the most extensively studied and used.
http://www.thehindu.com/todays-paper/tp-national/paper-waste-turned-into-nontoxicaerogel/article8207322.ece
Q.373) Consider the following w.r.t Red Panda
1. It is an arboreal animal
2. It is categorized as Threatened under IUCN Red List
3. They eat mostly bamboo, and may eat small mammals, birds and eggs etc
Select the appropriate code
a) Only 1
b) 1 and 2
c) 1, 2 and 3
d) 1 and 3

www.iasbaba.com

170

Current Affairs- Subject wise compilation of 60 Day Plan- 2016

Q.373) Solution (d)


Doubt: Threatened is a class which is further subdivided into 3 subclass namely, 1) Critically
Endangered 2) Endangered and 3) Vulnerable. So technically statement 2 is correct.
Clarification: Absolutely correct. Answer should be
Q.373) Solution (c)
Arboreal means spending most time on trees.
Red pandas are excellent climbers, and forage largely in trees. They eat mostly bamboo, and
may eat small mammals, birds, eggs, flowers, and berries. In captivity, they were observed
to eat birds, flowers, maple and mulberry leaves, and bark and fruits of maple, beech, and
mulberry
http://www.thehindu.com/features/kids/whos-the-other-panda/article7109266.ece
http://www.iucnredlist.org/details/714/0
Q.374) Consider the following statements with Rare earth metals
1. As per IUPAC they are 17 rare earth metals
2. Among the rare earth metals there are 15 rare earth metals belonging to lanthanoid
group and other two being scandium and yttrium
3. China has the largest reserves of rare earth metals
Select the correct option
a) 1 &2 only
b) 2&3 only
c) 1&3 only
d) All
Q.374) Solution (d)
Rare earth metals:
As per IUPAC total 17 elements are considered as rare earth metal:the 15 from
lanthanoid group and two others:Scandium and Yttrium.
China produces 97% of world production
India has 2nd largest reserve of REM.
Mostly found in placer deposits
High end use, especially in electronics, tablet, wind and solar energy
Few examples: lanthanum, dyprosium and yttrium
Use:
lanthanum: oil refineries
dyprosium :used in hybrid vehicles, wind turbines and stealth helicopters
yttrium : Used in military jet engines

www.iasbaba.com

171

Current Affairs- Subject wise compilation of 60 Day Plan- 2016

Q.375) Consider the fund SMILE as mentioned in the budget 2016-17


a) It is debt fund by SIDBI that will be used to provide soft term loans for MSME sectors to
meet debt to equity norms
b) It is fund that will be used to provide rehabilitation care for women affected by violence
c) It is fund used to develop scientific temper among students in school
d) None of these
Q.375) Solution (a)
Budget 2016-17 announced two fund namely Indian aspiration fund and SMILE fund
India Aspiration Fund (IAF) to be set up as a fund of funds under the Small Industries
Development Bank of India (SIDBI) in order to boost the startup ecosystem in the country.
The fund will have an initial corpus of Rs.2,000 crore. Life Insurance Corp. of India (LIC) will
be a partner and co-investor in the fund.
Another Fund
The second fund is a debt fund called SIDBI Make in India Loan for Enterprises (SMILE),
which was announced in the Union budget in February. The fund will provide soft term
loans and loans in the nature of quasi-equity to MSMEs to meet debt-to-equity norms and
pursue growth opportunities in existing MSMEs.
Q.376) Consider the following statements with respect to Kanheri caves
1. It is located in close proximity to Sanjay Gandhi National park in Madhya Pradesh
2. Kanheri caves are constructed with the influence of Buddhism
Select the correct answer using the codes given below
a) 1 only
b) 2 only
c) Both
d) None
Q.376) Solution (b)
Kanheri National park is present in Maharashtra
Q.377) Consider the following
1. George Yule
2. Sarojini Naidu
3. Annie besant
4. Chadrashekar Azad
5. Badruddin Tyabji

www.iasbaba.com

172

Current Affairs- Subject wise compilation of 60 Day Plan- 2016


Select the above personalities who presided over the sessions of Indian National congress
a) 1 and 4 only
b) 1,2,3 and 5 only
c) 2,3,4 and 5 only
d) All presided over INC sessions
Q.377) Solution (b)
Chandra Shekar Azad did not preside over INC sessions
Q.378) Silambham is a Martial art patronised by which of the following ancient Indian
kingdoms
a) Cholas
b) Cheras
c) Pandyas
d) Pallavas
Q.378) Solution (a)
A martial art called Silambam was patronised by the Chola rulers. Ancient and medieval
Tamil texts mention different forms of martial traditions but the ultimate expression of the
loyalty of the warrior to his commander was a form of martial suicide called Navakandam
Q.379) Identify the lake:
1. It is a freshwater lake.
2. It hosts the only floating national park in the world.
3. It is also famous for its floating phumdis.
Choose the correct lake exhibiting the above-mentioned characteristics:
a) Renuka Lake
b) Kolleru lake
c) Loktak Lake
d) Pong Dam Lake
Q.379) Solution (c)
Loktak Lake in Manipur
Largest freshwater lake in the north-eastern region of the country
Phumdis: heterogeneous mass of vegetation, soil, and organic matters at various stages
of decomposition
Only floating national park in the world: Keibul Lamjao
Keibugl Lamjao National Park: the last natural refuge of the endangered Sangai or
Manipur brow-antlered deer

www.iasbaba.com

173

Current Affairs- Subject wise compilation of 60 Day Plan- 2016


Renuka Lake in Himachal Pradesh
A natural wetland with freshwater springs and inland subterranean karst formations
High religious significance and is named after the mother of Hindu sage Parshuram
Kolleru Lake in Andhra Pradesh A bird sanctuary
Pong Dam Lake in Himachal Pradesh A water storage reservoir created in 1975 on the R.
Beas, in the low foothills of the Himalaya on the northern edge of the Indo-Gangetic plain
(All the four above-mentioned are Ramsar sites)
Q.380) Identify the country practicing the following type of agriculture:
1. Farmers here have developed a unique hydroponics system in which plants can be
grown on the wateron the floating organic bed of water hyacinth, algae and other
plant residues.
2. This environmentally friendly traditional cultivation technique utilizes the natural
resources of wetlands to grow vegetables and other crops almost all year round and
provides social, economic, agricultural and ecological benefits to the local population.
3. Recently, it was designated by FAO as Globally Important Agricultural Heritage
Systems for innovation, sustainability and adaptability.
Choose the correct option
a) India
b) Bangladesh
c) Myanmar
d) Japan
Q.380) Solution (b)
Globally Important Agricultural Heritage Systems (GIAHS)
Launched by UNs Food and Agriculture Organisation (FAO) in 2002
Promotes public understanding, awareness, national and international recognition of
Agricultural Heritage systemsfostering an integrated approach combining sustainable
agriculture and rural development
Note
Three sites in Japan as well
1. Ayu of the Nagara River System:
Sustainable inland fisheries of a specific type of fish (Ayu)
Benefits from clean waters of the Nagara River (amongst the cleanest rivers)
Upstream forest management and local participatory approach
2. Minabe-Tanabe Ume System
Despite possessing nutrient-poor slopes: Production of high-quality Ume (Japanese
apricots) and various kinds of fruits with the help of local participation in maintaining

www.iasbaba.com

174

Current Affairs- Subject wise compilation of 60 Day Plan- 2016


upper coppice forests for landslide prevention and maintenance of water, and Japanese
honeybee as pollinators
3. Takachihogo-Shiibayama Mountainous Agriculture and Forestry System
Scarce flat-land (steep mountain) Local people balances timber production with
diverse farming activities
Activities: terraced rice growing, shiitake mushroom cultivation, beef cattle raising, or
tea cultivation
Forest: Mosaic of conifers and broadleaf trees
Q.381) Consider the following statements regarding Turkey
1. Istanbul is the capital of Turkey
2. Syria, Bulgaria and Georgia are few nations that border Turkey
Choose the correct answer using the code given below
a) 1 only
b) 2 only
c) Both
d) None
Q.381) Solution (b)
Refer Altas
Q.382) Consider the following statements regarding Moscow declaration.
1. It was signed during third BRICS ministerial conference held at Moscow
2. It is concerned with ending gender discrimination, and providing Equal access to women
in Education ,Employment etc
Choose the correct answer using the code given below
a) 1 only
b) 2 only
c) Both
d) None
Q.382) Solution (a)
BRICS nations signed Moscow Declaration for supporting Multilateral Science Projects
Purpose-For co-investment of resources for supporting multilateral Research and
Development (R&D) Projects in mutually agreed areas.
It calls for collaboration in:
Co-operation for large research infrastructures, including mega-science projects along
with coordination of the existing large-scale national programme of member countries.

www.iasbaba.com

175

Current Affairs- Subject wise compilation of 60 Day Plan- 2016

Implementation and development of a BRICS Framework Programme for funding joint


multilateral research projects, innovation and technology commercialization.
Establishment of BRICS Research and Innovation Networking Platform

Q.383) Consider the following statements:


1. The dependence on formal credit increases with the reduction in the landholding size.
2. Pradhan Mantri Fasal Bima Yojana will replace the existing two schemes National
Agricultural Insurance Scheme and the Modified National Agricultural Insurance Scheme.
3. Governments liability on premium subsidy would be shared between Centre and States,
on a 75:25 basis
4. In the post-harvest losses, coverage will be available up to a maximum period of 14 days
from harvesting for those crops which are kept in cut & spread condition to dry in the
field
Choose the incorrect statement/s from the following options:
a) Both1 and 4
b) Only 3
c) Both 1 and 3
d) None of the above
Q.383) Solution (c)
The dependence on informal credit increases with reduction in the landholding size.
Governments liability on premium subsidy would be shared between Centre and States;
on a 50:50 basis.
Q.384) Consider the following states of India:
1. Arunachal Pradesh
2. Assam
3. Manipur
4. Jammu & Kashmir
Choose the State/s from the following option which does not follow the IPC
a) Both 1 and 2
b) Only 2
c) Only 4
d) None of the above
Q.384) Solution (c)
J&K has enacted a separate code (A370) Ranbir Penal Code (RPC)
RPC is based on: IPC
The code was introduced during the reign of Dogra dynasty with Ranbir Singh as its ruler.

www.iasbaba.com

176

Current Affairs- Subject wise compilation of 60 Day Plan- 2016

Q.385) Consider the following instances:


1. When the punishment or sentence is by a court Martial
2. When there is a violation of a Fundamental Right
Choose from the following option that allows Supreme Court to exercise its pardoning
power
a) 1 only
b) 2 only
c) Both 1 and 2
d) Neither 1 nor 2
Q.385) Solution (b)
Supreme Court exercises its pardoning power
Under: Article 32
When:
When there is a violation of a Fundamental Right
When the situation arises which exists in the realm of Public Interest Litigation
Q.386) Which among the following is correct in regard to Geographical Indications of
Goods or GI tags?
1) Any established organisation or authority can apply for GI Tag after paying a certain fee.
2) Registration of GI is valid for a period of 10 years (can be renewed from time 2 time).
3) Registered GI is a Public Property, which belongs to the producer of goods cant be
licensed or mortgaged.
4) A number of treaties administered by WIPO (World Intellectual Property Organisation)
provide for the protection of GI.
Choose the appropriate code:
a) 1 only
b) 1 and 2 only
c) 1, 2 and 4 only
d) All of the above
Q.386) Solution (d)
A number of treaties administered by WIPO (World Intellectual Property Organisation)
provide for the protection of GI (such as Paris Convention for the protection of Industrial
Property 1883, Lisbon Agreement for protection of affiliations of origin and their
International Registration) and also Articles 22- 24 of TRIPS deal with international
protection of GI within the framework of WTO.

www.iasbaba.com

177

Current Affairs- Subject wise compilation of 60 Day Plan- 2016

Q.387) The elements of one of the following faith dominate Sittannavasal painting
identify the dominating faith from the options given below:
a) Brahanical
b) Jainism
c) Buddhism
d) Tantric Cult
Q.387) Solution (b)
Sittanavasal Caves represent one of the best cave paintings of early medieval India. These
are example of rock-cut architecture based on Jain thought and ideologies. They have a
close form of Ajanta and Bagh caves.
These paintings show the extent of the penetration of Jainism in South and also reflects that
Jainism was popular in the South during those days.
Q.388) Identify this Solar System object/body that are composed mainly of frozen
volatiles such as methane, ammonia and water
a) Asteroid
b) Meteors
c) Kuiper Belt Object
d) Satellites of Pluto
Q.388) Solution (c)
Sometimes called the EdgeworthKuiper belt, it is a circumstellar disc in the Solar System
beyond the planets, extending from the orbit of Neptune (at 30 AU) to approximately 50 AU
from the Sun.
Home to three officially recognized dwarf planets: Pluto, Haumea, and Makemake
Makemake: 2nd brightest icy dwarf planet after Pluto

www.iasbaba.com

178

Current Affairs- Subject wise compilation of 60 Day Plan- 2016

*New Horizons Path

Q.389) Identify the committee set up to simplify the provisions of the Income Tax Act,
1961
a) Abhijit Sen Committee
b) Justice R.V.Easwar Committee
c) Ashok Mehta Committee
d) Naresh Chandra Committee
Q.389) Solution (b)
Justice R.V.Easwar Committee

Abolition of 13 cesses levied by various ministries: To reduce multiplicity of taxes,


associated cascading and to reduce cost of collection

Rationalise Tax Deducted at Source (TDS) provisions for the Income Tax: To improve
cash flow position of small tax payers

Also, suggested several taxpayer-friendly measures to improve the ease of doing


business in the country, accelerate process of tax dispute resolutions and reduce
litigation.

Abhijit Sen Committee: Was set up to look into the Impact of Futures Trading on
Commodity Prices

www.iasbaba.com

179

Current Affairs- Subject wise compilation of 60 Day Plan- 2016


Ashok Mehta Committee: On PRIs
Naresh Chandra Committee: To review the defence management in the country and make
suggestions for implementation of major defence projects
Q.390) The Archaeological Survey of India was established by
a) Sir William Jones
b) Lord Cornwallis
c) James Princep
d) Alexander Cunningham
Q.390) Solution (d)
Alexander Cunningham: Father of Indian Archaeology
Archaeological Survey of India: Statutory body; works as an attached Office of the Ministry
of Culture
Q.391) Identify the tool that allows developing countries to raise tariffs temporarily to
deal with import surges or price falls
a) Special Drawing Rights
b) Special Safeguard mechanism
c) GATT
d) National Treatment Obligation
Q.391) Solution (b)
Refer

www.iasbaba.com

180

You might also like